Cardio Exam Master

Lakukan tugas rumah & ujian kamu dengan baik sekarang menggunakan Quizwiz!

A 28-year-old south Asian immigrant who is in her second trimester of her first pregnancy presents to the emergency department complaining of worsening dyspnea, orthopnea and lower extremity edema. She has never experienced anything like this before. She has no past medical history; however, she admits to frequent sore throats and ear infections as a child. Which of the following is most likely to be heard on auscultatory exam? A diastolic decrescendo murmur heard at the left lower sternal border A diastolic low-pitched decrescendo murmur best heard at the cardiac apex A holosystolic murmur heard best at the cardiac apex A systolic crescendo-decrescendo murmur best heard at the right upper sternal border

A diastolic low-pitched decrescendo murmur best heard at the cardiac apex would be the most likely auscultatory finding on exam. This woman likely has mitral stenosis secondary to rheumatic heart disease. Mitral stenosis encountered in women of childbearing age is nearly always rheumatic in origin. Maternal and perinatal complications during pregnancy in women with mitral stenosis reflect the unfavorable interaction between the normal cardiovascular changes of pregnancy and the stenotic mitral valve. Pregnancy is a high flow state and blood volume, cardiac output, and heart rate are increased. In pregnant patients with mitral stenosis, this increases the pressure across the mitral valve and can lead to pulmonary edema and other signs and symptoms of heart failure. Although mitral stenosis is relatively uncommon in developed countries, it is a common condition in pregnant women with heart disease in areas where rheumatic heart disease is prevalent. Although rheumatic heart disease may affect other valves, including aortic and tricuspid, the mitral valve is the predominant lesion and repeated attacks of rheumatic fever over time causes mitral valve stenosis.

Eisenmenger syndrome

An atrial septal defect or ventricular septal defect is most often associated with a left-to-right cardiac shunt, but in the presence of extremely elevated right heart pressures (which can occur with severe pulmonary hypertension) that exceed the left heart pressures, that shunt will convert to a right-to-left shunt. A right-to-left shunt is a "cyanotic" shunt because it results in venous (deoxygenated) blood from the right side of the heart being diverted back to the systemic circulation. The only truly definitive treatment of Eisenmenger syndrome is lung transplantation with shunt closure or combined heart and lung transplantation.

A 72-year-old woman is admitted to the hospital following a syncopal episode. She began having dizziness and palpitations approximately 2 days ago. She admits to episodes of intermittent palpitations and dizziness over the last 3 months. EKG reveals atrial fibrillation. She is started on warfarin. Cardiology is consulted and recommends electrical cardioversion. Question When is the most appropriate time for electrical cardioversion to be performed?

Correct answer: 4 weeks Explanation 4 weeks is correct. Patients with atrial fibrillation are at risk for thromboembolic events. When the heart is in a state of fibrillation, blood is pumped incompletely and may pool in the heart. This collected blood may form a clot. Since the patient presented in atrial fibrillation, it is uncertain when her condition began and her risk of clotting is greater. If cardioversion is performed before the patient has received adequate anticoagulation, the patient is at risk for stroke. It is recommended that patients be on anticoagulation for at least 3 weeks prior to cardioversion to decrease the risk of thromboembolic event. Immediately and 2 weeks are both incorrect. The patient should have at least 3 weeks of anticoagulation before undergoing cardioversion to decrease the risk of stroke.

A 65-year-old man presents to the office with complaints of 6 months of bilateral buttock and thigh cramping pain that occurs after walking 20 feet and is completely and quickly relieved with resting. His past medical history included hypertension treated with atenolol, and he had a stroke 3 years ago. He also complains of impotence for approximately the same duration of time. What is the patient's physical exam likely to include?

Correct answer: Absent femoral, popliteal, and pedal pulses Explanation The correct response is absent femoral, popliteal, and pedal pulses. Chronic arterial insufficiency is characterized by intermittent claudication, which is a very specific symptom defined as crampy pain consistently reproduced by the same level of exercise and is completely and quickly relieved by rest. The other symptoms of peripheral vascular disease include ischemic ulceration, blackening or gangrene formation, pain at rest, which occurs with worsening of the disease and coldness of the limb. This patient has thigh and buttock claudication with associated impotence, which is consistent with aortoiliac occlusive disease. The location of the pain is dependent on the location of vascular compromise. Thigh and buttock claudication associated with impotence or erectile dysfunction is called Leriche syndrome. Aortoiliac occlusive is characterized by absent femoral, popliteal, and pedal pulses. The classic symptoms, originally described by Leriche, include a triad of claudication, impotence, and absent femoral pulses. This disease often remains stable for years and can actually improve with time as collaterals enlarge. Aortoiliac occlusive disease can be diagnosed via Doppler and confirmed with CT angiography, MRA, or angiography.

A 61-year-old man presents with chest pain for the last hour. An ECG is obtained and found to have T wave inversion and wide Q waves in leads II, III, and AVF. Question These ECG findings are most consistent with what condition?

Correct answer: Acute inferior myocardial infarction Explanation The correct answer is acute inferior myocardial infarction, as ECG findings include T wave inversion and wide Q waves in the inferior leads II, III, and AVF. T wave inversion without significant Q waves would suggest ischemia without myocardial infarction (MI). T wave inversion and wide Q waves in leads V1 and AVL would suggest a lateral MI. T wave inversion and wide Q waves in leads V1, V2, V3, or V4 suggest an anterior MI. Large R waves and ST segment depression in leads V1, V2, and/or V3 suggest a posterior MI

Case Ico-delete Highlights A 32-year-old woman is brought in via ambulance due to a fast heart rate. Her blood pressure is 114/76 mmHg, and her heart rate is 156 bpm. She complains of mild chest pain and shortness of breath. Her 12-lead EKG reveals retrograde P-waves that occur simultaneously with the QRS complexes but appear "hidden". Question What do you order next for this patient?

Correct answer: Adenosine 6 mg IV Explanation The clinical picture is suggestive of a paroxysmal supraventricular tachycardia (PSVT). The mechanism of PSVT is most often reentry involving the AV node. When administered via IV, adenosine is recommended as the first-line agent due to its brief duration of action and minimal inotropic activity. Bretylium is used when all other attempts have failed for patients with life-threatening ventricular tachycardia of fibrillation. Verapamil was once the treatment of choice for reentrant PSVT, but adenosine has assumed that role now. Verapamil can be used only if adenosine fails to terminate the arrhythmia or if there is a contraindication to the use of adenosine. Ibutilide is mainly used for the conversion of atrial fibrillation or flutter. Lidocaine is commonly used in hospitalized patients to suppress ventricular arrhythmias.

A 60-year-old man presents to your outpatient clinic for work up after having a syncopal episode during a walk 2 days prior to presentation. He mentions generalized fatigue for several months as well as "just not feeling well". He describes a non-specific mild angina that has been coming and going for the last several months; however, he has not sought medical care. Exam reveals a harsh systolic ejection murmur that radiates to the neck and is louder when the patient leans forward. Lung auscultation is clear throughout. He has no peripheral edema or calf pain. You obtain an EKG, which reveals left ventricular hypertrophy; a chest X-ray shows cardiac enlargement with small bilateral pleural effusions. Question What is the most appropriate treatment plan for this patient?

Correct answer: Admit to the hospital in order to undergo further testing Explanation The patient should be admitted to the hospital so that he might undergo further testing. The patients' history, exam, and diagnostics are consistent with a diagnosis of aortic stenosis. Due to the fact that the patient is symptomatic (syncope and angina) and there is diagnostic evidence of heart failure (pleural effusions, cardiomegaly, left ventricular hypertrophy), there is significant concern that this patient may deteriorate quickly. The best course of action at this time is hospital admission for continued diagnostic work up and treatment.

A 56-year-old man presents with moderately severe substernal and left anterolateral chest pain. There is some exacerbation of pain on inspiration, and it has been increasing in severity over the last 36 hours. He works as a truck driver and has a history of heavy cigarette smoking, hypertension, and obesity. Over the past week, he has experienced swelling and discomfort in his right calf. Examination shows BP of 90/55 mm Hg, P of 122/min, RR of 40/min, and temp of 37.6° C. The patient is mildly agitated and confused. Systemic examination reveals tachycardia, soft systolic murmur, and questionable ventricular gallop. Lungs show dullness to percussion at left base, with scattered crackles and wheezes throughout. Abdominal and neurological exams are negative. The right calf is 0.5 cm larger than left, with some deep tenderness and a trace of ankle edema. Laboratory analysis reveals hemoglobin 16.4g/dL, Hct 51%, WBC 12,300 cells/µL, PaO2 52 mm Hg, PaCO2 38 mm Hg, and pH 7.35. Chest radiograph shows borderline cardiomegaly and a prominent aorta, scattered patchy infiltrates bilaterally, and a small left pleural effusion. Question What is the most accurate diagnostic modality for diagnosing this patient's condition?

Correct answer: Computerized tomographic angiography (CTA) Explanation The correct response is computerized tomographic angiography. The clinical picture is most compatible with a diagnosis of pulmonary embolism (PE). Pulmonary embolism is a condition in which there is sudden lodgment of a blood clot in a pulmonary artery with cause subsequent obstruction of blood supply to the lung parenchyma. The clinical manifestations of PE are usually nonspecific and vary in frequency and intensity depending on the extent of pulmonary vascular occlusion, pre-embolic cardiopulmonary function, and the development of pulmonary infarction. Small thromboemboli may be asymptomatic. The most common symptoms of PE include dyspnea, pleuritic chest pain, cough, and hemoptysis. Based on the acuity and severity of pulmonary arterial occlusion, PE can be categorized into 4 types as massive PE, acute pulmonary infarction, acute embolism without infarction, and multiple pulmonary emboli. CTA is the initial imaging technique of choice in stable patients. The ventilation perfusion scanning (V/Q)/lung scan is also an important diagnostic modality for the diagnosis of PE with a reasonable possibility of either confirming the diagnosis (with a "high-probability" scan) or ruling it out (with a normal scan). However, CTA has largely replaced it as the initial modality of imaging in stable patients, and V/Q scanning is usually done if CT is not available or the person has a contraindication to CT or the intravenous dye. Impedance plethysmography could be helpful in explaining the leg tenderness and identifying the source of emboli, but it would not be definitive in confirming pulmonary embolism. The other tests address other possible diagnoses that are either of secondary importance (spirometry) or less likely (myocardial scan and cardiac ultrasound).

A 61-year-old man presents with a recent history of increased fatigue with mildly increased exertional dyspnea. The patient denies any significant past medical history but states that he had some heart problems as a child, though he was never clear as to what was the problem. On cardiac examination, you hear an early diastolic, soft decrescendo murmur with a high pitch quality, especially when the patient is sitting and leaning forward. No thrill is felt. Question Based on this patient's presentation, what do you expect as the patient's diagnosis?

Correct answer: Aortic regurgitation Explanation The correct answer is aortic regurgitation, as it presents as a soft, early diastolic, high-pitched murmur heard best when sitting and leaning forward. It is often a result of rheumatic heart disease, which may be inferred by the patient's history. Tricuspid stenosis presents as a diastolic rumbling murmur typically accompanied by a thrill heard louder on inspiration. Mitral stenosis is similar in presentation to tricuspid stenosis but is not affected by inspiration. Mitral valve prolapse and pulmonic stenosis are examples of systolic murmurs, in contrast to diastolic murmurs heard in this patient.

A 42-year-old woman with a history of dizziness and difficulty to exercise presents seeking help. She complains of fatigue, shortness of breath, and weakness for the past month. Upon physical exam, an EKG is conducted and an irregularly irregular heart beat at a rate of 90 beats a minute. Laboratory findings showed no abnormal findings. Upon ECHO, there do not appear to be any abnormalities. Question What is the most likely explanation of the findings?

Correct answer: Atrial fibrillation Explanation The clinical picture is suggestive of atrial fibrillation. This patient presents with symptoms that are commonly found in patients with atrial fibrillation; her EKG shows an irregularly irregular heartbeat is present, which is indicative of Afib. Congestive heart failure is not the correct answer because even though the patient may present with weakness and fatigue, the patient does not present with swollen legs or weight gain, all of which the patient does not express in the stem of the problem. Embolic stroke is not the correct answer because even though the patient may present with weakness and fatigue, the patient would also present with difficulty speaking, paralysis, or difficulty walking. Mitral valve regurgitation is not the correct answer because a patient with mitral valve regurgitation would not present with an irregularly irregular heartbeat. Mitral valve regurgitation can present with shortness of breath, fatigue, lightheadedness, and heart flutter. Supraventricular tachycardia is not the correct answer because the patient's EKG would present with a heat rate of about 180, which is not shown or described in the stem of the problem.

A 25-year-old Asian-Indian woman presents with pain, weakness, and increased numbness in her right hand for the last 24 hours. The patient also states that, for the last month, she has not felt very well; she has experienced malaise, night sweats, decreased appetite, and arthralgias. She notes that for the last week when she washed her hands with cold water, her right hand would turn blue. On clinical examination, the patient has fever, tachycardia, decreased pulse, and decreased blood pressure in the right arm; Babinski's sign is negative. Auscultation on the right supraclavicular space reveals a bruit. Laboratory findings are an elevated ESR and mild anemia. Question What is the most appropriate next step in the management?

Correct answer: CT angiography Explanation Complete CT angiography is the investigation of choice for a patient with suspicion of Takayasu's arteritis, an inflammatory and stenotic disease of medium- and large-sized arteries most likely in young Asian women between 15 and 30 years of age. Because of the predilection for aortic arch and its branches involvement, Takayasu's arteritis is often referred as the aortic arch syndrome or pulseless disease. The clinical course may be fulminant, hence the need for arteriography that shows the characteristic lesions with stenosis and post-stenotic dilatations, aneurysms, and increased collateral circulation. The most frequently affected site is the subclavian artery, manifested as claudication and Raynaud's phenomena. Steroids are used for treatment. Involvement of the aorta may lead to aortic insufficiency that would indicate the need for echocardiography, but aortic arteriography is a better test to make the diagnosis

A 55-year-old man presents with a 2-day history of confusion, increased respiratory rate, rapid pulse, notable malaise, thirst, and less-than-normal urination. In the morning, his wife gave him a multivitamin pill hoping that it would help; she tells you that he has long-standing hypertension and 1 week ago injured his neck in a car accident. On examination, you find BP 92/41 mm Hg, HR 150, RR 35, rapid and weak pulse, cutis marmorata, cold edematous extremities, and bluish discoloration of the tongue and nails. Question What kind of shock does this patient probably have?

Correct answer: Cardiogenic Explanation This patient most probably has cardiogenic shock. Shock is characterized by a significant reduction of systemic tissue perfusion, resulting in decreased oxygen delivery to the tissues. The effects of oxygen deprivation are initially reversible, but rapidly become irreversible when cell death, end-organ damage, multi-system organ failure occur; death can result. Therefore, prompt recognition and treatment are necessary. Cardiogenic shock is caused by inadequate circulation due to primary ventricular failure. Altered mental state is due to decreased cerebral perfusion and subsequent hypoxia. Peripheral pitting edema is caused by the rise in hydrostatic pressure that occurs in cardiac failure. Hypotension is due to the decrease in cardiac output. Rapid, weak, thready pulse is due to decreased circulation; tachycardia, cool, clammy, mottled skin (cutis marmorata), and cyanosis are due to vasoconstriction; and subsequent hypoperfusion of the skin, rapid and deep respirations are due to sympathetic nervous system stimulation and acidosis; fatigue is due to hyperventilation and hypoxia; and oliguria is due to insufficient renal perfusion. Neurogenic shock is caused by vasodilation, with pooling of the blood to a degree that adequate blood flow cannot be maintained, such as in a spinal cord injury. In a spinal cord injury, the loss of sympathetic cardiac nerve fibers at T1-T4 results in bradycardia and diaphoresis. Your patient has a cervical spine injury and no signs of vagal (parasympathetic) dominance. Hypovolemic shock is caused by a decreased amount of blood or fluids in the body. This decrease results from injuries that produce internal and external bleeding, fluid loss due to burns, and dehydration due to severe vomiting and diarrhea. Neck injury cannot be the cause of such a blood loss, as there is not enough space. Septic shock characterizes SIRS (systemic inflammatory response syndrome) and at least any 2 of the following: tachypnea, or PCO2 less than 32 mmHg signifying hyperventilation, white blood cell count either significantly low or elevated, tachycardia, fever, or hypothermia; there also must be the evidence of infection, signs of end-organ dysfunction, and refractory hypotension. Anaphylactic (allergic) shock occurs when an individual is exposed to a substance to which his or her body is sensitive. The individual may experience a burning sensation, loss of voice, hives, severe swelling, constricted airways, and difficulty breathing.

What is not a common cause of aortic stenosis?

Correct answer: Chronic intravenous drug abuse Explanation Intravenous drug abuse has not been described as a common cause of aortic stenosis. Common causes of aortic stenosis are: (1) Infants, children and adolescents (a) congenital aortic stenosis (b) congenital subvalvular aortic stenosis (c) congenital supraclavicular aortic stenosis (2) Young adults to middle age: (a) calcification and fibrosis of congenitally bicuspid aortic valve (b) rheumatic aortic stenosis (3) Middle aged to elderly (a) calcification of bicuspid valve (b) senile degenerative aortic stenosis (c) rheumatic aortic stenosis

A 67-year-old man with a prior history of endocarditis and a prosthetic mitral valve presents requesting antibiotic prophylaxis for a dental procedure that entails the extraction of one of his molars. He is able to speak and swallow normally and is presently asymptomatic. His physical exam is remarkable only for a "loudly-clicking S1" in the mitral position. He has a known allergy to penicillin, which results in urticaria and angioedema upon exposure. Question What is the best intervention concerning this patient?

Correct answer: Clarithromycin 500 mg PO 1 hour before the procedure Explanation Clarithromycin 500 mg PO 1 hour before the procedure is the correct response. The American Heart Association and the European Society of Cardiology currently recommend prophylactic antibiotics only for those patients at highest risk for severe morbidity or death from endocarditis. Cardiac conditions in which prophylactic antibiotics are recommended include high-risk cardiac lesions such as prosthetic heart valves, prior endocarditis, unrepaired cyanotic congenital heart disease including palliative shunts or conduits, completely repaired congenital heart defects during the 6 months after repair, incompletely repaired congenital heart disease with residual defects adjacent to prosthetic material, and valvulopathy developing after cardiac transplantation. This patient meets 2 of the identified high-risk cardiac lesions and should receive prophylaxis. His allergic history precludes the use of amoxicillin and cephalexin, while desensitization is unnecessary given available alternatives. The patient speaks and swallows without difficulty, making Clarithromycin the most suitable agent. Clindamycin would also be appropriate for this patient, but not via IV or IM.

A 6-week-old baby boy presents at your pediatric practice office by his mother. She reports that for the past week the baby has not been feeding well. Furthermore, he breaks out into a cold sweat on his forehead while feeding. Upon further questioning she reports that after extensive periods of crying he becomes extremely breathless, irritable, and extremely pale. The mother reports a normal vaginal delivery and denies any problems with her son at the time of discharge from the hospital following his birth. She reports a family history of congestive heart failure. Vitals are as follows: Pulse 130-regular, Respiration 34-regular, Blood pressure R arm 96/62 L arm 92/54 and R leg 70/42 L leg 74/40. Cardiac exam reveals 4+ carotid pulses bilaterally, 2+ brachial pulses bilaterally and absent femoral pulses bilaterally. A loud harsh systolic ejection murmur is noted at the base of the heart. The EKG reveals normal sinus rhythm with left ventricular hypertrophy. Question What is the most likely diagnosis?

Correct answer: Coarctation of the aorta Explanation The clinical picture is suggestive of coarctation of the aorta. It is the cause of about 6-8% of congenital heart disease cases. It is described as the narrowing of the aortic arch. An infant often has a normal exam at time of discharge after birth; however, poor feeding, dyspnea, irritability, pale skin, and poor weight gain often occur in the first 6 weeks of life. CHF by 3 months of age occurs in 20-30% of infants. The physical exam will reveal a higher BP in the upper extremities and absent or weak femoral pulses. There will be strong arterial pulsations in the neck and suprasternal notch and a harsh late systolic ejection murmur which is best heard at the base. An EKG may reveal left ventricular hypertrophy. A CXR may reveal rib notching and dilated L subclavian artery, however an echocardiogram is diagnostic. Atrial septal defect (ASD) is the cause of about 10% of congenital heart disease cases. With a small ASD the patient is often asymptomatic; however, with a large to moderate defect the associated symptoms often include exertional dyspnea, palpitations, cardiac failure, R ventricular heave, and cyanosis in infants. The physical exam usually reveals a murmur often described as a loud systolic ejection murmur near the 2nd and 3rd ICS and a midsystolic rumble at the lower R sternal border with a wide S2 split. Although an EKG may reveal right ventricular hypertrophy +/-, a right BBB and the CXR may reveal a large pulmonary artery an echocardiogram is diagnostic. Pulmonary stenosis is the cause of about 8-12% of congenital heart disease cases and can present as a neonatal emergency. Mild cases are asymptomatic. Moderate to severe disease may present with exertional dyspnea, syncope, chest pain, RV failure, and/or a parasternal heave. The cardiac exam often reveals a murmur that is described as harsh and loud, secondary to stenosis, along with a systolic ejection click with a prominent thrill at the 2nd and 3rd ICS. An echocardiogram is diagnostic. Tetralogy of Fallot is more common in children with Down Syndrome, fetal alcohol syndrome children, as well as children born to women who do not receive pre-natal care. An infant often has slowed growth and development, cyanosis at birth, and clubbing. An older child or adult may experience dyspnea with exertion that improves with squatting. In addition at approximately 2-4 months of age "hypoxic spells" or "tet spells" are noted. They are described as prolonged crying spells with increasing cyanosis, with crying there is a decrease in the intensity of the heart murmur. These spells usually occur after crying, feeding, or defecation. The heart murmur is usually audible at birth and described as a harsh systolic ejection murmur at the L upper sternal border. The EKG may reveal right ventricular hypertrophy or right atrial dilatation. The CXR reveals a small heart size or "boot" shaped cardiac shadow. Ventricular Septal Defect (VSD) is the cause of about 20-25% of congenital heart disease cases. Most often a newborn has no murmur immediately after birth, and most develop symptoms in the first weeks to months of life. The associated symptoms often include tachypnea as the 1st sign followed by poor feeding, diaphoresis, and eventually failure to thrive. In addition SOB, dyspnea, chest pain, and cyanosis can occur with large defects. Upon physical exam a murmur is often described as a loud harsh holosystolic murmur in the 3rd and 4th ICS along sternum, +/- middiastolic flow murmur, and a systolic thrill on the lower left sternal border may be discerned as well. Although an EKG may reveal right or left ventricular hypertrophy and a chest X-ray may reveal a left atrial or pulmonary artery enlargement, an echocardiogram is diagnostic.

A 51-year-old man with a history of Marfan syndrome presents to the emergency department with tearing chest pain that radiates to the back and neck. On examination, the patient is hypertensive, and a high-pitched decrescendo diastolic murmur at the left sternal border and diminished peripheral pulses are noted. Question Based on the patient's most likely diagnosis, what is the immediate diagnostic imaging modality of choice?

Correct answer: Computed tomography scan of the chest and abdomen (CT scan) Explanation The patient in the above scenario most likely has an acute aortic dissection. Aortic dissections occur when a tear develops in the intimal layer of the vessel, allowing blood to collect in the medial layer. These tears can be the result of the repetitive torque applied to the ascending and proximal descending aorta during the cardiac cycle, especially in patients with hypertension. Abnormalities of smooth muscle, elastic tissue, or collagen, as seen in Marfan syndrome, can also predispose vessels to dissection. There are 2 types of aortic dissection. Type A dissection involves the arch proximal to the left subclavian artery. Type B dissection typically occurs in the proximal descending thoracic aorta, just distal to the left subclavian artery. A CT scan is the immediate diagnostic imaging modality of choice for patients with suspected aortic dissection. The CT should include both the chest and the abdomen so that the extent of the dissection can be fully delineated.

A 73-year-old man presents with worsening shortness of breath with activity; it has been occurring over the last few months. Another issue the patient mentions is not being able to complete as many physical activities during the day as he could manage 3 months ago. Physical examination reveals hypotension, tachycardia, extremities that are cool to the touch, expiratory wheezing, and rhonchi auscultated during the pulmonary exam; there is also a diminished first heart sound with an S3 gallop heard during the cardiac exam. Question You order an ECG, but even before receiving the results, you are highly suspicious that the patient has what diagnosis?

Correct answer: Congestive heart failure Explanation This patient above is displaying multiple key signs and symptoms suggestive of heart failure. Heart failure may consist of primarily left ventricular (left side) or only right ventricular (right side) heart failure. Very often, patients will exhibit signs and symptoms of both right- and left-sided heart failure. Common signs and symptoms of heart failure include those of low cardiac output and congestion as well as fluid overload; they include dyspnea, exertional dyspnea, orthopnea, chronic nonproductive cough, edema, hepatic congestion, loss of appetite, or even nausea. Tachycardia, hypotension, reduced pulse pressure, cold extremities, diaphoresis, crackles in the lungs, expiratory wheezing, and rhonchi can be seen. Cardinal cardiac signs include a parasternal left, enlarged/sustained left ventricular impulse, and even an S3 gallop.

A 58-year-old African American man with type II diabetes mellitus and no known kidney disease presents due to an elevated blood pressure reading; it was taken 4 weeks ago. The previous reading was 138/88 mm Hg. He was given antihypertensive lifestyle recommendations, and he was advised to follow up at today's appointment. Today, he denies any symptoms; however, his blood pressure is currently 134/86 mm Hg. Question What is the most appropriate intervention at this time?

Correct answer: Continue to reinforce lifestyle recommendations Explanation As per the H&P findings in this case, this patient meets the JNC 8 goals for diabetic patients of <140/90 and is not classified as hypertensive. JNC 8 has simplified treatment and removed the previous classification of "prehypertension." You should therefore continue to reinforce lifestyle recommendations and monitor the patient. If this patient were to become hypertensive, the JNC 8 treatment algorithm recommends, for an adult patient with this profile (diabetes, no chronic kidney disease, of black ethnicity) and after continued lifestyle interventions, to initiate a thiazide-type diuretic (such as hydrochlorothiazide) or a calcium channel blocker (CCB) (such as amlodipine), either alone or in combination.

A 25-year-old African-American man presents with blood pressure issues. He states that he attended a work-sponsored health fair; he had his blood pressure taken, and the health fair representative strongly urged him to make an appointment to be seen by his primary care provider. The fair occurred 3 months ago. He has randomly checked his blood pressure at different times since the health fair in local grocery stores, and although he does not remember the numbers, he knows that "they were above normal". The patient also believes that both his mother and father are taking blood pressure medication, but he is not 100% sure. Vitals at this time reveal a BMI of 30 kg/m2 and a waist circumference of 41 inches; blood pressure in the left arm is 175/95 mm Hg, and in the right arm it is 172/99 mm Hg. Question Based upon the diagnosis at this time, what would most likely be seen during the fundoscopic examination of this patient?

Correct answer: Cotton-wool patches Explanation The correct response is cotton-wool patches. Fundoscopic examination of any patients diagnosed with essential hypertension is a priceless component; it can reveal how severe the high blood pressure has been anatomically and physiologically on the body up to that point in time. Cotton-wool patches usually appear as gray or white ovoid lesions that have soft borders. These occur secondary to infarction of the nerve fibers within the fundus area; hypertension is a common etiology that can lead to the presence of this pathology. Drusen bodies are fundoscopic findings that are yellowish, round spots; they vary in size, but they are generally small. Edges of these areas may appear soft or hard. Although Drusen bodies may be seen in normal aging patients, it is very commonly related to the diagnosis of age-related macular degeneration. Microaneurysms are considered a hallmark finding of diabetic retinopathy. Deep retinal hemorrhages are also commonly seen in patients with diabetes. A macular star results from punctuate exudates that radiate from the fovea in a star pattern; this is seen is patients who are experiencing malignant hypertension.

A 57-year-old woman presents with progressive dyspnea and lower extremity edema. She has noticed the symptoms occurring over the last 3 months and also notes increasing fatigue over the last year. Past medical history is significant for diabetes and hypertension. Past social history is significant for previous alcohol abuse; patient has been sober for approximately 5 years. On physical examination, an S3 gallop is heard. Electrocardiogram reveals ST-T changes. Echocardiogram reveals left ventricular dilatation with an ejection fraction of 35%. Question What is the most likely diagnosis in this patient?

Correct answer: Dilated cardiomyopathy Explanation Dilated cardiomyopathy is correct. Dilated cardiomyopathy occurs when the left ventricle becomes dilated and weakened. As a result, the heart is unable to efficiently pump blood, leading to symptoms of fluid overload. The majority of cases are idiopathic, but alcohol abuse is a cause. An S3 gallop may be heard on cardiac auscultation. Echocardiogram will reveal a dilated left ventricle with decreased ejection fraction. Hypertrophic cardiomyopathy is incorrect. Hypertrophic cardiomyopathy occurs when the myocardium is abnormally thickened, causing impaired contractile function. It is associated with a hereditary syndrome. It typically presents with dyspnea, chest pain, and syncope. Echocardiogram will reveal left ventricular hypertrophy. Restrictive cardiomyopathy is incorrect. Restrictive cardiomyopathy is characterized by impaired diastolic filling with maintenance of the heart's ability to contract. Echocardiogram will reveal a small or normal sized left ventricle with decreased function. Inflammatory pericarditis is incorrect. Inflammatory pericarditis is caused by inflammation of the pericardium. It may be caused by infection or systemic disease. Patients typically present with pleuritic chest pain. Electrocardiogram commonly reveals diffuse ST elevation. Echocardiogram may be normal but often reveals a pericardial effusion. Constrictive pericarditis is incorrect. Constrictive pericarditis occurs when the pericardium becomes thickened and dilated. Echocardiogram may occasionally show a thickened pericardium. It is difficult to distinguish from restrictive cardiomyopathy and may require cardiac catheterization for diagnosis.

A 73-year-old Caucasian man presents with worsening shortness of breath with activity over the last few months. Another issue the patient mentions is not being able to complete as many physical activities during the day as he could do 3 months ago. Physical examination reveals mild hypotension, tachycardia, extremities that are cool to the touch, expiratory wheezing and rhonchi auscultated during the pulmonary exam, and a diminished first heart sound with an S3 gallop heard during the cardiac exam. You order an ECG, but even before receiving the results, you are highly suspicious of the most likely diagnosis. Question What medication should you begin in this patient on to help alleviate his symptoms?

Correct answer: Diuretic Explanation The correct response is diuretic. The patient above is displaying multiple key signs and symptoms suggestive of heart failure. Heart failure may consist of primarily left ventricular (left side) or only right ventricular (right side) heart failure. Very often patients will exhibit signs and symptoms of both right and left sided heart failure. Common signs and symptoms of heart failure include those of low cardiac output and congestion as well as fluid overload. This could include dyspnea, exertional dyspnea, orthopnea, chronic nonproductive cough, edema, hepatic congestion, loss of appetite, and even nausea. Tachycardia, hypotension, reduced pulse pressure, cold extremities, diaphoresis, crackles in the lungs, expiratory wheezing, and rhonchi may also be present. Cardinal cardiac signs include a parasternal left, enlarged/sustained left ventricular impulse, and even an S3 gallop. More often than not chronic medical management of patients with heart failure involves a combination of various medications. Diuretics are seen as the most effective way to provide symptomatic relief in patients with moderate to severe heart failure that's resulting in dyspnea and fluid overload. Often, inhibitors of renin-angiotensin-aldosterone system (ACE inhibitors, ARBs) are used in conjunction with diuretics, as well as potentially a beta-blocker that may be included in this multimodal approach too. Statin medications specifically have not been shown in at least 2 distinct trials—the CORONA and the GISSI-HF trials—to be beneficial in patients with heart failure. It can be argued that vascular disease is present in many of these patients and this is the reason to initiate this therapy, it will not directly help the signs and symptoms of heart failure. An antiarrhythmic medication would only be considered in a patient experiencing heart failure if there are any treatable dysrhythmias present. The patient does not exhibit any symptoms or signs distinctly directing one to believe there is an arrhythmia present. There is no substantial benefit to having patients begin a direct-acting vasodilator and therefore this would be an incorrect choice. It should be pointed out that ACE inhibitors also have some vasodilating properties and have overall found to be more beneficial for heart failure patients. Anticoagulant therapy may be considered in patients who experience heart failure and have evidence of atrial fibrillation, have a history of having thromboemboli, or who have had a recent anterior myocardial infarction. Our patient does not fall into any of these categories according to the information above.

A 12-year-old presents with an injury of his left arm and leg. He states that he felt dizziness during the 2nd mile of the long distance run organized by the school. He fell and lost the consciousness for several seconds, but after that he felt "normal". His father has been diagnosed with Emery-Dreifuss muscular dystrophy type 1. On examination, you find a few superficial excoriations; there is also symmetric humero-peroneal weakness involving the biceps, triceps, and peroneal muscles. There is also atrophy and contractures of Achilles-heel, elbows, and posterior neck. Question After taking care of his injuries, what test should you order?

Correct answer: EKG Explanation The correct response is EKG. Cardiac disease can take form of the bradycardia, atrial arrhythmias (including atrial fibrillation/flutter), AV conduction defect, or even atrial or ventricular cardiomyopathy. Pacemakers are often needed by the age of 30. Minimal follow up requirements in patients are annual cardiac assessment (ECG, Holter, echocardiography) and the monitoring of respiratory function. In most cases of Emery-Dreifuss muscular dystrophy, CK levels are mildly elevated; however, CK values will not be useful in this case because both mild muscle injury and muscle exertion affect CK values. The use of the test for lactate dehydrogenase (LDH) as a general indicator of the presence and the severity of the tissue damage (both acute and chronic) will not contribute to the management of Emery-Dreifuss muscular dystrophy.

A 32-year-old man with no significant past medical history presented with dyspnea, palpitations, feelings of anxiety, and dizziness, all of which occurred earlier in the morning following a brisk walk. He denied any prior episodes, illicit drug use, alcohol or cigarette use, skipping meals, or caffeine intake. He further denied fever, chills, chest pain, history of murmurs, cough, edema, rashes, syncope, headache, psychiatric, or focal neurological complaints. The physical examination demonstrated a fast, regular pulse with a constant-intensity first heart sound, but was otherwise normal. An EKG was performed, which revealed a short PR interval plus a slurred upstroke at the beginning of the QRS complex. Question What is the most likely mechanism responsible for this patient's presentation?

Correct answer: Early excitation due to accessory pathways between the atria and ventricles Explanation Patients with Wolff-Parkinson-White syndrome have an additional aberrant muscular or nodal tissue connection (bundle of Kent) between the atria and ventricles. This conducts more rapidly than the slowly conducting AV node, and one ventricle is excited early. The manifestations of its activation merge with the normal QRS pattern, producing a short PR interval and a prolonged QRS deflection slurred on the upstroke, with a normal interval between the start of the P wave and the end of the QRS complex ("PJ interval"). The QRS complexes show an abnormal morphology with a width greater than the baseline QRS complex (often >0.11 second), with the characteristic initial slurring referred to as a delta wave. The paroxysmal atrial tachycardias seen in this syndrome often follow an atrial premature beat. This beat conducts normally down the AV node but spreads to the ventricular end of the aberrant bundle, and the impulse is transmitted retrograde to the atrium. A circus movement is thus established. Less commonly, an atrial premature beat finds the AV node refractory but reaches the ventricles via the bundle of Kent, setting up a circus movement in which the impulse passes from the ventricles to the atria via the AV node. Conduction delay in the proximal part of the right or left branches will cause a bundle branch block, not tachycardia. Rapidly firing ectopic foci in pulmonary veins have been shown to be the underlying mechanism of most paroxysmal atrial fibrillation. Pre-excitation occurring via an atrio-His bundle bypasses the AV node slow conduction area and, therefore, does not modify the QRS complex morphology (unlike the EKG findings in this patient). Inappropriate (nonparoxysmal) sinus tachycardia is an incessant and symptomatic tachycardia caused by inappropriately enhanced automaticity of sinus node pacemaker cells. Characteristics include an atrial rate 100 beats/min at rest or with minimal activity, an inappropriate chronotropic response, with a rapid rise in heart rate with mild exertion, a normal P wave axis and morphology during tachycardia, and absence of an underlying cause.

A 35-year-old woman presents with fatigue and yellowish coloration of her eyes and skin that started several weeks after noneventful implantation of the prosthetic mechanical heart valve. Physical examination reveals the presence of regurgitant murmur and subicterus. Laboratory results are: hemoglobin 7.0 g/dL; reticulocytes 21%; white blood cells 11,500/µL; platelets 80,000/µL; and undetected levels of haptoglobin. In lactate dehydrogenase, the direct and indirect bilirubin levels are all elevated (3,100 U/L, 2.1 and 1.2 mg/dL, respectively). Peripheral blood smear shows burr and helmet cells (schistocytes) and polychromasia. Both direct and indirect Coombs' tests are negative. You suspect microangiopathic hemolytic anemia. Question What is the next step in management?

Correct answer: Echocardiography Explanation Anemia, reticulocytosis, and jaundice are the characteristics of hemolytic anemias. Red blood cells (RBC) survival is shortened; bone marrow increases erythroid production that results in the increased number of reticulocytes; and RBC breakdown manifests as increased unconjugated bilirubin and lactate dehydrogenase and decreased (undetectable) haptoglobin. Microangiopathic hemolytic anemia occurs when the red cell membrane is damaged in circulation, leading to intravascular hemolysis and the appearance of schistocytes (fragmented erythrocytes). Hemolytic anemia due to mechanical damage is seen with prosthetic mechanical heart valves. Your first and most important next step in the management of this patient will be to find out if there is a dysfunction of the prosthetic valve. Echocardiography will reveal the presence of significant leaking or valve dysfunction. Hemolytic anemias are generally caused by intrinsic (defects in erythrocytes membranes, enzyme systems, and hemoglobin; mostly hereditary) and extrinsic factors (immune and microangiopathic hemolytic anemias). Hemoglobin electrophoresis will be useful in the diagnosis of thalassemia or sickle cell anemia that is caused by intrinsic hemoglobin defect. Anemias in those disorders are usually hypochromic and microcytic.

A 25-year-old man presents after experiencing pronounced shortness of breath while at the gym. You cannot obtain satisfying information from him during the interview and you think that he may have an intellectual disability disorder. On examination he appears short for his age and you notice webbed neck, dental malocclusion, antimongoloid slanting of the eyes, and hypogonadism. Auscultation reveals high-pitched systolic ejection murmur maximal in the second left interspace with radiation to the left shoulder and ejection click that decreases with inspiration. His second heart sound (P2) is delayed and soft. The impulse of right ventricle is increased, and you palpate a thrill at second left intercostal space. Question What will be your best next diagnostic step?

Correct answer: Echocardiography/Doppler Explanation Although the patient's clinical features resemble Turner syndrome, a chromosomal condition that affects development in females, the intellectual disability and distinctive physical features of this male patient suggest Noonan Syndrome instead. Cardinal features of Noonan Syndrome include unusual facies (i.e., hypertelorism, down-slanting eyes, webbed neck), short stature, bleeding diathesis, chest deformity, intellectual disability, skeletal, neurologic, genitourinary, lymphatic, eye, and skin findings, all of which may be present to varying degrees. Most notably for the patient 50% of Noonan Syndrome patients have congenital heart disease and is often associated with pulmonary stenosis. It is one of the most common genetic syndromes associated with congenital heart disease, similar in frequency to Down syndrome. Assessment of a nonspecific sign like intellectual developmental will probably not be diagnostic of Noonan Syndrome or pulmonic stenosis. Addressing the serious cardiac dysfunction before the intellectual disability or chromosomal condition is of primary concern to selecting the best next diagnostic step. The patient likely has pulmonic stenosis as noted by his dyspnea on exertion and loud ejection click heard to precede the high-pitched systolic ejection murmur. These murmurs characteristically decrease with inspiration (the only right heart auscultatory event that decreases with inspiration, as all others increase). This is due to increased RV filling from inspiration prematurely opening the valve during atrial systole. Delayed and soft P2 and a thrill at a second left intercostal space are also signs of pulmonic stenosis.

A 70-year-old woman with a history of hypertension, hyperlipidemia, and myocardial infarction presents with a 3-day history of shortness of breath at rest. She has found it difficult to walk short distances due to this shortness of breath. Additionally, she complains of orthopnea and nocturnal dyspnea. She denies cough, fever, chills, nausea, abdominal pain, vomiting, diarrhea, rashes, and edema. Upon physical examination, the patient is short of breath and requires numerous pauses during the conversation. She is tachycardic and diaphoretic, and her extremities are cool. There is a diminished first heart sound, S3 gallop, laterally displaced PMI, bibasilar rales and dullness to percussion, and expiratory wheezing noted. There is no jugular vein distention (JVD) noted; however, 2+ pitting edema of the lower extremities to the level of the mid-calf is evident. Question What diagnostic test result would be considered most useful in differentiating a cardiac from noncardiac cause of this patient's presentation?

Correct answer: Elevations of B-type natriuretic peptide Explanation This patient's presentation is consistent with congestive heart failure. Rapid measurement of B-type natriuretic peptide (BNP) or its precursor, N-terminal proBNP (NT-proBNP), can aid clinicians in differentiating between cardiac and noncardiac causes of dyspnea. The major source of plasma BNP is the cardiac ventricles, and the release of BNP appears to be in direct proportion to ventricular volume and pressure overload. BNP levels greater than 80 pg/mL have a specificity greater than 95% and a sensitivity greater than 98% in the diagnosis of heart failure; higher BNP levels correlate with higher sensitivity and specificity of the presence of heart failure. Electrolyte levels, thyroid assessments, EKG, and chest x-ray findings are important diagnostic modalities useful in the evaluation of CHF, but they cannot readily differentiate cardiac versus noncardiac causes of this patient's presentation to the extent of B-type natriuretic peptide.

A patient is presenting with substernal crushing chest pain and shortness of breath for just 15 minutes prior to arriving to the emergency department. An EKG is obtained and appears suspicious for an acute myocardial infarction. What would most likely be seen in this patient's EKG?

Correct answer: Hyperacute T-wave Explanation The earliest presentation of an acute myocardial infarction is the hyperacute T wave. If found, this is treated the same as an ST segment elevation type of infarction (STEMI). The hyperacute T wave is considered very rare in clinical practice, as it only exists for 2-30 minutes after the onset of an infarction. Hyperacute T waves must also be distinguished from the characteristic peaked T waves, which commonly are seen with hyperkalemia. A Q-wave elongation may appear hours to as much as 24 hours after the onset of the infarction. ST-T segment elevation is more than likely due to an acute change, while depression is not as specific. Ventricular bigeminy may indicate heart irritability due to an MI, but it is more of a conduction problem. P- wave inversion would likely represent retrograde atrial depolarization or electrode misplacement, or extremely rarely dextrocardia. It is not likely due to myocardial infarction.

A 22-year-old woman presents with an 8-month history of amenorrhea. Further questioning elicits additional pertinent positives of backaches, headaches, hirsutism, and acne. Physical examination reveals a female patient with a moon-shaped facies, multiple purple striae, and significant central obesity (body mass index of 36). Question Considering the likely diagnosis, what other abnormality would be expected?

Correct answer: Hypertension Explanation This patient likely is suffering from Cushing syndrome, which is also known as hypercortisolism. Consequences of excessive levels of circulating cortisol, no matter the etiology, will lead to signs and symptoms such as central obesity but thin extremities, a moon face, a buffalo hump, supraclavicular fat pads, protuberant abdomen, hirsutism; there may also be oligomenorrhea, amenorrhea, or in men, possible erectile dysfunction. Backaches, headaches, acne, purple striae, and impaired wound healing may also be found in these patients. The correct response is (secondary) hypertension; although secondary hypertension only accounts for 5 - 10% of all hypertension cases, Cushing's syndrome has been found to be a potential cause for these cases. Its main cause is via the mineralocorticoid effects of excess glucocorticoids. Hypotension is an incorrect response. Hypokalemia secondary to aldosteronism is what would potentially lead to secondary hypertension. Exophthalmos is a physical exam sign that is evident in patients with hyperthyroidism. If carotid bruits are auscultated, this is usually a sign of carotid stenosis.

A 68-year-old man with a past medical history of diabetes mellitus type II, hypothyroidism, and hyperlipidemia presents with intermittent bouts of moderate to severe "squeezing, pressure, and tight" left-sided chest pain, which have been occurring over the last 3 months. Additionally, he admits to associated shortness of breath and nausea during these episodes, which are provoked upon exposure to the cold weather, during times of emotional stress, and following the consumption of a meal. Chest pain lasts approximately 10-15 minutes and resolves spontaneously. He denies diaphoresis, fever, chills, abdominal pain, reflux, regurgitation, diarrhea, cough, and pleurisy. Question What pharmacologic agent should be avoided in the management of this patient?

Correct answer: Immediate release nifedipine (short-acting) Explanation This patient's most likely diagnosis is angina pectoris. Short-acting agents, such as short-acting nifedipine, should be avoided due to increased cardiovascular and mortality risks in some patients, as well as significant adverse effects such as reflex tachycardia. Amlodipine, oral nicardipine, and long-acting nifedipine are effective treatment options for chronic stable angina.

A 42-year-old man with a past medical history of hypertension presents with a 6-week history of intermittent fever. He has an associated cough, dyspnea, anorexia, arthralgias, abdominal pain, diarrhea, a widespread rash throughout his body, and back pain. He has come to see you because he has experienced acute left upper and lower extremity weakness and painless hematuria since this morning. He denies chills, a history of travel, sick or confined contacts, exposure to animals, bites, stings, cigarette smoking, otalgia, sore throat, swollen glands, drug use, dysuria, preceding GI or GU infections, previous surgeries, or sexual contact in the past year. His physical exam is remarkable for fever, a generalized petechial rash and petechiae of the mucous membranes, dark red linear lesions of the nailbeds, tender subcutaneous nodules of the digital pads, and nontender maculae on the palms and soles. His heart is notable for a new harsh, medium pitched pansystolic murmur at the apex with radiation to axilla, reduced strengths to the left upper and lower extremities, and splenomegaly. Question What is the most likely diagnosis?

Correct answer: Infective endocarditis Explanation This patient is demonstrating signs and symptoms representative of infective endocarditis. The symptoms of early infective native valve endocarditis include low-grade fever, anorexia, weight loss, influenzalike syndromes, polymyalgia, pleuritic pain, and abdominal symptoms (e.g., right upper quadrant pain, vomiting, postprandial distress, appendicitis-like symptoms). When appropriate therapy is delayed for weeks or months, additional clinical features, embolic or immunological in origin, develop. These include acute meningitis with sterile spinal fluid, hemiplegia in the distribution of the middle cerebral artery, regional infarcts that cause painless hematuria, infarction of the kidney or spleen, unilateral blindness caused by occlusion of a retinal artery, and myocardial infarction arising from embolization of a coronary artery. A very common physical exam finding is a murmur; the development of a new regurgitant mumur is a major modified Duke Criterion (others include: 2 positive blood cultures and evidence of echocardial involvement documented by echocardiography). Other findings are petechiae of the palpebral conjunctivae, the dorsa of the hands and feet, the anterior chest and abdominal walls, the oral mucosa, and the soft palate, nontender maculae of the palms and soles (Janeway lesions), subungual hemorrhages (splinter hemorrhages), Osler nodes, digital clubbing, oligoarthritis similar to rheumatoid arthritis, Reiter syndrome, or Lyme disease, splenomegaly, and, rarely, Roth spots. A definite diagnosis of endocarditis can be made if 2 major criteria, 1 major and 3 minor, or 5 minor (presence of a predisposing condition, fever > 38 °C, vascular or immunologic phenomena) modified Duke Criteria are met. Reactive arthritis, also known as Reiter syndrome, is associated with HLA-B27 and is classified as a type of seronegative spondyloarthropathy. It develops 2-4 weeks after a genitourinary or gastrointestinal infection and is characterized by the triad of urethritis, conjunctivitis, and arthritis. Syphilis is an infectious venereal disease caused by the spirochete Treponema pallidum; it is transmissible by sexual contact with infectious lesions, from mother to fetus in utero, via blood product transfusion, and occasionally through breaks in the skin that come into contact with infectious lesions. Primary syphilis occurs within 3 weeks of contact with an infected individual whose characteristic lesion (chancre) is usually a solitary, raised, firm, red papule that can be several centimeters in diameter. Secondary syphilis usually presents with a cutaneous eruption within 2-10 weeks after the primary chancre and is most florid 3-4 months after infection. A localized or diffuse nonpruritic and bilaterally symmetrical mucocutaneous macular rash with lesions 5-10mm in diameter are distributed on the trunk and proximal extremities. Papular lesions 3-10mm in diameter appear afterwards, often become necrotic, and are distributed especially over the palms and soles. There is generalized nontender lymphadenopathy, patchy alopecia, and condylomata lata. Tertiary syphilis is characterized by cutaneous gumma formation, cardiovascular effects (aneurysms and valvular incompetence), and neurosyphilis (meningitis, Argyll Robertson pupil, sensory, personality, and cognitive changes, and tabes dorsalis). Systemic lupus erythematosus is a chronic inflammatory disease characterized by an autoantibody response to nuclear and cytoplasmic antigens. It is most common in black females; more than 90% of cases of SLE occur in women, frequently starting at childbearing age. It can affect almost any organ system, and the presentation and course are highly variable. However, the cutaneous manifestations of SLE include 3 American College of Rheumatology (ACR) lupus diagnostic criteria: malar rash, photosensitivity, and discoid lupus, which are not seen in this male patient. Raynaud pleuritis and sicca are also common in SLE, but not endocarditis. Lyme disease is due to systemic infection with the spirochete Borrelia burgdorferi and the body's immune response to the infection. It is caused by the bite of Ixodes ticks. Erythema migrans along with constitutional symptoms predominate stage I of Lyme disease.

After a serious argument with his wife about his diet and habits (he is overweight and a heavy smoker), your 55-year-old neighbor went to a pub and got drunk. On the way back to home, he felt sudden chest pain radiating to the left shoulder, shortness of breath, sweating, and anxiety. His ECG shows pathological Q wave and ST elevation. Laboratory results show that troponin levels are elevated. Question What caused the rise in troponin levels in this patient?

Correct answer: Injury to the myocardial cell membrane Explanation Profound disturbances in membrane function and the lack of the generation of ATP are signs of irreversible cell injury. Whatever the mechanism(s) of membrane damage, the end result is a massive leak of intracellular materials and a massive influx of calcium, which results in cell death. When the cellular membrane is irreversibly damaged, intracellular proteins leak across the degraded membrane into the peripheral circulation. Those proteins, when tissue specific, are called markers. Markers of cellular injury and death specific to cardiac muscle that can be detected in the blood are a specific isoform of the enzyme creatine kinase and of the contractile protein troponin. Fatty change represents the intracytoplasmic accumulation of triglyceride (neutral fats). Although fatty changes in myocardial cells may be present in obese patients, they are not the cause of elevated troponin. Cell adhesion molecules are proteins localized on the surface of the cell. They bind cells to other cells or to the extracellular matrix. They are not the cause of elevated troponin. Besides, their synthesis is decreased in the presence of elevated steroids after myocardial infarction. Troponin is a complex of regulatory proteins that are integral to the contraction of skeletal and cardiac muscles. It is a marker for several heart disorders, but it is not a cause of myocardial infarction. The presence of a coexisting viral infection is neither mentioned, nor supposed. Even if it exists, it cannot be the cause of elevated troponin. Ethanol can be the contributing factor for the development of the myocardial infarction; however, it does not elevate troponin.

A 68-year-old man with a past medical history of diabetes mellitus type II, hypothyroidism, and hyperlipidemia presents with a 1-hour history of constant moderate to severe "squeezing, pressure, tight" left-sided chest pain. Additionally, he admits to shortness of breath and nausea. He notes that he has had a 1-week history of similar, recurrent chest pain of about 10 minutes' duration that had been occurring following exposure to the cold weather and consumption of a meal. He denies chills, abdominal pain, diarrhea, cough, and pleurisy. Question What physical exam finding is an anticipated finding consistent with this patient's most likely diagnosis?

Correct answer: Jugular venous distension Explanation This patient's most likely diagnosis is acute myocardial infarction. Common general physical exam findings include anxiousness, diaphoresis, and a low-grade fever. Bradycardia may occur most commonly with inferior infarctions, while tachycardia may be due to increased sympathetic activity, low cardiac output, or arrhythmia. Blood pressures may be high in hypertensive patients or low in patients with shock. Jugular venous distension reflects right atrial hypertension, which may indicate right ventricular infarction or elevated left ventricular filling pressures. Atrial gallops (S3) are common, with ventricular gallops (S4) less common and reflecting significant left ventricular dysfunction. The skin is often pale, cool, and moist, with cyanosis and cold temperatures indicating low output. A grouped, vesicular rash over the precordium is inconsistent with acute myocardial infarction and most likely represents thoracic herpes zoster. Chest wall tenderness may be observed in chest wall pain syndromes, such as costochondritis. A deviated trachea may suggest a pneumothorax, pleural effusion, atelectasis, aneurysm, or lung mass. Splenomegaly is not a finding common to myocardial infarction.

Case Ico-delete Highlights A 48-year-old man is brought to the ED due to difficulty breathing, fatigue, and intermittent chest pain for the past month. On further questioning, he states that the breathing seems to worsen when lying down. On physical exam, you note elevated respiratory and heart rates ,and pale, sweaty skin. On auscultation, rales are noted as well as a 3rd heart sound. Question What is the most likely diagnosis?

Correct answer: Left ventricular failure Explanation The clinical picture is suggestive of left ventricular failure (LVF). Clinical presentation includes dyspnea, orthopnea, and paroxysmal nocturnal dyspnea. The patient may also have hemoptysis, chest pain, fatigue, nocturia, and confusion. On physical exam, the patient may present with cold, pale, sweaty skin; tachypnea and tachycardia; rales; and 3rd and 4th heart sounds. This is diagnostically different from right ventricular failure.

A 25-year-old woman presents with intermittent palpitations that are associated with lightheadedness; she admits to a past medical history of having a self-described "hole in her heart". These seem to occur upon significant exertion and when she is "stressed out." She denies chest pain, shortness of breath, wheezing, hemoptysis, cough, syncope, abdominal pain, rashes, peripheral edema, diaphoresis, and vomiting. Her physical exam is remarkable for a mid-to-late systolic click; it is followed by a high-pitched, 'whooping' late systolic crescendo-decrescendo murmur, and it is heard best at the apex. The click occurs earlier with standing and upon Valsalva strain, and it also occurs later in the cardiac cycle with squatting and sustained handgrip. Question What is the most likely diagnosis?

Correct answer: Mitral valve prolapse Explanation This patient's diagnosis is mitral valve prolapse (MVP). It most commonly occurs in young women and in patients with heritable disorders of connective tissue, including Marfan's syndrome, osteogenesis imperfecta, and Ehlers-Danlos syndrome. Rarely, it occurs as a sequel to acute rheumatic fever, in ischemic heart disease, in various cardiomyopathies, as well as in 20% of patients with ostium secundum atrial septal defect. MVP varies in its clinical expression, ranging from only a systolic click and murmur with mild prolapse of the posterior leaflet to severe mitral regurgitation due to chordal rupture and leaflet flail; in North America, MVP is now the most common cause of isolated severe MR that requires surgical treatment. The most important finding is the mid- or late- (non-ejection) systolic click following S1, and it is thought to be generated by the sudden tensing of slack, elongated chordae tendineae or by the prolapsing mitral leaflet when it reaches its maximum excursion. Systolic clicks may be multiple, and representing mitral valve regurgitation, they may be followed by a high-pitched, late systolic crescendo-decrescendo murmur, occasionally "whooping" or "honking" and heard best at the apex. The click and murmur occur earlier with standing, during the strain phase of the Valsalva maneuver, and with any intervention that decreases LV volume, exaggerating the propensity of mitral leaflet prolapse. Conversely, squatting and isometric exercises, which increase LV volume, diminish MVP; the click-murmur complex is delayed, moves away from S1, and may even disappear. Patients with MVP most frequently have symptoms of autonomic dysfunction, including easy fatigability, dizziness, and atypical chest pain. Further symptoms include palpitations, light-headedness, and syncope. The murmur of aortic stenosis is characteristically an ejection (mid) systolic murmur that commences shortly after the S1, increases in intensity to reach a peak toward the middle of ejection, and ends just before aortic valve closure. It is characteristically low-pitched, rough and rasping in character, and loudest at the base of the heart, most commonly in the 2nd right intercostal space. It is transmitted upward along the carotid arteries. Exertional dyspnea, angina pectoris, and syncope are the 3 cardinal symptoms; they become most apparent beginning in the 6th decade. Ventricular septal defect is the most common congenital cardiac malformation. Blood flows from the left ventricle to the right ventricle and presents as a harsh, blowing holosystolic murmur with a thrill localized to the 4th left intercostal space. This murmur may decrease with Valsalva and handgrip. Findings consistent with aortic regurgitation consist of tachycardia and widened pulse pressures. The classic auscultatory finding is a decrescendo, diastolic, high-pitched murmur loudest at the left sternal border that is accentuated with the patient leaning forward in full expiration. An Austin-Flint murmur may be evident, characterized by a low-pitched, mid-diastolic rumbling murmur due to blood jets from the murmur striking the anterior leaflet of the mitral valve, which results in premature closure of the mitral leaflets. Corrigan pulse ('water-hammer' pulse) is described as an abrupt distention and quick collapse on palpation of the peripheral arterial pulse; booming systolic and diastolic sounds auscultated over the femoral arteries characterize Traube sign ("pistol-shot" pulse). Rheumatic fever is the leading cause of mitral stenosis (MS). The first heart sound (S1) is usually accentuated and slightly delayed. The pulmonic component of the second heart sound (P2) also is often accentuated, and the 2 components of the second heart sound (S2) are closely split. The opening snap (OS) of the mitral valve is most readily audible in expiration at, or just medial to, the cardiac apex. It is followed by a low-pitched, rumbling, diastolic murmur; it is heard best at the apex with the patient in the left lateral recumbent position, and it is accentuated by mild exercise carried out just before auscultation.

A 56-year-old man presents with primary hypertension. His additional medical history includes only nephrolithiasis. Past analysis of his kidney stones has revealed a calcium oxalate composition. Question What class of antihypertensives might best be avoided due to his history of nephrolithiasis?

Correct answer: Loop diuretics Explanation Loop diuretics reduce blood pressure by decreasing fluid retention and normalizing plasma volume. Loop diuretics act on the ascending limp of the loop of Henle and block the Na+/K+/2Cl- cotransporter. They are highly potent because this transporter is responsible for the greatest proportion of renal salt and water absorption. Loop diuretics also increase urinary calcium excretion, which may enhance the formation of calcium oxalate kidney stones. The thiazide diuretics act in the kidney at the luminal side of the distal convoluted tubule to inhibit the Na+/Cl- cotransporter and therefore increase urinary salt and water excretion. This leads to decreases in extracellular fluid and plasma volumes. Thiazide diuretics also directly reduce peripheral arteriolar resistance, thereby improving both systolic and diastolic blood pressures. In addition to their antihypertensive effect, thiazide diuretics reduce urinary calcium excretion by indirectly increasing the activity of the 2Na+/Ca2+, also in the distal tubule. This occurs because relatively more sodium is delivered to this transporter due to the blockade of the Na+/Cl- cotransporter. Therefore, they can be helpful in controlling kidney stones that occur as a result of hypercalciuria.

A 49-year-old woman presents with a history of gradual onset of reduced exercise tolerance while working out at the gym. She is afebrile and otherwise feels well. Cardiac exam reveals a III/VI diastolic rumbling murmur located at the apex; it is heard best in the left lateral position. Question What is the most likely diagnosis?

Correct answer: Mitral stenosis Explanation The correct answer is mitral stenosis; it is a rumbling diastolic murmur heard best at the apex in the left lateral position. It may be a result of underlying rheumatic heart disease. Tricuspid stenosis presents as a diastolic rumbling murmur that is typically accompanied by a thrill heard along the left sternal border rather than at the apex; it is louder on inspiration. Mitral valve prolapse and pulmonic stenosis are examples of systolic murmurs; a diastolic murmur is heard in this patient. Aortic regurgitation presents as a soft early diastolic, high-pitched murmur heard best when sitting and leaning forward.

Case Ico-delete Highlights A 22-year-old woman presents due to palpitations. She denies chest pain and shortness of breath; she has not had any recent infections. Aside from a tonsillectomy as a child, she has no significant past medical history. Examination reveals a thin woman in no acute distress. Lungs are clear to auscultation bilaterally. Cardiac exam reveals a mid-systolic click. Question What is the most likely diagnosis?

Correct answer: Mitral valve prolapse Explanation The above patient is suffering from mitral valve prolapse. Mitral valve prolapse occurs when the mitral valve protrudes back into the left atrium. It is most frequently found in young, thin women. Patients may be asymptomatic, or they may experience dyspnea, fatigue, palpitations, and chest pain. Mitral valve prolapse will produce a mid-systolic click on auscultation. Mitral stenosis occurs when the mitral valve becomes narrowed, which is often due to rheumatic fever or congenital heart disease. Patients may also complain of similar symptoms, but they will have an opening snap and diastolic murmur on cardiac auscultation. Mitral regurgitation may be caused by mitral valve prolapse. However, this patient has only a mid-systolic click on cardiac examination, and if her mitral valve prolapse had progressed to mitral regurgitation, then a pansystolic murmur would be heard. Aortic stenosis develops due to either congenital narrowing of the aortic valve or calcification of the aortic valve. Patients will have a systolic ejection murmur on examination, which may radiate into the neck. Aortic regurgitation is associated with rheumatic heart disease, congenital abnormalities, infection, and hypertension. It is associated with a diastolic murmur.

A 30-year-old woman with no significant past medical history presents with a history of recurrent palpitations. These episodes occur primarily upon exertion. She recalls periodic bouts of anxiety, panic attacks, and lightheadedness. She denies fever, chills, changes in weight, chest pain, shortness of breath, rashes, diaphoresis, abdominal pain, nausea, and vomiting. She denies any history of cigarette smoking, drug, or alcohol use. Her physical exam revealed normal vital signs. The cardiac exam revealed a high-pitch late systolic click at the apex. The valsalva maneuver and a standing position result in prolongation of the murmur and a movement of the click to earlier in the cardiac cycle. The remainder of her examination is normal. Question What is the most likely diagnosis?

Correct answer: Mitral valve prolapse Explanation This patient's most likely diagnosis is mitral valve prolapse. The classic auscultatory finding is a mid-to-late systolic click, which is present due to the leaflets prolapsing into the left atrium, resulting in tensing of the mitral valve apparatus. It may or may not be followed by a high-pitched, mid-to-late systolic murmur at the cardiac apex. Activities that reduce end-diastolic volume, such as Valsalva maneuver or a standing position, cause an earlier click and a prolonged murmur. In the supine position, especially with the legs raised for increased venous return, left ventricular diastolic volume is increased, resulting in a click later in systole and a shortened murmur. Patent ductus arteriosus is characterized by an abnormal patency of the ductus arteriosus, which diverts blood from the right side of the heart to the systemic circulation during fetal life. Blood typically shunts from the higher-pressured left side (systemic circulation) to the lower-pressured right side (pulmonary circulation). This murmur is described as a hollow, machinery-like murmur that is continuous throughout the cardiac cycle. Mitral regurgitation is a medium to high-pitched apical, holosystolic, harsh murmur, which may radiate to the left axilla or less commonly to the left sternal border. It may be soft or loud and associated with a decreased S1. The murmur of mitral stenosis is a decrescendo low-pitched rumble usually limited to the apex with an accentuated opening S1 and opening snap that follows S2. The murmur associated with hypertrophic cardiomyopathy is a loud, harsh systolic murmur present along the left sternal border. The gradient and the murmur may be enhanced by maneuvers that decrease ventricular volume, such as an upright posture, standing, or Valsalva maneuver. It is decreased by increasing ventricular volume or vascular resistance, which occurs with squatting, sustained handgrip, lying down, or straight leg raises.

A 74-year-old man with a past medical history of hyperlipidemia, myocardial infarction, rheumatic heart disease, and hypertension presents for a routine evaluation. He states that he has no complaints and feels well. The physical exam reveals an elderly man in no apparent distress whose vital signs are within normal limits. The cardiac exam is noteworthy for a soft S1 to auscultation, a systolic thrill palpable at the cardiac apex, lateral displacement of the point of maximal impulse, and a holosystolic murmur of IV/VI intensity, which is loudest at the apex and radiates to the axilla. The intensity of the murmur does not increase with inspiration. The remainder of the exam is normal. Question What EKG finding would be most likely associated with this patient's presentation?

Correct answer: Negative P-wave deflection in V1 Explanation The correct answer is Negative terminal P-wave deflection in V1. This patient's murmur is identified as mitral regurgitation (MR). The murmur of MR is caused by turbulent flow through incompetent mitral valves, from the left ventricle (LV) into the left atrium, during ventricular systole. It is a high-pitched, "blowing" holosystolic murmur best heard at the apex and usually radiates to the axilla. The intensity of an MR murmur does not increase with inspiration, which helps distinguish it from the murmur tricuspid regurgitation. The S1 heart sound is a combination of the mitral and tricuspid valves closing. In MR, valve closure is incomplete and S1 may be noticeably quieter. A grade 4 murmur is loud and has a palpable thrill. The point of maximum impulse (PMI) is normally felt at the left 5th intercostal space, half inch medial to the left midclavicular line (precordium). Complications of MR include LV enlargement. Eventually, the heart decompensates and gradually the left atrium (LA) will enlarge. Lateral displacement of the PMI usually indicates cardiomegaly. In patients with MR in sinus rhythm, evidence of LA enlargement is characterized in lead V1 by a P-wave that is initially positive but is followed by a negative deflection at the wave's terminus. There may be a bimodal P wave in some leads. LA depolarization will last longer than normal so duration of the P-wave is longer than normal. P-wave amplitude and PR interval remain unchanged. A large P-wave deflection in lead II is incorrect as that typically accompanies a right atrial hypertrophy. Wide QRS complexes might be seen in left BBB, pacemaker or hyperkalemia, ST elevation is classically seen in ST-segment elevation myocardial infarction (STEMI). Pericarditis, hyperkalemia, pulmonary embolism, and Brugada syndrome may also be associated with ST elevation.

A 45-year-old woman presents with chest pain. Her EKG reveals ST elevations. She undergoes cardiac catheterization; all of her vessels are clean and no revascularization is performed. Question What is the best therapy for this patient?

Correct answer: Nifedipine Explanation This patient is exhibiting Prinzmetal's angina (vasoconstriction); she does not have any stenosis. The best therapy for her is a calcium channel blocker (e.g., nifedipine). Diazoxide is used in the treatment of acute hypertension. Beta blockers should not be prescribed in this patient as she does not have any stenosis. Ibuprofen would only be acceptable if this patient was experiencing costochondritis.

An 8-month-old boy presents for a routine evaluation. His mother states that the child is gaining weight and feeding appropriately and has been without fever, chills, dyspnea, or other abnormal objective signs. Upon physical examination, the examiner noticed a loud, harsh holosystolic murmur in the left third and fourth interspaces along the sternum that was associated with a systolic thrill. The vital signs were normal, with normal length and weight and a physiologic splitting of S2. There was no cyanosis, edema, or hepatomegaly. Question What is the most appropriate course of action for this patient?

Correct answer: Observation Explanation Observation is the correct response. This patient's presentation represents a ventricular septal defect (VSD), with the history and physical exam suggesting a small VSD as most likely. Children with small ventricular septal defects (VSDs) are asymptomatic and have an excellent long-term prognosis. Cardiac catheterization is used in circumstances of pulmonary hypertension of unknown reactivity or in assisting the decision making regarding the need for surgery by determining the pulmonary-to-systemic flow ratio in small-to-moderate defects with only mild left ventricular enlargement. Membranous and muscular septal defects may spontaneously close in childhood as the septum grows and hypertrophies; small shunts in asymptomatic patients do not require surgery or medical intervention. Uncontrolled CHF with growth failure and recurrent respiratory infection is an indication for surgical repair. In symptomatic patients, diuretics (furosemide) may be used to relieve pulmonary congestion. Angiotensin-converting enzyme (ACE) inhibitors reduce both the systemic and pulmonary pressures, thereby reducing the left-to-right shunt. Digoxin may be used if symptoms persist despite diuretic and ACE-inhibitor use. MRI is used only when ultrasonography is not feasible or when ultrasonographic findings are not diagnostic. It is also used as a substitute in cardiac catheterization for diagnostic purposes.

A 45-year-old man is admitted to the hospital with fever, weakness, weight loss, muscle and testicular pain, and a rash on his legs. He states that his symptoms began about 1 week ago. The testicular pain began about 1 day ago and has increased significantly over the last 24 hours. He denies recent illness or injury and states that has been in good health for as long as he can remember. On physical exam, the patient was well-developed, well-nourished, and in mild physical distress. His blood pressure was elevated at 152/94 mm Hg, and a chest radiograph was negative. Laboratory analysis revealed an elevated sedimentation rate and C-reactive protein, elevated BUN, and creatinine. His red blood cell count was decreased, and his ANCA was negative. An arteriogram showed diffuse arterial saccular aneurysms and narrowing of the arteries. Question What is the most likely cause of this patient's symptoms?

Correct answer: Polyarteritis nodosa Explanation Polyarteritis nodosa is a rare autoimmune disease that can affect medium-sized arteries in any organ of the body. The cause is unknown. It commonly affects nerves, skin, intestines, muscles, and joints. It is seen more in middle-aged people and results in decreased blood supply to affected body parts. Men and women are affected approximately equally. There has been some association made with hepatitis B. 3 criteria need to be met out a list of possible symptoms before the diagnosis of polyarteritis nodosa can be made. These include: muscle pain, a positive hepatitis B surface antigen or antibody test, elevated BUN or creatinine, livedo reticularis (a mottled purplish skin discoloration over the extremities or torso), an elevated diastolic blood pressure (greater than 90 mm Hg), weight loss of 4 kg or more, a biopsy positive for vasculitis, dilated arteries or constricted blood vessels (seen by arteriogram), testicular pain or tenderness, or nerve disease. It is diagnosed by examining the tissues. This is usually achieved through a biopsy. Histological analysis usually shows focal necrotizing arteritis and a mixed cellular infiltrate within the walls of the affected blood vessels. Testing for an elevated sedimentation rate and C-reactive protein will also help with the diagnosis. Patients often have an elevated white blood cell count and a decreased red blood cell count. Urinalysis may show proteinuria and hematuria. Polyarteritis nodosa is usually treated with high doses of cortisone and immunosuppressive drugs such as cyclophosphamide or azathioprine. If the patient also has hepatitis B, antiviral medications (e.g., interferon-alpha) may be included in treatment. Kawasaki disease is normally found in children. The biopsy results are characteristic of polyarteritis nodosa, but none of the other choices. The negative ANCA also makes SLE unlikely.

A 62-year-old man presents for evaluation of facial swelling. During the history, he states that he has also experienced headaches, dizziness, and visual problems over the past few weeks. He admits that he has a 70-pack-per-year history of smoking. Upon examination, you note swelling not only of the face, but also of the neck and upper extremities. You appreciate diminished breath sounds and tactile fremitus in the right upper lobe. Question What is the most likely diagnosis?

Correct answer: Superior vena caval obstruction Explanation The clinical picture is suggestive of superior vena caval obstruction. Patients typically present with swelling of the face and neck, dizziness, syncope, visual disturbances, and stupor. The most frequent causes are neoplasms (e.g., carcinoma of the lung, lymphoma, and mediastinal tumors).

An 80-year-old man was treated for ventricular arrhythmias. He presents 1 month later with joint pain. He also has an unusual mask-like rash over his face and body. Discontinuation of drug therapy causes the symptoms to abate. What drug was most likely administered to this patient?

Correct answer: Procainamide Explanation The correct response is procainamide. This question addresses issues related to the treatment of arrhythmias and lupus-like side effects. The initial treatment should control arrhythmias using lidocaine, unload the heart using a balanced vasodilator such as nitroglycerin or nitroprusside, and use aspirin and heparin to prevent further clot formation. Once discharged, patients are frequently placed on atenolol to control catecholamine-induced arrhythmias. Finally, long-term control of arrhythmias is accomplished using drugs such as procainamide or quinidine. Unfortunately, procainamide can cause lupus-like side effects. The use of thrombolytics has reduced myocardial damage caused by thrombus formation. Streptokinase, urokinase, and tissue plasminogen activator (tPA) all will lyse clots. This leads to coronary reperfusion. If performed within the first several hours post-clot, myocardium will be spared. The use of aspirin and heparin are directed at prevention of platelet aggregation and clot stabilization. Tocainide and amiodarone are antiarrhythmics, but they do not cause lupus-like side effects. Propranolol is a beta-blocker; it does not cause lupus-like side effects. Phenytoin is an anticonvulsant, not an antiarrhythmic, but it may cause lupus-like side effects in a small number of people.

A 23-year-old woman gives birth to a healthy male infant in the hospital. On postnatal day 6, she begins to experience some abdominal discomfort as well as cramping lower pelvic pain. Suddenly, she begins to bleed profusely from her vagina. A rapid clinical assessment reveals a boggy sub involuted uterus. Her temperature is 37 degrees centigrade; systolic BP is 70. Heart rate is 130, and she appears pale. Her heart and lungs seem normal. Question With secondary post partum hemorrhage as her working diagnosis, what is the next best step in management?

Correct answer: Resuscitation with fluids Explanation Resuscitation with fluids is the correct answer. Regardless of the cause of hemorrhage, when a patient is exsanguinating, the first priority is to restore a degree of hemodynamic stability before the patient descends into irreversible shock. From the given blood pressure, heart rate, and general appearance, in addition to an obvious source of blood loss, hemorrhagic shock is the diagnosis; therefore, resuscitation should be the 1st step.

A 45-year-old woman with no significant past medical history presents with a 4-month history of a dull, aching heaviness sensation in her proximal right leg. She notes that this sensation is provoked by extended periods of standing and walking, and is relieved when she lies in a recumbent position. Her past medical history is remarkable for pregnancy 4 times, the most recent being approximately 2 years ago. She denies a history of smoking, trauma, injuries, fever, chills, chest pain, shortness of breath, hemoptysis, cough, skin changes and coolness, and peripheral edema. Her physical exam reveals several dilated, tortuous, elongated veins along the medial right thigh, which are especially pronounced upon standing. The remainder of the physical exam is normal. Question What is most appropriate therapeutic approach for this patient at this time?

Correct answer: Sclerotherapy Explanation This patient's diagnosis is varicose veins. Sclerotherapy can be used to treat varicose veins. It involves the injection of an irritating solution into the varicose vein to promote an inflammatory response, scarring, and obliteration of the lumen. Warfarin is indicated for myocardial infarction or cerebrovascular accident prevention and in patients with atrial fibrillation, mechanical heart valves, or deep venous thrombosis. Diuretics are ineffective for varicose veins. Cilostazol and clopidogrel both inhibit platelets. Clopidogrel is used in CAD and CVA prevention, as well as post-ST-segment MI. Cilostazol lowers lipid levels and is indicated in peripheral arterial disease.

A 43-year-old patient presents with a 3-hour history of acute onset of palpitations and dyspnea. There is no other significant medical history. Chest X-ray and echocardiography are normal. EKG reveals absence of P wave, with variable R-R interval with atrial rate of 300/min and ventricular rate of 120/min. Patient is afebrile, radial pulse is 90/min, BP is 110/70 mm Hg, and RR is 18/min. A diagnosis of newly detected atrial fibrillation is made. Question What is the next course of action?

Correct answer: Start diltiazem therapy Explanation The correct response is to start diltiazem therapy. In atrial fibrillation (AF), ventricular response is usually irregular. AV node is refractory because atrial fibrillation produces a large number of impulses; not all impulses reach the ventricles. The non-conducted atrial impulses due to AV nodal refractory period are known as concealed conduction; therefore, ventricular response is relatively slow compared to atrial rate. The initial management in a hemodynamically stable patient (i.e., no hypotension or angina) with atrial fibrillation involves control of ventricular rate (goal <100 beats/min) by administering diltiazem or other rate-controlling drugs (e.g., verapamil, metoprolol, esmolol, or propranolol). If there is spontaneous conversion to sinus rhythm, the patient can be discharged after assessment for the cause of AF; otherwise, the patient requires anticoagulation and electrical/chemical cardioversion at a later date. Amiodarone and propafenone are the pharmacological agents used in chemical cardioversion. AV nodal ablation and permanent pacemaker implantation are highly effective in improving symptoms in patients with AF who are experiencing symptoms related to a rapid ventricular rate, and who cannot be adequately controlled with anti-arrhythmic or negative chronotropic medications.

A 15-year-old girl presents with a 1-hour history of rapid heartbeat, faintness, sweating, and nervousness. She is also experiencing shortness of breath and chest pain. The patient has no significant past medical history. There is no history of similar episodes. The patient is on no medications, and she denies illicit drug use. On exam, BP is 70/60 mm Hg, and pulse is 200 bpm. RR is 22/min. She is afebrile. She looks pale, and her palms are slightly sweaty. She is not comfortable sitting up, so she prefers lying down. She looks slightly apprehensive. Her heart and lung exam are negative except for the tachycardia; except for cool sweaty hands, a brief abdominal and extremity exam are non-revealing. The physician quickly places the paddles on the patient's chest to record the rhythm; this shows a narrow-complex regular tachycardia at 210 bpm. He requests oxygen, IV line, and continuous monitoring. An EKG is in the process of being completed. Question How should the patient's condition be managed?

Correct answer: Synchronized cardioversion Explanation Synchronized cardioversion is the correct response. Tachycardia can be classified based on the appearance of the QRS complex on the ECG as narrow and wide complex tachycardia. Narrow complex tachycardia consists of sinus tachycardia, atrial fibrillation, atrial flutter, AV nodal reentry, and atrial tachycardia (ectopic and reentrant). Wide complex tachycardia consists of ventricular tachycardia and supraventricular tachycardia with aberrancy. The patient presents with a tachycardia and associated serious symptoms of faintness, shortness of breath, chest pain and apprehension, mild hypotension, and peripheral vasoconstriction. Therefore, while young patients can tolerate a rapid heartbeat for some time, this patient would be classified as hemodynamically unstable. The treatment of choice for unstable patients with a narrow complex tachycardia would be immediate synchronized cardioversion. However, whenever possible, it is advisable to provide analgesia and sedation for conscious patients before cardioversion. In stable patients, vagal maneuvers such as carotid sinus massage or Valsalva maneuver should be the initial approach; it should be followed by administration of adenosine. Performing a carotid sinus massage and asking the patient to perform a Valsalva maneuver are relatively innocuous means of increasing vagal tone, and there is the potential for breaking the narrow-complex tachycardia. If the vagal maneuvers do not succeed, the physician may wish to give a fluid bolus and try medications such as adenosine (6mg rapid IV push over 3 seconds). Adenosine works in 90% of narrow complex tachycardias, and with vagal maneuvers, it can be viewed as the initial management choice. The adenosine can be repeated twice at doses of 12 mg IV; the doses must be spaced at 1 - 2 minutes apart, and the adenosine must be injected into a vein that is close to the heart (at the very least, a brachial vein with elevation of the arm); it should be followed by a 20cc saline push. If there is still no rate conversion, then diltiazem or verapamil can be administered.

A 75-year-old African-American man presents with a 5-month history of gradually progressive dyspnea that is especially pronounced when climbing stairs. He also has been noticing that his ankles and lower legs have "gotten larger" over roughly the same time period, which no longer allows him to fit into his sneakers. He denies fever, chills, chest pain, palpitations, cough, pleurisy, calf pain, abdominal complaints, sick contacts, or travel. His psychosocial history is noteworthy for chronic alcohol use. His physical exam reveals bibasilar rales, JVD of 5cm, an S3 gallop, a holosystolic murmur at the apex that radiates to the left axilla, and 2+ pitting edema to the level of the mid-calves bilaterally. A bedside echocardiogram was remarkable for biventricular enlargement. Question What additional physical exam finding would be expected in this patient?

Correct answer: Tachycardia Explanation This patient's presentation is significant for dilated cardiomyopathy. Dilated cardiomyopathy occurs more often in African-Americans than Caucasians, and it occurs in men more frequently than women. Often no cause can be identified, but chronic alcohol abuse, major catecholamine discharge, myocarditis, postpartum state, and doxorubicin are frequent causes. Chronic tachycardia may also precipitate a dilated cardiomyopathy that may improve over time if rate control can be achieved. Amyloidosis, sarcoidosis, hemochromatosis, and diabetes may rarely present as dilated cardiomyopathies, as well as the more classic restrictive picture. Signs and symptoms of heart failure develop gradually and include rales, an elevated jugular venous pressure (JVP), cardiomegaly, S3 gallop, murmurs of functional mitral or tricuspid regurgitation, peripheral edema, or ascites. In severe CHF, Cheyne-Stokes breathing, pulsus alternans, pallor, and cyanosis may be present. Sinus tachycardia is common. Other common ECG abnormalities include left bundle branch block and ventricular or atrial arrhythmias. The chest radiograph reveals cardiomegaly, evidence for left and/or right heart failure, and pleural effusions.

A 3-month-old male infant presents for a routine evaluation. His mother states that the child is gaining weight and feeding appropriately and has been without fever, chills, dyspnea, or other abnormal objective signs. Upon physical examination, the examiner noticed a loud, harsh holosystolic murmur in the left third and fourth interspaces along the sternum that was associated with a systolic thrill. There were no other abnormalities. Question What is correct regarding this patient's diagnosis?

Correct answer: The smaller the defect is, the louder the associated murmur. Explanation This patient's presentation represents a ventricular septal defect (VSD). Congenital VSDs occur in various parts of the ventricular septum. The smaller the defect, the greater the gradient from the left ventricle to the right ventricle and the louder the murmur. Newly-arising VSDs are uncommon in adults. A left-to-right shunt is present unless there is associated right ventricular hypertension. Patients with the typical murmur as the only abnormality have a normal life expectancy; however, they have an increased risk of associated bacterial endocarditis in smaller shunts. Membranous and muscular septal defects may spontaneously close in childhood as the septum grows and hypertrophies; small shunts in asymptomatic patients do not require surgery or other intervention.

A 62-year-old man with a 15-year history of hypertension presents with severe tearing chest pain radiating through to the back. His blood pressure is 180/110 mmHg, heart rate is 120 BPM, and respiratory rate is 34/min. Physical exam findings include neck negative for bruits/jugular venous distension (JVD), lungs clear to auscultation, heart regular rhythm despite tachycardia, normal S1/S2 with an S4 present, and a grade III/IV diastolic rumbling murmur noted with the patient leaning forward. Radial pulses are 1+ on right and 3+ on left. EKG reveals a sinus tachycardia and evidence of left ventricular hypertrophy. Question What is the most likely etiology of this patient's symptoms?

Correct answer: Thoracic aortic dissection Explanation The clinical picture is suggestive of a thoracic aortic dissection. This condition is most common in men between the 6th and 7th decades of life. On physical examination, peripheral pulses and blood pressures may be increased, diminished, or unequal. A diastolic murmur of aortic insufficiency may be heard.

Case Ico-delete Highlights A 72-year-old man with a history of poorly controlled HTN and previous myocardial infarction presents with a nocturnal cough, bilateral ankle swelling, and dyspnea on exertion. He denies any fever, chills, URI symptoms, chest pain, headache, N/V, diaphoresis, or syncope. He further denies smoking, alcohol, or drug use. Physical exam reveals bipedal edema and bibasilar crackles. A chest x-ray is remarkable for enlargement of the cardiac silhouette and interstitial infiltrates, while EKG analysis indicates deep S waves in lead V1 and tall R waves in lead V5. Question What is the most appropriate next step in the evaluation and management of this patient?

Correct answer: Transthoracic echocardiography Explanation This patient has clinical manifestations and initial diagnostic test results suggestive of congestive heart failure. The most useful test is an echocardiogram, which is a noninvasive and ionizing-free imaging modality that reveals the size and function of both ventricles and atria. Furthermore, it can be used to evaluate for the presence of pericardial effusions, valvular abnormalities, intracardiac shunts, and segmental wall-motion abnormalities. An endocardial biopsy may aid in the diagnosis of restrictive cardiomyopathy and constrictive pericarditis but is invasive. Chest CT scanning offers the superior delineation of anatomic features to that of plain radiographs; the use of contrast serves to distinguish vascular from nonvascular structures. It does not, however, measure cardiac function. Coronary angiography measures left ventricle ejection fraction and identifies regional wall motion. However, it is an invasive procedure and should follow an echocardiogram. Bronchoalveolar lavage is indicated in the bronchoscopic investigation of infiltrative, malignant, occupational, and infectious lung diseases.

Question While doing rounds one morning, you come upon a 42-year-old man who uses IV drugs and presents with a fever and Roth spots. Based on your most likely diagnosis, which of the following findings would be the most definitive for the diagnosis?

Correct answer: Two positive blood cultures with Staphylococcus aureus and development of a new regurgitant murmur Explanation Clinical criteria, also known as the Modified Duke criteria, is widely utilized to establish the diagnosis of endocarditis. The criteria are classified as either major criteria, which include: 2 positive blood cultures for a microorganism that typically causes endocarditis; evidence of endocardial involvement via an echocardiogram (vegetation, abscess); development of a new regurgitant murmur. Minor criteria include: vascular phenomena (skin hemorrhages, emboli, aneurysms, or pulmonary infarction); fever >100.4°F (38°C); immunologic phenomenon (glomerulonephritis, Osler's nodes, Roth spots, rheumatoid factor); and positive blood cultures that do not meet the specifics of the major criteria. The correct answer is 2 positive blood cultures with Staphylococcus aureus and development of a new regurgitant murmur. Since 2 major criteria are identified, a definitive diagnosis of infective endocarditis can be made with 80% accuracy. The presence of 1 major criterion and 3 minor criteria, or even if there are 5 minor criteria listed, can also qualify in this 80% accuracy diagnosis range. The diagnosis is possible but not highly likely to be infective endocarditis if the patient displays the following: 1 major and 1 minor criteria, or 3 minor criteria. Any less than these should lead a healthcare provider to suspect a different diagnosis. One positive blood culture with Staphylococcus aureus with Osler's nodes and Roth spots contains 2 minor criteria. Two positive blood cultures with Streptococcus pneumoniae with cutaneous hemorrhages and glomerulonephritis has 1 major and only 2 minor criteria. Evidence of endocardial vegetation on echocardiography with Osler's nodes has only 1 major and 1 minor criteria. Fever >100.4°F (38°C) with evidence of endocardial vegetation on echocardiography and glomerulonephritis has 1 major and 2 minor criteria.

A 3-month-old male infant presents for a well-baby check. There have been no other changes since the last visit. Upon exam, the infant is pink and well-appearing. Cardiovascular exam reveals a grade III/VI high-pitched, harsh pansystolic murmur heard best at the left sternal border, 4th intercostal space (ICS). No additional murmurs are heard. The remainder of the exam is unremarkable. Question What is the most likely diagnosis?

Correct answer: Ventricular septal defect Explanation The correct answer is ventricular septal defect (VSD). The patient is usually relatively asymptomatic. Classically, there is a harsh, pansystolic murmur heard best over the 4th intercostal space and the left upper sternal border. Atrial septal defect (ASD) is characterized by a right ventricular heave and an ejection systolic murmur heard at the pulmonic area. Coarctation of the aorta typically presents with pulse lag in the lower extremities and a blowing systolic murmur heard best in the left axilla and back. Aortic regurgitation is a diastolic murmur that is heard in the right upper sternal border. Patent ductus arteriosus (PDA) is typically characterized by bounding pulses and a rough, machinery murmur that is heard at the left sternal border or left infraclavicular area. It is often seen with VSD and coarctation of the aorta.

A 62-year-old man with a 15-year history of hypertension presents with severe, tearing chest pain radiating through to the back. Blood pressure is 180/110 mmHg, heart rate is 120 bpm, and respiratory rate is 34/min. Physical examination findings include neck negative for bruits/JVD, lungs clear to auscultation, regular heart rhythm, normal S1/S2 with an S4 present, and a grade III/IV diastolic rumbling murmur noted with the patient leaning forward. Radial pulses are 1+ on right and 3+ on left. EKG reveals a sinus tachycardia and evidence of left ventricular hypertrophy. A STAT chest X-ray is obtained. Question What finding is most consistent with the presumptive diagnosis?

Correct answer: Widening of the superior mediastinum Explanation With an aortic dissection, the most common finding on chest X-ray is a widened mediastinum. Prominent pulmonary hila are associated with pulmonary hypertension. Kerley B lines are usually seen with pulmonary edema and congestive heart failure. Blunted costophrenic angles are associated with pulmonary effusions. There are numerous conditions that can produce prominent cardiac shadowing, but it is not seen in an aortic dissection.

A 20-year-old student presents with a "weird" heart sound. He indicates that while in anatomy lab, they were practicing with stethoscopes and listening to other classmate's heart sounds as a part of the course curriculum. One student said that his heart had a rumbling sound. On further history of the patient, he indicated that he had acute rheumatic fever twice as a teen. Question How is the murmur best auscultated in the most likely diagnosis for this patient?

Correct answer: With the patient lying on the left side, at the apex of the heart Explanation The murmur of mitral valve stenosis can be best heard at the apex with the patient in the left lateral position (lying on left side). The most common cause of mitral stenosis is rheumatic fever. Since the murmur can be best heard at the apex, all options with "base of the heart" are eliminated. Standing decreases the heart size and decreases the sound of mitral stenosis. Valsalva maneuver will decrease the sound of the murmur.

A 79-year-old man with a past medical history of diabetes mellitus, hypertension, and hyperlipidemia presents with severe chest pain and dyspnea. He appears pale, apprehensive, and diaphoretic. He is in a confused state, and he is agitated. His pulse is weak and tachycardic, with a systolic blood pressure of 60 mm Hg. He has a narrow pulse pressure, tachypnea, a weak apical impulse, and significant jugular venous distention. His lungs are free of crackles. Bedside electrocardiogram reveals ST-segment elevations in the anterior and septal leads. Question What is the preferred initial pharmacologic agent of choice for this patient?

Dopamine Explanation This patient's exhibits signs and symptoms of cardiogenic shock due to myocardial infarction. Treatment of cardiogenic shock includes general supportive measures of oxygen, aspirin, heparin, and 'gentle' fluid challenges (250 cc) if there is no overt pulmonary edema. For more serious hypotension (MAP < 70 mm Hg), dopamine or norepinephrine may be given, with a target systolic pressure of 80 to 90 mm Hg (and not > 110 mm Hg). Norepinephrine is a potent alpha-adrenergic agonist with minimal beta-adrenergic agonist effects. Norepinephrine can increase blood pressure successfully in patients who remain hypotensive following dopamine. Dopamine is a precursor of norepinephrine and epinephrine and has varying effects according to the doses infused. A dose of less than 5 mcg/kg/min causes vasodilation of renal, mesenteric, and coronary beds. At a dose of 5 - 10 mcg/kg/min, beta1-adrenergic effects induce an increase in cardiac contractility and heart rate. At doses of approximately 10 mcg/kg/min, alpha-adrenergic effects lead to arterial vasoconstriction and an elevation in blood pressure. The blood pressure increases primarily as a result of the inotropic effect. The undesirable effects are tachycardia, increased pulmonary shunting, as well as the potential for decreased splanchnic perfusion and increased pulmonary arterial wedge pressure. In the setting of acute myocardial infarction (MI), dobutamine use could increase the size of the infarct because of the increase in myocardial oxygen consumption that may ensue. In general, avoid dobutamine in patients with moderate or severe hypotension (e.g., systolic blood pressure < 80 mm Hg) because of the peripheral vasodilation. If hypotension is moderate (e.g., mean arterial pressure [MAP] 70 to 90 mm Hg), dobutamine infusion may be used to improve cardiac output and reduce left ventricular filling pressure. Tachycardia and arrhythmias occasionally occur during dobutamine administeration, especially at higher doses.

A 37-year-old Caucasian man has presented with shortness of breath (SOB). Further history reveals that the patient has been extremely fatigued in the last few weeks, experiencing excessive night sweats, possesses a worsening cough, chest pain, as well as general aches and pains. The patient currently is not taking any prescribed medications and is allergic only to penicillin. He has a history of on-and-off intravenous drug use and admits to last using around 1 month ago. Along with an urgent inpatient admission, you are planning to initiate orders to have the patient undergo an echocardiogram and obtain blood cultures, among other actions. Question Based on the most likely diagnosis for this patient, which of the following pharmaceutical interventions is most appropriate?

Intravenous vancomycin Explanation The patient in the above scenario is experiencing infective endocarditis, most likely secondary to his injection drug use. In injection drug users, Staphylococcus aureus accounts for at least 60% of bacterial endocarditis cases; up to 90% of patients in these similar scenarios will have the tricuspid valve infected. Typically, all patients with infective endocarditis will have a fever during the course of the disease; this is evident through the patient admitting to having significant night sweats. Duration is usually a few days and progressively worsens over a matter of weeks, which is consistent with the case. Symptoms can include dyspnea, cough, arthralgias, arthritis, and diarrhea, as well as abdominal, back, or flank pain. Blood cultures will confirm the diagnosis of infective endocarditis. An echocardiogram will show the overall mechanical function of the heart as well as possible evidence of the involved valves, such as vegetation. In regard to pharmaceutical treatment, intravenous vancomycin or gentamycin would be the most appropriate choice for the patient scenario. All penicillins are ruled out due to the patient's history of a penicillin allergy; therefore, intravenous nafcillin and oxacillin are incorrect choices. Intravenous cefazolin, a first-generation cephalosporin, would not be the best answer due to the patient's allergy. Although the risk of an allergic reaction is low, usually around 15%, this answer is not the most acceptable. Intravenous corticosteroids, as well as intravenous diuretics, would not begin to resolve infective endocarditis.

Kussmaul breathing

Kussmaul breathing is a deep, labored, and gasping breathing pattern seen often in severe metabolic acidosis (diabetic ketoacidosis, renal failure). In metabolic acidosis, breathing is first rapid and shallow, but later on, as acidosis worsens, breathing gradually becomes Kussmaul breathing.

Still murmur

Still murmur is recognized as the most common innocent murmur of early childhood. It is described as musical, vibratory, short, and high-pitched.

An elderly man presents with an acute myocardial infarction. He quickly develops significant signs and symptoms of cardiogenic tamponade; it subsequently progresses to shock. Signs and symptoms include hypotension, altered mental status, cold clammy skin, as well as metabolic acidosis evident on laboratory tests. Question Given the most likely diagnosis, what diagnostic study will be most helpful in demonstrating the severity of this patient's condition?

hide Correct answer: Lactate levels Explanation The correct response is lactate levels. Early identification of shock is a necessity to help improve any patient's chance of survival and recovery. It is also critical to lead to reversing the cause of the shock and initiating early resuscitation efforts. Early on, patients suffering from shock may have only a few or very subtle symptoms such as tachypnea, tachycardia, hyper or hypothermia, weak or bounding peripheral pulses, delayed capillary refill, or even pale or cool skin. Decreased mental status, weak or absent central pulses, central cyanosis, hypotension or bradycardia are ominous and later signs indicating the shock has progressed. Shock is generally classified into 4 major categories, depending on its etiology: cardiogenic, hypovolemic, distributive, or obstructive. Cardiogenic shock occurs as a consequence of cardiac pump failure. Myopathic, mechanical, or arrhythmic issues lead to cardiogenic shock. Whatever the cause there is typically an acute loss of 15 - 20% of circulating blood volume that leads to lack of blood to pump throughout the circulatory system. The lactate level is a critical diagnostic study in determining the severity of shock that the patient is experiencing. Alterations of the lactate level will provide and early and extremely objective evaluation of not only a patient's overall response to treatment of the shock, it will also lend a hand to showing the prognosis of how that patient will recover from the episode of shock. Lactate is the product created from cell metabolism and can dangerously accumulate with there is a state of insufficient oxygen levels, or hypoxia. For this reason, no matter what category of shock the patient may be experiencing, lactate levels can be viewed as an indirect marker of oxygen debt, tissue hypoperfusion, and overall severity of the episode of shock.

A newborn child is routinely evaluated in the pediatrician's office 1 month following delivery. The mother reports that the patient is behaving normally and is feeding well. The physical exam is remarkable for a murmur, which is located at the 2nd left intercostal space. The murmur is continuous throughout cardiac systole and diastole, nonradiating, and of a "machinery" quality. There is additionally a widened pulse pressure. The skin and mucosa are without cyanosis, and there is no evidence of fluid retention. Question What is the most likely diagnosis?

hide Correct answer: Patent ductus arteriosus Explanation This infant is most likely exhibiting signs consistent with patent ductus arteriosus. Patent ductus occurs more commonly in females and has an incidence rate of up to 50% in infants born with a birth weight of less than 1 kilogram. Patent ductus arteriosus is characterized by an abnormal patency of the ductus arteriosus, which delivers blood from the right side of the heart to the systemic circulation during fetal life. Blood typically shunts from the higher-pressured left side (systemic circulation) to the lower-pressured right side (the pulmonary circulation). Normally, this ductus closes following spontaneous respiration of the newborn, as well as a lowering of endogenous prostaglandin and adenosine levels. The patency is normally sealed with fibrous tissue following a duration of 2-3 weeks. Ventricular septal defect is the most common congenital cardiac malformation. Blood flows from the left ventricle to the right ventricle and presents as a harsh, blowing holosystolic murmur with a thrill localized to the fourth left intercostal space. This murmur may decrease with Valsalva and handgrip. Atrial septal defect is characterized by a shunting of blood from the left atrium to the right atrium, whose murmur is mild in intensity, located in the pulmonic area, and is associated with a widely-split S2 heart sound.

A 63-year-old woman with a significant history of hypertension, hyperlipidemia, and myocardial infarction presents to the emergency room with shortness of breath at rest. She has found it difficult to walk short distances due to shortness of breath. Additionally, she complains of orthopnea, nocturnal dyspnea, and generalized abdominal discomfort. She denies cough, fever, chills, anxiety, chest pain, pleurisy, nausea, vomiting, diarrhea, rashes, lightheadedness, and syncope. She is afebrile, but tachypnic and diaphoretic. There is a diminished first heart sound, S3 gallop, laterally displaced PMI, bibasilar rales and dullness to percussion, and expiratory wheezing noted. The abdominal exam reveals distension, with hepatomegaly in the right upper quadrant. There is 2+ pitting edema of the lower extremities to the level of the mid calf, and the extremities are cool. Question What additional finding is expected in this patient?

orrect answer: Elevated jugular venous distension Explanation This patient's presentation is most consistent with congestive heart failure. Manifestations reflect impaired pumping ability of the heart, reduced renal blood flow, and activation of the sympathetic compensatory mechanisms. In chronic heart failure, patients frequently appear malnourished and occasionally cachectic. Due to frequently increased sympathetic activity in patients with heart failure, there may be pallor and coldness of the limbs and cyanosis of the digits because of vasoconstriction. Elevations in the jugular venous height may be measurable. Systemic venous hypertension can be detected by abnormal distention of the internal jugular veins. Although the jugular venous pressure normally declines on inspiration, it can rise in patients with right-heart failure (Kussmaul sign). Persistent elevation of the jugular venous pressure is one of the earliest and most reliable signs of right-heart failure. The cardiac exam may reveal a parasternal lift, an enlarged and sustained left ventricular impulse, a diminished first heart sound, S3 gallop, and an S4 with diastolic heart failure. Contributory murmurs may also be evident. Other common symptoms of heart failure include fatigue, exercise intolerance, and generalized weakness, particularly in the limbs. These symptoms are secondary to low cardiac output with decreased perfusion of skeletal muscles and can occur with exertion or at rest; they may be worsened after eating because of the increased splanchnic demand for blood flow, which may stress the limited reserve. Fluid retention results in symmetrical peripheral pitting edema.

Ataxic

Ataxic breathing (Biot's respiration) is an abnormal pattern of breathing characterized by groups of quick, shallow inspirations followed by regular or irregular periods of apnea. It is generally a poor prognostic sign. Biot's respiration is caused by the medullary lesion due to strokes or trauma or by pressure on the medulla due to uncal or tentorial herniation.

Cheyne-Stokes respiration

Cheyne-Stokes respiration is an abnormal pattern of breathing characterized by repetitive progressively deeper and sometimes faster breathing, followed by a gradual decrease in breathing that results in temporary apnea. Cycles usually take 30 seconds to 2 minutes. Increased CO2 during the period of apnea causes compensatory hyperventilation. Hyperventilation in turn causes the decrease in CO2, which causes apnea and the cycle to restart. Causes include CNS dysfunction, cardiac failure with low cardiac output, sleep, hypoxia, or profound hypocapnia.

A 75-year-old African American man presents with a 5-month history of gradually progressive dyspnea that is especially pronounced when climbing stairs. He also has been noticing that his ankles and lower legs have "gotten larger" over roughly the same time period, which does not allow him to fit into his sneakers any longer. He denies fever, chills, chest pain, palpitations, cough, pleurisy, calf pain, abdominal complaints, sick contacts, or travel. His psychosocial history is noteworthy for chronic alcohol use. His physical exam reveals tachycardia, bibasilar rales, jugular vein distention (JVD) of 5 cm, an S3 gallop, and 2+ pitting edema to the level of the mid-calves, bilaterally. Question What is the expected echocardiogram finding in this patient?

Correct answer: 4-chamber dilatation with thin left ventricular walls Explanation This patient's presentation is significant for dilated cardiomyopathy. Dilated cardiomyopathy occurs more often in African-American populations than in Caucasians, and in men more often than women. Often no cause can be identified, but chronic alcohol abuse, major catecholamine discharge, myocarditis, being postpartum, and doxorubicin are frequent causes. On echocardiography, 4-chamber dilatation is a common, but not uniform, finding. Also present is an increased LV end-diastolic diameter and volume with decreased fractional shortening, thinning LV walls, increased E point-septal separation, LA enlargement, and limited mitral and aortic valve opening (caused by low stroke volume). Intracardiac thrombi are frequently observed and are most often found in the LV apex. M-mode imaging of the mitral leaflets may demonstrate a B bump, or notch, just before systolic valve closure, indicating elevated LV diastolic pressure. Mitral annular dilatation and secondary MR are common. Asymmetric septal hypertrophy is found in hypertrophic cardiomyopathy. Typical echocardiographic findings of restrictive cardiomyopathy include a diffuse increase of ventricular thickness in the absence of marked ventricular chamber dilation and marked biatrial enlargement. Valvular vegetations are indicative of infective endocarditis, while abnormal segmental wall motion during systole or diastole signifies ischemic coronary heart disease.

While making rounds in the NICU, you are asked to evaluate an 8-hour-old male neonate; he is in respiratory distress. On evaluation, you note the neonate to be in mild (but progressing) respiratory distress. He has minimal perioral cyanosis. He is tachycardic and tachypneic. Blood pressure is stable, but you note a blood pressure discrepancy between the upper and lower extremities. He also has absent femoral pulses. Question You place the appropriate consolations; in the meantime, what is the drug of choice to order at this time?

Correct answer: A prostaglandin Explanation The correct response is a prostaglandin. This neonate likely has coarctation of the aorta. Coarctation of the aorta (CoA) is caused by narrowing in the aortic arch causing obstruction of left ventricular blood flow.1 Classically, it affects male patients 3 times more frequently than female patients.1 The patient having an increased difficulty in breathing at 8 hours of age likely indicates the ductus arteriosus is closing (typically closes 6 - 10 hours after birth).1 A patent ductus arteriosus (PDA) allows for left to right blood shunting, and therefore acts as a compensating mechanism. Once the PDA closes, there is an abrupt increase in afterload, which leads to respiratory distress. The goal of pharmaceutical therapy is to keep the ductus arteriosus open, reduce afterload, and maintain blood pressure. At this time, giving a prostaglandinto keep the ductus arteriosus patent will allow the patient to continue to compensate until definitive surgical treatment is completed.

A 70-year-old woman presents with shortness of breath at rest over the past 3 days. She has found it difficult to walk short distances due to shortness of breath. Additionally, she is experiencing confusion, orthopnea, nocturnal dyspnea, and lightheadedness. She denies cough, fever, chills, diaphoresis, anxiety, chest pain, pleurisy, nausea, abdominal pain, vomiting, diarrhea, rashes, and syncope. On physical examination, the patient is short of breath, requiring numerous pauses during conversation. She is afebrile; however, she is tachycardic, diaphoretic, and her extremities are cool. There is a diminished first heart sound, S3 gallop, laterally displaced PMI, bibasilar rales, and dullness to percussion and expiratory wheezing noted. An elevated JVD and 2+ pitting edema of the lower extremities is evident. Question What statement regarding this patient's condition is correct?

Correct answer: Activation of the renin-angiotensin-aldosterone system occurs Explanation This patient's presentation is most consistent with congestive heart failure (CHF). In developed countries, coronary heart disease with resulting myocardial infarction and loss of functioning myocardium (ischemic cardiomyopathy) is the most common cause of systolic heart failure. The most frequent cause of diastolic cardiac dysfunction is LVH, commonly resulting from hypertension; however, conditions such as hypertrophic or restrictive cardiomyopathy, diabetes, and pericardial disease can produce the same clinical picture. The most common causes of CHF in the United States are CAD, systemic hypertension, nonischemic dilated cardiomyopathy, and valvular heart disease. Rare causes of dilated cardiomyopathy include infiltrative diseases (hemochromatosis, sarcoidosis, amyloidosis, etc.), other infectious agents, metabolic disorders, cardiotoxins, and drug toxicity. In CHF, lower cardiac output leads to a reduction of renal blood flow and glomerular filtration rate, causing sodium and fluid retention. In CHF, the renin-angiotensin-aldosterone system is activated, leading to further increases in peripheral vascular resistance and left ventricular afterload as well as sodium and fluid retention. Heart failure is also associated with increased circulating levels of arginine vasopressin, which also serves as a vasoconstrictor and inhibitor of water excretion.

Case Ico-delete Highlights A 63-year-old woman presents with a 1-hour history of left shoulder pain and nausea. She has a past medical history of coronary artery disease and had a stent placed 5 years ago. An ECG shows large R waves and ST segment depression in leads V1, V2, and V3. Question These ECG findings are most consistent with what condition?

Correct answer: Acute posterior myocardial infarction Explanation The correct answer is acute posterior myocardial infarction, as the ECG findings are consistent with this diagnosis. ST depression and larger R waves in V1, V2, V3 are consistent with acute posterior myocardial infarction. T wave inversion without significant Q waves would suggest ischemia without myocardial infarction (MI). ST segment elevation and wide Q waves in leads I and AVL would suggest a lateral MI. ST segment elevation and wide Q waves in leads II, III, and AVF would suggest an inferior MI. ST segment elevation and wide Q waves in the anterior leads (V1, V2, V3, and/or V4) would suggest an anterior myocardial infarction.

On routine EKG, a 59-year-old Caucasian man is found to have developed new-onset atrial fibrillation with a ventricular response rate of 98 beats per minute. The atrial fibrillation is asymptomatic; it has not been associated with dizziness, palpitations, chest pain, or shortness of breath. Duration of atrial fibrillation is unknown. His last EKG was 6 months ago. The patient's past medical history is notable for hypertension and coronary artery disease with prior inferior MI and RCA stent. He has no history of stroke or TIA. Allergies: bee stings. Usual home medications: aspirin 81 mg daily, hydrochlorothiazide 25 mg daily, metoprolol succinate (Toprol XL) 25 mg daily. He quit smoking 20 years ago. The patient does not want to take warfarin anticoagulation because of the need for frequent lab testing with warfarin. The patient's dose of beta-blocker is adjusted. Echocardiogram and thyroid function tests are ordered for further evaluation. In addition, a stress echo test is ordered to rule out the progression of CAD. The option of cardioversion and the timing thereof is discussed with the patient. Question What other action is most appropriate for this patient?

Correct answer: Advise the patient that his risk of stroke is increased, and recommend that he take a novel anticoagulant for stroke prevention. This patient has hypertension and vascular disease with prior MI. His CHA2DS2-VASC score is 2; oral anticoagulation is recommended.

A 66-year-old man with a past medical history of myocardial infarction 2 years ago, aortic regurgitation, congestive heart failure, atrial fibrillation, and chronic obstructive pulmonary disease is presently being monitored in the hospital. Myocardial infarction has been ruled out. An EKG performed upon admission revealed significant Q waves in the anterior leads but no evidence of an acute myocardial infarction. A diagnostic echocardiogram confirms moderate aortic and mitral valve regurgitation and a left-ventricular ejection fraction of 30%. He denies any complaints upon bedside evaluation. His physical exam reveals a blood pressure of 95/55 mm Hg and tachycardia. Continuous bedside ECG monitoring notes wide, monomorphic QRS complexes with a heart rate of 160 beats per minute that spontaneously resolve within 20 seconds, reverting to the pattern identified upon admission. Question What antiarrhythmic agent would be the pharmacologic treatment of choice in the management of this patient?

Correct answer: Amiodarone Explanation The bedside electrocardiogram was remarkable for nonsustained, monomorphic ventricular tachycardia. The patient with hemodynamically stable VT in the setting of significant left ventricular dysfunction should be treated with intravenous amiodarone. When antiarrhythmic drug therapy is chosen to prevent recurrence or when VT is accompanied by hemodynamic instability, amiodarone is the treatment of choice. Patients in VT with hemodynamic compromise, congestive heart failure, chest pain, or ischemia should be treated promptly with DC cardioversion. Ventricular tachycardia in association with acute MI may be treated with lidocaine, but this is not first-line therapy in other circumstances. If the patient is hemodynamically stable and has normal or only mild left ventricular dysfunction, intravenous procainamide may promote conversion. In patients with stable sustained VT, intravenous procainamide is a reasonable first choice. However, intravenous procainamide may cause hemodynamic instability because of its negative inotropic effects. Given this patient's low blood pressure, it should thus be avoided. It should also be avoided in CHF or in patients with a prolonged QT interval.

A 65-year-old man presents with a 15-minute history of left-sided chest pain, which he describes as "crushing". He denies any trauma to the chest; he was sitting at home reading when it began. On examination, there is an apical systolic murmur. He is somewhat lightheaded and hypotensive. He has a history of coronary artery disease and has occasionally experienced chest pain with exertion; however, this is the first time he has experienced chest pain without any exertion. The first ECG on admission shows some ST segment depression, but this is not present on an ECG taken 30 minutes later. Chest X-ray reveals no abnormalities. D-dimer is normal but C-reactive protein is elevated. Question What is the most probable cause of his chest pain?

Correct answer: Unstable angina Explanation Unstable angina pain is usually described as tightness or pressure on the chest, and it lasts for 10-20 minutes. It is usually left-sided or retrosternal, and it may radiate to the jaw and neck. It occurs at rest or may be precipitated by physical exertion and emotional stress. On examination, a systolic murmur of mitral regurgitation may be auscultated during the pain. High-sensitivity troponin tests are often positive. C-reactive protein is another marker associated with unstable angina.

A 66-year-old woman with a history of a multinodular goiter presents to the office after a recent hospitalization for a newly diagnosed cardiac arrhythmia. She relates that she has continued to see the cardiologist and is being treated with an oral medication. She denies any symptoms of hyper or hypothyroidism. She denies any change in the size of her gland or associated dysphagia or dyspnea. Thyroid function studies continue to remain in the normal range. Question When following this patient with multinodular goiter, what cardiac drug should be avoided or monitored closely due to the risk of inducing a Jod-Basedow effect?

Correct answer: Amiodarone (Cordarone) Explanation The correct answer is amiodarone (Cordarone), which is an iodine-containing class lll antiarrhythmic. It inhibits peripheral conversion of thyroxine (T4) to triiodothyronine (T3) and is 39% inorganic iodine, thus exposing the thyroid to a potentially large source of iodine. Initially, amiodarone can inhibit conversion and release of T4 and can contribute to iodine toxicity, which can result in hypothyroidism (Wolff-Chaikoff effect). This can usually be managed by decreasing the dose of amiodarone with/without thyroid hormone supplement. For unknown reasons, some patients, especially those with pre-Graves or a multinodular goiter, may then develop the Jod-Basedow effect where they overcome the initial iodine suppression and experience a surge of iodine uptake, resulting in an increased production of thyroid hormone and hyperthyroidism. This hyperthyroid state puts the patient at high risk for exacerbation of the cardiac arrhythmia. Therefore, if any new signs of arrhythmia appear in a patient treated with amiodarone, hyperthyroidism should be considered.

A 56-year-old man presents with a 1-week history of palpitations and shortness of breath. He has a long-standing history of poorly controlled hypertension. Physical examination reveals an elevated blood pressure of 190/98 mmHg, elevated jugular venous pressure (JVP), mild hepatomegaly, bilateral pedal edema, and rales at the lung bases. Diagnostic studies reveal concentric left ventricular hypertrophy without significant valvular abnormalities on echocardiogram. Question What drug is beneficial in the treatment of the patient's condition mainly by causing afterload and preload reduction?

Correct answer: Angiotensin-converting enzyme inhibitor (e.g., enalapril) Explanation Angiotensin-converting enzyme inhibitor (e.g., enalapril) reduces both preload and afterload. The above signs and symptoms suggest a diagnosis of congestive heart failure (CHF), wherein the heart fails to adequately maintain the circulation of blood. The manifestations of CHF are cardiomegaly, elevated JVP, hepatomegaly, pedal edema, and pulmonary edema. Loop diuretics (e.g., furosemide) reduce preload by reducing fluid volume. Positive inotropic agents (e.g., digoxin) act primarily by increasing cardiac contractility. They have less impact on afterload and preload. Sodium channel blocker (e.g., procainamide) is a class Ia antiarrhythmic agent used in ventricular and supraventricular arrhythmias. Arterial vasodilators (e.g., hydralazine) are used to reduce the afterload by decreasing peripheral vascular resistance and increasing heart rate, stroke volume, and cardiac output.

A 72-year-old man with known hypertension and high cholesterol presents with acute chest pain. He describes the pain as an excruciating tearing pain radiating to his neck. His blood pressure in his right arm is 105/65 mm Hg, and in his left arm 140/90 mm Hg. He also has a decreased pulse in his right arm as compared to his left. On auscultation of the chest, a III/VI early diastolic murmur is best heard at the left sternal border, third intercostal space. His EKG shows a sinus tachycardia with no acute changes. Question What is the most likely diagnosis?

Correct answer: Aortic dissection Explanation The clinical picture is suggestive of an aortic dissection. This condition is most common in the 6th and 7th decades and occurs more frequently in men. More than 2/3 of patients have a history of hypertension. The most common symptom is a sudden, severe pain with a "ripping" or "tearing" pain in the anterior chest or neck. On physical examination, peripheral pulses and blood pressures may be diminished or unequal. A diastolic murmur of aortic insufficiency may be heard.

A 62-year-old woman with a long-standing history of hypertension presents with severe headache; it started this morning and is rapidly worsening. During the interview, she suddenly collapses. Your brief examination shows that she responds with extensor posturing on external stimuli. Her deep tendon reflexes are 3, and you elicit Babinski bilaterally. You also notice that her breathing has a peculiar pattern: deep inspiration with a pause at full inspiration, followed by a brief insufficient release and the end-inspiration pause. Question How do you best describe her respiratory pattern?

Correct answer: Apneusis Explanation In the hypertensive patient with sudden loss of consciousness and decerebrate response (extensor posturing), you should consider brain stem hemorrhage. Abnormal breathing patterns can be observed in both pontine and medullary lesions; they sometimes can be prognostic. Her breathing pattern is apneustic. Apneustic breathing pattern characterizes deep, gasping inspiration with a pause at full inspiration followed by a brief, insufficient release and the end-inspiration pause before expiration. Lesion in the pons or upper medulla causes the removal of input from the vagus nerve and the pneumotaxic center. Normally, apneustic center of the lower pons promotes inspiration by stimulation of the dorsal respiratory center in the medulla to delay the 'switch off' signal of the inspiratory ramp provided by the pneumotaxic center of pons. Therefore, athe pneustic center controls the intensity of breathing. Apneusis is an ominous sign, with a generally poor prognosis.

A 62-year-old man with a 15-year history of hypertension presents with severe tearing chest pain radiating through to the back. Blood pressure is 180/110 mmHg, heart rate 120 bpm, and respiratory rate 34/min. Physical examination findings include neck negative for bruits/JVD, lungs clear to auscultation, heart regular rhythm, normal S1/S2 with an S4 present, and grade III/IV diastolic rumbling murmur noted with the patient leaning forward. Radial pulses are 1+ on right and 3+ on left. EKG reveals a sinus tachycardia and evidence of left ventricular hypertrophy. A STAT chest X-ray shows a widening of the mediastinum. Question Which of the following medication classes delivered intravenously is now needed to stabilize the patient?

Correct answer: Beta blocker Explanation Useful agents in regards to aortic dissection include beta blockers to reduce the force of contraction and heart rate, as well as to lower blood pressure. Vasodilators can be used to reduce blood pressure. Sympathomimetic drugs would elevate blood pressures, worsening the condition. Positive inotropic agents increase the strength of muscular contraction, which can worsen the condition. Alpha blockers do not reduce contractility and rate of the heart. ACE inhibitors are primarily used in the treatment of hypertension and congestive heart failure.

A 74-year-old Caucasian man presents with progressive exertional shortness of breath, lower extremity edema, and lightheadedness over the previous 7 months. He has a past medical history of HIV, hepatic cirrhosis secondary to chronic alcohol abuse and hemochromatosis, obesity, and thiamine deficiency. His symptoms are improved with rest. The physical examination reveals rales, an elevated JVP, cardiomegaly, S3 gallop rhythm, a high-pitched, blowing holosystolic murmur at the apex, peripheral edema, and abdominal distension suggestive of ascites. A bedside EKG noted sinus tachycardia with nonspecific ST-T wave changes and Q waves. An echocardiogram of the patient is shown in the image. Question What is correct regarding the clinical intervention for this patient?

Correct answer: Beta-blockers have demonstrated reductions in cardiovascular mortality Explanation This patient's presentation and the attached echocardiogram findings of dilated chambers and thin walls suggest an underlying diagnosis of dilated cardiomyopathy. Treatment of dilated cardiomyopathy is essentially the same as treatment of chronic heart failure (CHF). Beta-blockers have moved to the forefront of heart failure treatment. Several trials have shown that beta-blockers are both safe and effective in the treatment persons with dilated cardiomyopathy and that adding beta-blockers to outpatient management yields great reductions in mortality rates. Carvedilol, bisoprolol, and metoprolol CR/XL are the only agents currently approved by the US Food and Drug Administration (FDA) for use in patients with heart failure. Standard therapy for heart failure should include ACE inhibitors, beta-blockers, diuretics, and an aldosterone antagonist. Digoxin is a second-line drug, but it remains favored as an adjunct by some clinicians. Calcium channel blockers should generally be avoided unless absolutely necessary for rate control in atrial fibrillation.

Correct answer: Increased pulmonary venous pressure Explanation In left-sided congestive heart failure, the predominant feature is low cardiac output and elevated pulmonary venous pressure, resulting in dyspnea. As dyspnea worsens, the patient will also begin to experience shortness of breath at rest, which is worsened in the supine position. Tricuspid function is not related to CHF. Peripheral vascular resistance typically increases in CHF, which is designed to help maintain perfusion to vital organs. Mucus plugging is not associated with CHF.

Correct answer: Bibasilar rales Explanation This patient's presentation is most consistent with congestive heart failure. Manifestations reflect impaired pumping ability of the heart, reduced renal blood flow, and activation of the sympathetic compensatory mechanisms. In chronic heart failure, patients frequently appear malnourished and occasionally cachectic. Due to frequently increased sympathetic activity in patients with heart failure, there may be pallor, diaphoresis, and coldness of the limbs and cyanosis of the digits because of vasoconstriction. Sinus tachycardia is often observed and usually develops in an effort to maintain the cardiac output when heart failure is decompensated or the stroke volume is significantly decreased. Pulmonary signs and symptoms include shortness of breath and dyspnea (which may be exertional), orthopnea, paroxysmal nocturnal dyspnea, a chronic nonproductive cough, wheezing, bibasilar crackles, and dullness to percussion. In heart failure, hepatic congestion and edema of the abdominal wall and intra-abdominal organs may occur. Liver enlargement and tenderness on palpation are marked by epigastric fullness and dullness to percussion in the right upper quadrant. Congestion of abdominal organs may be present with ascites, abdominal fullness and enlargement, early satiety, bloating, anorexia, nausea, vomiting, constipation, and upper abdominal discomfort. Elevations in the jugular venous height may be measurable. Systemic venous hypertension can be detected by abnormal distention of the internal jugular veins. Although the jugular venous pressure normally declines on inspiration, it can rise in patients with right-heart failure (Kussmaul sign). Persistent elevation of the jugular venous pressure is one of the earliest and most reliable signs of right-heart failure. The cardiac exam may reveal a parasternal lift, an enlarged and sustained left ventricular impulse, a diminished first heart sound, S3 gallop, and an S4 with diastolic heart failure. Contributory murmurs may also be evident. Other common symptoms of heart failure include fatigue, exercise intolerance, and generalized weakness, particularly in the limbs. These symptoms are secondary to low cardiac output with decreased perfusion of skeletal muscles and can occur with exertion or at rest; they may be worsened after eating because of the increased splanchnic demand for blood flow, which may stress the limited reserve. Fluid retention results in symmetrical peripheral pitting edema. Nocturia is a common and early symptom in heart failure. Renal filtration of sodium and water is decreased in patients with compromised LV function due to redistribution of blood flow away from the kidneys in the upright position and during physical activity. Urine formation is enhanced in the recumbent position (at night) when renal stimulus for vasoconstriction decreases and venous return to the heart increases. Oliguria is associated with a markedly reduced cardiac output and is usually a sign of terminal heart failure, indicating a poor prognosis.

A 27-year-old woman presents with a 3-day history of "sharp", diffuse chest pain. She states the pain is worse with movement and deep breathing. On examination, it is noted that the patient prefers to sit upright and lean forward; she states, "I feel better in this position". Vital signs include a BP of 126/72 mm Hg; HR is 82, RR is 18, O2 sat is 96% RA, and temp is 101.3? F. On exam, you appreciate a friction rub. Question What set of diagnostics should you order?

Correct answer: CXR, CBC, BMP, BHCG, EKG, and ECHO Explanation The correct response is CXR, CBC, BMP, BHCG, EKG, and ECHO. This patient presents with acute pericarditis. Inflammation of the pericardium can occur as the result of an infectious source (including viruses, bacteria, TB, fungal, and parasitic sources), medications, systemic diseases (such as SLE), post MI (also known as Dressler's syndrome), uremia, or malignancy; there may also be no known cause.1 Pericarditis typically presents with substernal cheat pain that is aggravated by deep breathing and alleviated by leaning forward.1 The hallmark of pericarditis the finding of pericardial friction rub on exam.1 In addition to exam findings, patients with suspected pericarditis should undergo a CXR (may reveal cardiomegaly if a pericardial effusion is present), a CBC (often demonstrates leukocytosis), a BMP (to evaluate for uremia), a BHCG (necessary prior to imaging in a woman of childbearing age), EKG (may demonstrate diffuse ST segment elevation), and an ECHO (needed to evaluate for pericardial effusion and/or tamponade).1 A DDimer is not necessary in this patient; the possibility of a PE being the cause of her symptoms is low. Review of the patient's vital signs reveal a fever with a normal HR and O2 sat, making PE an unlikely cause.

A 77-year-old man presents with significant persistent hypotension that has been worsening for 1 hour. Further investigation of this patient reveals a significant increase in the patient's heart rate as well as substantial tachypnea. Urinary output is too minimal to measure, and upon quick physical examination, the patient's extremities are cool to touch. After beginning fluid resuscitation for shock protocol, the patient's symptoms appear to be worsening. Question Based on this scenario, what type of shock would this patient be categorized as having?

Correct answer: Cardiogenic shock Explanation The correct response is cardiogenic shock. Early identification of shock is a necessity to help improve any patient's chance of survival and recovery. It is also critical to lead to reversing the cause of the shock and initiating early resuscitation efforts. Early on, patients suffering from shock may have only a few or very subtle symptoms such as tachypnea, tachycardia, hyper or hypothermia, weak or bounding peripheral pulses, delayed capillary refill, or even pale or cool skin. Decreased mental status, weak or absent central pulses, central cyanosis, hypotension or bradycardia are ominous and later signs indicating the shock has progressed. Shock is generally classified into 4 major categories, depending on the etiology of the hock: cardiogenic, hypovolemic, distributive, or obstructive. Cardiogenicshock occurs as a consequence of cardiac pump failure. Myopathic, mechanical, or arrhythmic issues lead to cardiogenic shock. Whatever the cause there is typically an acute loss of 15 - 20% of circulating blood volume that leads to lack of blood to pump throughout the circulatory system. The prompt treatment of hypoperfusion and hypotension is essential in the initial management of cardiogenic shock. This is usually initiated by use of vasopressors or inotropic agents. Usually both pharmacologic as well as nonpharmacolgic methods of circulation support are promptly initiated to reverse hypotension, maintain vital organ perfusion, and maintain coronary perfusion pressures as high as possible until intervention of the etiology of the cardiogenic shock can be treated. Administering fluids will worsen the signs and symptoms that a patient is displaying who is has cardiogenic shock.

Case Ico-delete Highlights A 70-year-old man with a history of hyperlipidemia, hypothyroidism, osteoarthritis, and hypertension presents for a routine evaluation. He denies any complaints today and otherwise has no significant past medical history. His physical examination is remarkable for a pulse rate of 44 beats per minute and a blood pressure of 150/94 mm Hg. An EKG assessment reveals sinus bradycardia and a type II Mobitz heart block. Question What antihypertensive should be avoided in the management of this patient?

Correct answer: Metoprolol Explanation Metoprolol is contraindicated in sinus bradycardia, 2nd or 3rd degree AV-block, overt heart failure, or cardiogenic shock. Commonly used medications that cause or contribute to bradycardia do so by enhancing vagal tone (for instance, digitalis), reducing the facilitation of AV conduction that results from sympathetic tone (e.g., -blockers and antiarrhythmia agents with -blocking properties, such as sotalol and propafenone), or direct action on SA and AV conduction tissue (e.g., verapamil and diltiazem). Simple withdrawal of these medications will reverse bradycardia. Fosinopril (ACE-inhibitor), hydrochlorothiazide (diuretic), olmesartan (angiotensin receptor antagonist), and aliskiren (direct rennin inhibitor) will not have inhibitory effects at the AV-node and are therefore acceptable considerations in managing this patient's hypertension, given his bradycardia and heart block.

A 77-year-old man presents with significant persistent hypotension that has been steadily worsening for 1 hour. The patient's history reveals he was originally admitted with the primary diagnosis of a myocardial infarction. Further investigation of this patient reveals a significant increase in the patient's heart rate as well as substantial tachypnea. Urinary output is too minimal to measure, and upon quick physical examination, the patient's extremities are cool to touch. A Swan-Ganz catheter is in place, and it offers the following information: Cardiac Output ↓, Central Venous Pressure ↑, Pulmonary Artery Obstructive Pressure ↑, as well as the Systemic Vascular Resistance being ↑. Question Based on this scenario, what type of shock would this patient be categorized as having?

Correct answer: Cardiogenic shock Explanation The correct response is cardiogenic shock. Early identification of shock is a necessity to help improve any patient's chance of survival and recovery. It is also critical to lead to reversing the cause of the shock and initiating early resuscitation efforts. Early on, patients suffering from shock may have only a few or very subtle symptoms such as tachypnea, tachycardia, hyper or hypothermia, weak or bounding peripheral pulses, delayed capillary refill, or even pale or cool skin. Decreased mental status, weak or absent central pulses, central cyanosis, hypotension, and/or bradycardia are ominous and later signs that indicate that the shock has progressed. Shock is generally classified into 4 major categories, depending on the etiology causing it: cardiogenic, hypovolemic, distributive, or obstructive. Cardiogenicshock occurs as a consequence of cardiac pump failure. Myopathic, mechanical, or arrhythmic issues lead to cardiogenic shock. A patient with cardiogenic shock will display increased heart rate and systemic vascular resistance; there will also be decreased cardiac output, pulse pressure, and mixed venous oxygen saturation. Distributive shock results in increased heart rate, cardiac output, pulse pressure, and mixed venous oxygen saturation. When looking at the classification of shock, septic, anaphylactic, and neurogenic shock are all categorized as a type of distributive shock, making these incorrect responses.

A 58-year-old woman presents with a 3-month history of postprandial abdominal pain. This crampy pain occurs 30 minutes after eating, every time. Due to these symptoms, the patient has lost 30 pounds and is afraid to eat. Her past medical history includes hypertension treated with enalapril, coronary artery disease for which she has undergone a right coronary artery stent, and she underwent a carotid endarterectomy for symptomatic carotid stenosis. She has smoked 2 packs of cigarettes a day for 30 years. Question What is the best initial test for this patient?

Correct answer: Computerized tomography angiography (CTA) Explanation The current American College of Radiology recommends Computerized tomography angiography (CTA) with IV contrast as the initial imaging study for both acute and chronic mesenteric ischemia, noting it is a "Fast noninvasive study that also evaluates other causes of abdominal pain." Mesenteric duplex ultrasound is a good screening test to detect chronic mesenteric ischemia. Combined B-mode and color Doppler ultrasound analyze flow through the mesenteric arteries and identifies stenosis as an elevated velocity. Limiting factors include obesity and intestinal gas, which may obstruct the ability to obtain a good ultrasound image. Mesenteric angiography was previously the "gold standard" investigation of choice for the diagnosis of arterial mesenteric ischemia. Due to the risks associated with this invasive procedure, CT angiography is often substituted for angiography. The role of CT angiography and MRA remains undefined when diagnosing chronic mesenteric ischemia and has not yet met their potential as replacements for mesenteric angiography.

A 58-year-old man presents with worsening shortness of breath; it has been especially problematic over the last 4 months. He states that the SOB has been noticeably severe with exertion, and this has drastically decreased the patient's ability to do any physical activities. He is now experiencing SOB at rest. Physical examination reveals mild abdominal distension secondary to ascites as well as 4+ bilateral peripheral lower extremity edema. Past medical history includes that the patient underwent radiation and chemotherapy for lung cancer about 7 years ago. Question Given the likely diagnosis, what is one of the main goals when utilizing pharmacologic agents for this patient?

Correct answer: Decrease pulmonary congestion Explanation The patient above is most likely experiencing an acute exacerbation of restrictive cardiomyopathy. Restrictive cardiomyopathy is more likely in this patient because any patients with a history of radiation and/or chemotherapy for past malignancies have a higher incidence of developing this unfortunate consequence. Restrictive cardiomyopathy, just as the name implies, is a disorder of the heart muscles where the ventricular walls stiffen and restrict the heart's' ability to fill with blood, therefore restricting the overall pumping action of the heart. Since the systolic function is impaired, this can lead to a backup of blood in the lungs and veins, which will eventually backup even more peripherally such as the neck veins and even in the liver. These patients are also at a much higher risk of developing heart failure. For this reason, any pharmacologic agents that will decrease pulmonary congestion must be utilized to decrease or stop this backup.

A 54-year-old man presents with a 30-minute history of oppressive retrosternal chest pain that radiates to the left arm. The physical examination reveals a BP of 170/100 mm Hg, and; a pulse of 90/min; RR is 24/min, with normal cardiac and lung auscultation. The initial EKG shows an elevation of ST segment in leads V3 and V4. What artery is most likely compromised in this patient?

Correct answer: Diagonal branch of left anterior descending artery Explanation The correct response is the diagonal branch of left anterior descending artery. The patient's history is compatible with an acute myocardial infarction. The EKG shows an elevation of ST segment in leads V3 and V4, which reflect the anterior wall of the left ventricle. The diagonal branch of the left anterior descending artery-LAD (a branch of left coronary artery) supplies this anatomic area. The LAD-septal branch supplies the septum, represented by leads V1 and V2. The LCA-circumflex branch supplies the high lateral wall of left ventricle, represented by V5, V6, DI, and aVL. The RCA (right coronary artery) posterior descending branch supplies the inferior and posterior wall of left ventricle, represented by leads DII, DIII, and aVF. The RCA-proximal branch supplies the right ventricle, inferior, and posterior wall of left ventricle, represented by leads V4R, DII, DIII, and aVF.

A 52-year-old woman presents for a routine checkup. She has 2 children, and she attained menopause 1 year prior to presentation. Pap smears, mammogram, and DEXA bone scan are normal. She is a non-smoker. Her previous biennial checkups were always normal. Her BP is 142/96 mm Hg, and pulse is 72 bpm. Her lab values are as follows: Fasting blood sugar: 112 Postprandial blood sugar: 138 Total cholesterol: 190 LDL: 102 TSH: normal levels Question What is the next best step in the management of this patient?

Correct answer: Diet and exercise Explanation The correct response is diet and exercise. The patient has an elevated blood pressure and borderline blood sugar levels. Since all previous checkups have been normal, initiation of therapy at this stage is unwarranted. Reassurance is incorrect. Both blood sugar and blood pressure are at borderline levels; the patient needs to be educated about the probability of developing hypertension and diabetes. An acceptable and recommended initial step is the modification of diet with regular aerobic exercise.

A 74-year-old Caucasian woman with a history of hypertension, hyperlipidemia, and myocardial infarction presents with shortness of breath upon exertion, lightheadedness, increased fatigue, and palpitations for 3 days. She has found it difficult to walk short distances due to shortness of breath. She denies any orthopnea, nocturnal dyspnea, abdominal pain cough, fever, chills, diaphoresis, anxiety, chest pain, pleurisy, cough, nausea, vomiting, diarrhea, rashes, or syncope. She is well nourished and afebrile, but tachypnic. Her cardiac exam reveals an irregularly irregular pulse, hypotension, diminished first heart sound, S3 gallop, and laterally displaced PMI. Her lung, peripheral vascular, and abdominal exams are normal. An echocardiogram is remarkable for a left ventricular ejection fraction of 25% while an electrocardiogram displayed the included results. Question What agent would be most appropriate in the management of this patient at this time?

Correct answer: Digoxin Explanation The correct response is digoxin. The other responses are incorrect. This patient is presenting with signs and symptoms consistent with congestive heart failure. The echocardiogram confirms a reduced left-ventricular ejection fraction, while the electrocardiogram identifies atrial fibrillation. Overall, digoxin use usually is limited to CHF patients with left-ventricular systolic dysfunction in atrial fibrillation or to patients in sinus rhythm who remain symptomatic despite maximal therapy with ACE inhibitors and β-adrenergic receptor antagonists. Although digoxin does not confer a survival benefit, it has reduced the number of hospitalizations that occur as a result of worsening heart failure.

How do you best describe dilated cardiomyopathy?

Correct answer: Dilatation and impaired contraction of one or both ventricles. Explanation The description "dilatation and impaired contraction of one or both ventricles" explains dilated cardiomyopathy. "Left ventricular volume is either normal or reduced and diastolic dysfunction is almost always present" explains hypertrophic cardiomyopathy. "Nondilated ventricles with impaired ventricular filling" explains restrictive cardiomyopathy. "Transient systolic dysfunction of the apical and/or mid segments of the left ventricle that is often provoked by stress" explains Takotsubo syndrome. "Diffuse thickening of the left ventricular endocardium secondary to proliferation of fibrous and elastic tissue" explains endocardial fibroelastosis.

A 52-year-old Caucasian man who is a frequent visitor to the emergency department presents due to being "being short of breath". The patient is currently known to be homeless and is frequently in and out of shelters. Past chief complaints that have brought him to the emergency department indicate that he has a chronic issue with alcohol overuse. Today, the patient is experiencing severe shortness of breath at rest. Physical examination findings reveal rales auscultated in bilateral lung fields, an S3 gallop, and elevated JVP. Question Based on the history and physical exam findings presented, what is the most likely diagnosis in this patient?

Correct answer: Dilated cardiomyopathy Explanation The most likely explanation for the patient's history and physical examination findings is the diagnosis of dilated cardiomyopathy. Many times cardiomyopathies will present themselves as worsening heart failure signs and symptoms. Dilated cardiomyopathy may be idiopathic; common causes include alcohol overuse, myocarditis, or even genetic disease states. Symptoms, such as worsening dyspnea with activity and eventually at rest, paroxysmal nocturnal dyspnea, cough, fatigue, and even swelling extremities may by experienced. Signs found on the physical examination include rales heard during auscultation, elevated jugular venous pressure, cardiomegaly, S3 gallop rhythm, murmurs of mitral or tricuspid regurgitation, peripheral edema, or even ascites. A key feature to distinguish this as dilated cardiomyopathy would be left ventricular dilatation and dysfunction. This is not restricted cardiomyopathy due to the fact that right heart failure usually predominates over left heart failure in this pathology. Significant symptoms and signs of pulmonary hypertension will also be evident. Dyspnea and fatigue will be present and only slight cardiomegaly. Restricted cardiomyopathy also may have only slight or even normal left ventricular function and size. Causes include amyloidosis, post-radiation, diabetes, or post-open heart surgery.

In order to test for orthostatic changes, blood pressure and pulse are measured with the patient first supine then standing. What are the criteria for a positive orthostatic change (going from supine to standing)?

Correct answer: Drop in systolic pressure 20 mm Hg, drop in diastolic pressure of 10 mm Hg, increase in pulse of 20 beats per minute

A 3-month-old male infant has been brought into the pediatric clinic for assessment. The newborn's mother states that her child is not gaining adequate weight despite a regular breast feeding schedule. She additionally has noted that the child appears to get "very tired and inactive" during and after breast feedings, and that she is able to feel copious amounts of sweat on the child's skin following feedings. She denies any known illness in her child and recalls a normal birth. The general survey reveals a weight and length in the fortieth percentile, tachycardia, and tachypnea. The cardiac exam is remarkable for a bounding and hyperdynamic precordium, a holosystolic harsh murmur that is audible over the lower LSB, and a loud second component of the second heart sound. Question What is the best next diagnostic step for this child?

Correct answer: Echocardiogram Chest radiography, magnetic resonance imaging (MRI), and electrocardiography (ECG) may all provide useful information in the workup of a ventricular septal defect (VSD). Detailed echocardiography is now preferred in most institutions. Echocardiography provides the information required for potential surgical closure of a VSD.

Case Your patient is a 78-year-old woman who is an inpatient status post-colectomy for colon cancer. On post-operative day 3, her oral temperature is noted to be elevated to 100.6° F. Chest x-ray and urinalysis are both negative for signs of infection. An infectious disease consult is placed in order to better define the patient's new fever. Question What physical examination findings would support a diagnosis of superficial thrombophlebitis?

Correct answer: Erythema and tenderness along the vein with IV insertion Explanation Superficial thrombophlebitis is an inflammation of a vein and can be related to a septic condition. One risk factor for superficial thrombophlebitis is presence of an IV catheter, which this patient would have due to being an inpatient and post-operative. 65-78% of cases are caused by Staphylococcus aureus, and about 88 out of 100,000 patients experience a superficial thrombophlebitis while admitted. The physical examination would reveal tenderness, erythema, and edema of the vein where the IV is present. 84% of patients would also have signs of systemic sepsis, including fever in about 70% of cases. A patient with superficial thrombophlebitis would not have a positive Homans Sign. A positive Homans Sign is defined as pain in the calf with dorsiflexion of the foot while the patient's knee is slightly bent. Although this test only has about 50% sensitivity, when present it is indicative of a deep vein thrombosis. Erythema and tenderness at the patient's incision site would not indicate superficial thrombophlebitis, as this is not a site where this would occur. Instead, a patient with erythema and tenderness of any incisional site post-operatively could have an infection.

Question An 8-year-old girl, who weighs 70 lbs and is 52 inches tall, has been diagnosed with stage I HTN. Physical examination and lab studies are normal. Which additional test is most likely to detect target-organ damage abnormalities in this child?

Correct answer: Echocardiogram Explanation Left ventricular hypertrophy (LVH) best demonstrates target-organ damage due to hypertension. Echocardiogram should be done at the time of diagnosis and periodically thereafter. Echocardiogram assesses left ventricular mass. LVH has been found in 34-38% of children with untreated mild hypertension. Concentric hypertrophy is associated with a higher risk of adverse cardiovascular outcomes in adults. Eccentric hypertrophy is associated with an intermediate risk for adverse cardiovascular outcomes. Heart size is correlated to body size; left ventricular mass index is usually correlated with height (meters squared), although some laboratories use height alone as the index variable. The established cutoff point for children and adolescents at the 99th percentile is 51 grams/meter 2. Outcome standards based on left ventricular mass index are not available for children. LVH due to obesity and hypertension is maladaptive whereas LVH due to physical conditioning may be adaptive. The presence of LVH in the presence of hypertension indicates that anti- hypertensive therapy should be started or, if already started, intensified. Secondary hypertension is more common in children than adults. Secondary hypertension accounts for 70-85% of pre-adolescent hypertension and 5-15% of adolescent hypertension1. In pre-adolescents, causes are renal (60-70%), coarctation of the aorta (10-20%), renovascular (5-10%), reflux nephropathy (5-10%), endocrine (3-5%), tumors (1-5%), and other (1-5%). In adolescents, the proportions are the same for causes of secondary hypertension1. In children with hypertension, additional evaluation may be indicated. This is particularly applicable to very young children, children with stage 2 hypertension, and children or adolescents with clinical findings suggesting another cause. A thorough history and physical examination is the first step. Blood pressure should be checked in arm and leg. Renal disease is assessed with Creatinine, blood urea nitrogen, urinalysis, urine culture, and renal ultrasound. Plasma renin levels, if low, suggest mineralocorticoid-related disease. Plasma renin levels are higher in patients with renal artery stenosis. Approximately 15 % of children with renal artery stenosis, however, have normal plasma renin levels. Renovascular imaging should be done in young children with stage 1 hypertension and all children and adolescents with stage 2 hypertension. Different tests are used for imaging including renal scan or isotope scintigraphy, duplex Doppler flow studies, 3-dimensional CT scan, arteriography, and magnetic resonance angiography (MRA).

A 43-year-old man presents for a general physical exam. He states that he has no significant past medical history. On physical examination, you note that the patient has a 3/6 diastolic murmur; it is heard best in the right upper chest. Blood pressure is 152/62 mm/Hg. No other abnormalities are noted. Question What study is best in regard to getting an accurate diagnosis for this patient?

Correct answer: Echocardiogram with color Doppler Explanation This patient most likely has aortic regurgitation. A murmur that is best heard in the aortic location and the wide pulse pressure found on blood pressure assessment both point toward aortic regurgitation. The very best study to evaluate heart valves is an echocardiogram, which will give a picture of the valve and the blood flow through it when a Doppler is also utilized. This study can also evaluate the size of the left ventricle.

An 82-year-old man with a past medical history of hypertension, dyslipidemia, type II diabetes, and chronic kidney disease is being evaluated for progressive exercise-induced fatigue and shortness of breath over the last year. He also admits to more recent chest pain and lightheadedness, both of which occur with ambulation. He denies cough, fever, chills, lower extremity edema, or abdominal complaints. The physical exam revealed a narrow pulse pressure following blood pressure assessment. His cardiac exam noted a laterally displaced point of maximal impulse, as well as a mid-systolic ejection murmur that is low-pitched, rough, rasping in character, and loudest in the second right intercostal space. This murmur radiates to the bilateral carotid arteries. His peripheral vascular exam demonstrated a delayed peak of his radial pulsations. Question Ico-delete Highlights What diagnostic procedure would be best for this patient?

Correct answer: Echocardiography Explanation This patient's presentation is consistent with aortic stenosis (AS). In most patients with severe AS, there is left ventricular hypertrophy. The key findings on transthoracic echocardiography are thickening, calcification, and reduced systolic opening of the valve leaflets, and left ventricular hypertrophy. Echocardiograms are also useful in identifying coexisting valvular abnormalities, differentiating valvular aortic stenosis from other forms of left ventricular outflow obstruction, and for measurement of the aortic root and proximal ascending aortic dimension.

A 30-year-old woman with no significant past medical history presents to the emergency room with severe, retrosternal, and left precordial chest pain; it has occurred over the past 3 days. Pain is sharp in quality; it is referred to the neck, arms and left shoulder. It is pleuritic. It is aggravated by inspiration and coughing as well as lying in a supine position. Her review of systems is remarkable for a low grade fever, myalgias, and arthralgias. It is relieved by sitting up and leaning forward. Upon physical exam, her vital signs are normal, and her skin is warm and dry. Auscultation of the chest wall with the diaphragm at the left lower sternal border reveals a high-pitched, rasping, scratching, and grating sound, most audible at the end of expiration. A bedside electrocardiogram reveals the following imaging. Question What is an additional expected diagnostic finding in this patient?

Correct answer: Elevated ESR and CRP levels Explanation The correct response is elevated ESR and CRP levels. This patient is demonstrating manifestations of acute pericarditis, which is most likely of a viral etiology. Viral or idiopathic acute pericarditis occurs at all ages, but it is more common in young adults and is often associated with pleural effusions and pneumonitis. The almost simultaneous development of fever and precordial pain, often occuring 10 to 12 days after a presumed viral illness, constitutes an important feature in the differentiation of acute pericarditis from AMI, in which chest pain precedes fever. Acute pericarditis occurs in association with illnesses of known or presumed viral origin, and it is probably caused by the same agent. Commonly, there is an antecedent infection of the respiratory tract, and viral isolation and serologic studies are negative. Coxsackievirus A or B, influenza, echovirus, mumps, herpes simplex, chickenpox, adenovirus, cytomegalovirus, Epstein-Barr, and HIV are all viruses capable of causing viral pericarditis.

A 74-year-old man with a past medical history of diabetes mellitus, hypertension, and hyperlipidemia presents with severe chest pain and dyspnea. On examination, he is confused, agitated, pale, apprehensive, and diaphoretic. His pulse is weak and tachycardic, with a systolic blood pressure of 80 mmHg. He has a narrow pulse pressure, tachypnea, a weak apical impulse, significant jugular venous distention, and pulmonary crackles. Bedside electrocardiogram reveals ST-segment elevations in the anterior and septal leads, while a portable chest X-ray notes diffuse pulmonary congestion. Question What is the most appropriate step in the management of this patient?

Correct answer: Emergent percutaneous coronary intervention Explanation This patient's exhibits signs and symptoms of cardiogenic shock due to myocardial infarction with pulmonary edema.

A 3-year-old child suddenly dies while playing. According to the parents, the child did not have any serious illnesses. 8 weeks prior to death, the child suffered from a fever accompanied by skin rashes. The child's symptoms subsided after treatment. The autopsy reveals a massive aneurysmal dilatation of the long length of the proximal coronary arteries with an occluding thrombus. Histologically, the arteries show an intense transmural arteritis. The arteries in the other organs are normal. Question What is the most likely diagnosis?

Correct answer: Kawasaki disease Explanation Kawasaki disease is an acute arteritis involving large, medium-sized, and small arteries (often the coronary arteries), and it is associated with mucocutaneous lymph node syndrome. The mucocutaneous syndrome is characterized by fever; conjunctival and oral erythema; edema of the hands and feet; erythema of the palms and soles; a skin rash (often with desquamation); and enlargement of the cervical lymph nodes. It is usually self-limited. Approximately 20% develop cardiovascular sequelae with the range of severity from asymptomatic vasculitis of the coronary arteries, coronary artery ectasia, or aneurysm to giant coronary artery aneurysm (7 to 8 mm) with rupture or thrombosis, myocardial infarction, or sudden death. Acute fatalities occur in 1% of the cases due to coronary artery thrombosis or rupture of coronary artery aneurysm. Vasculitis resembles polyarteritis nodosa with necrosis and pronounced inflammation, affecting the entire thickness of the vessel wall. Takayasu's arteritis is a granulomatous vasculitis of medium-sized and larger arteries. Takayasu first described it in 1908 as a clinical syndrome characterized by ocular disturbances and marked weakening of the pulses in the upper extremities (Pulseless disease). This is related to the fibrous thickening of the aortic arch with narrowing, or virtual obliteration, of the origins or more distal portions of the great vessels arising in the arch. Microscopically, there is adventitial mononuclear infiltrate with perivascular cuffing of the vasa vasorum (initially). Later, there is intense mononuclear inflammation in the media, in some cases accompanied by granulomatous changes replete with giant cells and patchy necrosis of the media. In addition, when it heals, the inflammation is replaced by marked collagenous fibrosis involving all the layers of the vessel wall accompanied by lymphocytic infiltration. Giant cell arteritis, or temporal arteritis, is the most common of the vasculitides. It is an acute, and chronic, granulomatous inflammation of the medium and small arteries. It affects mainly the temporal, vertebral, and ophthalmic arteries. The short segments of 1 or more arteries develop nodular thickening with the reduction of the lumen, which may become thrombosed. Histologically, there is granulomatous inflammation of the inner half of the media, centered on the internal elastic membrane. It is marked by mononuclear infiltrate, multinucleated giant cells of both foreign body and Langhans type, and fragmentation of the internal elastic lamina. Sometimes the granulomas may be absent, and there is only nonspecific polyarteritis without giant cells. The healed stage of both will reveal only collagenous thickening of the vessel wall. Polyarteritis nodosa is systemic vasculitis characterized by necrotizing inflammation of the small or medium-sized vessels, typically involving renal arteries and visceral vessels, but sparing the pulmonary circulation. There is neither glomerulonephritis nor vasculitis of the arterioles, capillaries, and venules. It particularly involves the branching points, also known as the points of bifurcation. The involvement is segmental and may involve only a portion of the circumference. It causes segmental erosion with weakening of the arterial wall, with aneurysmal dilatation or localized rupture. Initially, there is transmural inflammation of the vessel consisting of neutrophils, eosinophils, and mononuclear infiltrate with fibrinoid necrosis. Later, the inflammation is replaced by collagenous fibrosis. Within the same vessel, various stages of inflammation may be seen. Microscopic polyangiitis is also called hypersensitivity, or leukocytoclastic vasculitis. It generally affects arterioles, capillaries, and venules. The lesions are thought to represent a hypersensitivity reaction that involves the skin, mucous membranes, lungs, brain, heart, gastrointestinal tract, kidneys, and muscle. Necrotizing glomerulonephritis and pulmonary capillaritis are common. The lesions are histologically similar to polyarteritis nodosa, but muscular and large arteries are spared. Histologically, segmental fibrinoid necrosis of the media may be present; however, in some, the change is limited to infiltration with neutrophils. Greater than 80% of the patients have ANCA (anti-neutrophil cytoplasmic antibodies), most often p-ANCA. In many cases, reaction to an antigen such as drugs (e.g. penicillin), microorganisms (e.g. streptococci), heterologous proteins, or tumor antigens can be traced as the precipitating cause, but there are few or no immune deposits in this type of vasculitis. Simple removal of the offending agent may help most patients with cutaneous vasculitis, but those with systemic disease may develop organ failure unless treated. Refer to the table for additional information.

Case Ico-delete Highlights An elderly patient is admitted with an acutely severe myocardial infarction and quickly develops significant signs and symptoms of cardiogenic shock. These include hypotension, altered mental status, cold clammy skin, as well as metabolic acidosis seen on laboratory tests. Question What is the most appropriate initial pharmaceutical choice for a patient suffering from this type of shock?

Correct answer: Vasopressors Explanation The correct response is vasopressors. Early identification of shock is necessary to help improve any patient's chance of survival and recovery. It is also critical in regard to reversing the cause of the shock and initiating early resuscitation efforts. Early on, patients suffering from shock may have only a few or very subtle symptoms, such as tachypnea, tachycardia, hyper or hypothermia, weak or bounding peripheral pulses, delayed capillary refill, and/or pale or cool skin. Decreased mental status, weak or absent central pulses, central cyanosis, hypotension, or bradycardia are ominous and indicate the shock has progressed. Shock is generally classified into 4 major categories, depending on the etiology causing it: cardiogenic, hypovolemic, distributive, or obstructive. Cardiogenic shock occurs as a consequence of cardiac pump failure. Myopathic, mechanical, or arrhythmic issues lead to cardiogenic shock. Whatever the cause there is typically an acute loss of 15 - 20% of circulating blood volume that leads to lack of blood to pump throughout the circulatory system. The prompt treatment of hypoperfusion and hypotension is essential in the initial management of cardiogenic shock. This is usually initiated by use of vasopressors or inotropic agents. Usually, both pharmacologic as well as nonpharmacolgic methods of circulation support are promptly initiated to reverse hypotension, maintain vital organ perfusion, and maintain coronary perfusion pressures as high as possible until intervention of the etiology of the cardiogenic shock can be treated.

A 35-year-old woman presents with a 5-day history of nervousness and palpitations. She has had diarrhea for most of the previous month and has had a 3 kg weight loss despite an increased appetite. She also reports increased fatigue and sweating. Her vital signs are an irregular pulse of 114 bpm, blood pressure of 125/75 mm Hg, respirations of 18/min, and a temperature of 37.8° C. Physical exam reveals exophthalmos, a mass in the midline of her neck that moves with deglutition, fine resting tremor, and hyperactive reflexes. Her thyroid stimulating hormone (TSH) level is low. An ECG is performed, revealing atrial fibrillation with a ventricular response ranging between 110 and 130/min. Question What would be the most appropriate initial intervention regarding her arrhythmia?

Correct answer: Esmolol Explanation Atrial fibrillation is the most common chronic arrhythmia, frequently developing in the course of hyperthyroidism. The initial steps in the management of atrial fibrillation is to control the ventricular response and prevent the risk of stroke. A short acting beta-blocker (e.g., esmolol) is a good choice for the rapid management of heart rate. Other drugs used in the management of atrial fibrillation include: Calcium channel blockers (e.g., verapamil or diltiazem) Digoxin when calcium channel blockers and beta-blockers are not suitable Once rate control has been achieved, the mainstay of treatment for patients with atrial fibrillation and hyperthyroidism involves restoration of euthyroid status by the use of antithyroid drugs. Restoration of euthyroid status is frequently associated with conversion to sinus rhythm. Electric cardioversion should not be performed prior to anticoagulation in patients with atrial fibrillation that has been present for more than 48 hours. In addition, the maintenance of sinus rhythm after cardioversion is unlikely until the patient is restored to a euthyroid state.

A 1-month-old infant is being evaluated for rapid breathing, feeding difficulty, lethargy, and poor weight gain. The physical exam is notable tachypnea, tachycardia, a cardiac gallop, and a medium-pitched systolic murmur, which is best heard posteriorly in the interscapular area with radiation to the left axilla, apex, and anterior precordium. A prominent anterior chest heave is also observed. The lower extremities demonstrate a 12 mmHg pressure difference as compared to the upper extremities. Additionally, there are delayed femoral pulsations; his upper extremity pulsations are normal. Question What is correct regarding the long-term management of this patient?

Correct answer: Evaluation by a cardiovascular surgeon is essential for definitive treatment. Explanation Evaluation by a cardiovascular surgeon is essential for definitive treatment. This patient's manifestations suggest a diagnosis of coarctation of the aorta.

Case Ico-delete Highlights A 66-year-old man has a past medical history of amyloidosis, hyperlipidemia, and hypereosinophilic syndrome; he presents with a 1-year history of progressive dyspnea upon exertion. Currently, he is only able to walk 3 blocks before having to stop to catch his breath. Additionally, he has noticed increased lower extremity swelling, abdominal "bloating", and a loss of appetite. Despite not eating as much, he has noticed a 15-pound weight gain over the past 6 weeks. His physical exam reveals bipedal pedal edema, increased jugular venous pressure (with Kussmaul's sign noted), abdominal distension with shifting dullness, and hepatosplenomegaly. The precordium is without any heaves, lifts, or thrills. An EKG demonstrates a normal sinus rhythm without abnormalities. An echocardiogram is performed; it reveals the attached image. Question What pharmacotherapeutic agent would be most beneficial to this patient at this time

Correct answer: Furosemide (Lasix) Explanation The correct response is furosemide. This patient's most likely diagnosis is restrictive cardiomyopathy. The goal of treatment in this form of cardiomyopathy is to reduce symptoms by lowering elevated filling pressures without significantly reducing cardiac output. Low to medium dose diuretics, such as furosemide (Lasix), lower preload and may provide symptomatic relief. Small initial doses should be administered to prevent hypotension; patients are frequently extremely sensitive to alterations in left ventricular volume. Higher doses may be needed if the serum albumin level is low secondary to concomitant nephrotic syndrome. In addition, beta blockers and cardioselective calcium channel blockers, such as verapamil and diltiazem, may be of benefit; they increase left ventricular filling time, improve ventricular relaxation, and decrease compensatory sympathetic stimulation. ACEIs and angiotensin II inhibitors are poorly tolerated in patients with amyloidosis. Even small doses may result in profound hypotension. Digoxin should be used with caution, since it is potentially arrhythmogenic, particularly in patients with amyloidosis; other medications such as amiodarone should be considered. While this patient is not displaying evidence of atrial fibrillation, those with atrial fibrillation should be anticoagulated. In patients with atrial fibrillation, the rate should also be controlled adequately; this is typically accomplished with amiodarone and beta-blockers. No published data are available on the use of intravenous (IV) inotropic or vasodilator drugs such as Apresoline (hydralazine)

A 55-year-old man presents for re-evaluation of his blood pressure; he has no significant past medical history. At his visit 3 weeks ago, his blood pressure was 145/90 mm Hg. He admits to somnolence, confusion, and nonspecific bilateral visual disturbances over the past month; he denies any eye pain, blindness, ocular discharge, or floaters. His blood pressure today is 185/110 mm Hg. His fundoscopic exam reveals the following image. Question What is an additional expected manifestation in this case?

Correct answer: Headache Explanation Mild-to-moderate primary (essential) hypertension is largely asymptomatic for many years. The most frequent symptom, which is a headache, is also very nonspecific. Accelerated hypertension is associated with somnolence, confusion, visual disturbances, and nausea and vomiting; they are common to hypertensive encephalopathy. Hypertensive retinopathy may cause papilledema, intraretinal hemorrhages, nerve fiber layer infarcts (cotton-wool spots), arteriovenous (AV) nicking, banking or beading, or copper or silver wiring of the retinal vessels.

A 33-year-old man presents for an initial visit at a new primary care office. He has not seen a health care provider at all in the past 5 years. His past medical history reveals a coarctation of the aorta repair at age 13, after which he saw a cardiologist yearly until age 18. Since then, he has not had insurance and has only sought care for urgent problems in acute care clinics. Question What is a common complication of coarctation for which primary care should regularly monitor this patient?

Correct answer: Hypertension Explanation Approximately 25% of surgically corrected coarctation of the aorta patients have persistent or recurrent hypertension years after surgery because of changes in the renin-angiotensin system. They often have disproportionate systolic hypertension with exercise. These patients are also at risk for recoarctation, aortic aneurism (especially near the repair site), and left ventricular hypertrophy. Coarctation patients need to have their blood pressure monitored regularly, and if they develop hypertension, it should be aggressively controlled. It is also best to have these patients see a cardiologist as well.

A 15-year-old girl is referred to a cardiologist's office for workup of hypertension. Her mother reports a normal pregnancy and birth. There is no family history of heart disease. On physical exam you note the following: BP 140/70 left and right upper extremities, 90/70 left and right lower extremities, HR 85/min, RR 20/min. Brachial and femoral pulses are incongruent. You note pulsations in the suprasternal notch. Cardiac auscultation reveals a III/VI systolic ejection murmur. What would you expect to see on chest radiography?

Correct answer: Left ventricular hypertrophy and a notch in the aorta Explanation Left ventricular hypertrophy (LVH) and a notch in the aorta are correct because this is what is seen in patients who have coarctation of the aorta. The notch in the aorta signifies the coarctation and the LVH is the result of hypertension and aortic stenosis. Common physical exam findings in patients with the disorder include hypertension, higher blood pressures in the upper extremities than in the lower extremities, a delay between the brachial and femoral pulses, and a systolic ejection murmur as a result of a bicuspid aortic valve (1). The patient above exhibits all of these exam findings. Right ventricular hypertrophy (RVH) and large pulmonary arteries is incorrect as this is seen with atrial septal defect (1).

A 25-year-old woman had an episode of angina. On physical exam you notice a systolic heart murmur that is best heard at the 2nd right intercostal space. Valsalva, standing, and handgrip maneuvers decrease the intensity of the murmur. A transesophageal echocardiogram reveals a bicuspid aortic valve among other findings. Question What adaptive response is most likely to be present in the left ventricular wall muscle of this patient's heart?

Correct answer: Hypertrophy Explanation The clinical and echocardiographic findings are consistent with aortic stenosis. Left ventricular hypertrophy is most likely present. Hypertrophy is the increase in cell size and function capacity commonly observed in cardiac muscle cells. In this case, the congenital anomaly in the aortic valve creates an obstacle (aortic stenosis) to the blood flux out of the left ventricle. This leads to the systolic murmur and an increase in pressure inside the ventricle. The increased chamber pressure acts chronically on the cardiac muscle cells, leading to the adaptive response of hypertrophy. Macroscopically, the result is an increase in the thickness of the left ventricular myocardial walls. Cardiac and skeletal muscle cells are unable to replicate after the embryogenesis period; therefore, they are classified as permanent cells. When they face a stimulus that signals to proliferation, hypertrophy is the only adaptive response that can occur. Hyperplasia is an increase in the number of cells; it occurs in tissues that are able to replicate when facing a stimulus that signals to proliferation (labile and stable tissues). For example, skin keratinocytes are labile cells and respond with hyperplasia when facing an irritative stimulus. The smooth muscle cells of the uterine walls are stable cells and respond with hypertrophy and hyperplasia when facing proliferative hormonal stimulus during pregnancy. Fibrosis is not an adaptive response; it is the proliferation of fibroblasts and the deposit of collagen in a reparative/reactive process following cell death and/or inflammation (e.g., myocardial infarction). While angina is a key symptom of myocardial infarction, the age of this patient (25) makes this a remote possibility. Metaplasia is the reversible change of 1 cell type to another, usually in response to irritation (e.g., bronchioloalveolar squamous metaplasia in smokers). It is uncommon in the heart. Atrophy results in decreased muscular mass. Atrophy is an adaptive cellular response defined by loss of cells/cell substance leading to a decrease in organ size and function ability. Causes include decreased workload, restriction of nutrients, ischemia, and aging.

A 42-year-old man has had systolic blood pressure in the 140's and diastolic blood pressure in the 80's on several occasions despite changing his diet and exercise regimen. His physician decides to start him on hydrochlorothiazide. Question What electrolyte abnormality may be associated with hydrochlorothiazide?

Correct answer: Hypokalemia Explanation Hydrochlorothiazide is a thiazide diuretic. It is associated with hypokalemia, hypomagnesemia, hypercalcemia, and hyponatremia. Thiazide diuretics are less likely to cause electrolyte abnormalities than loop diuretics, such as lasix; however, patients should still be monitored for their development.

A 62-year-old man has a 15-year history of hypertension, hyperlipidemia, myocardial infarction, and congestive heart failure. He presents with a 10-day history of shortness of breath. He finds it difficult to walk short distances due to shortness of breath. He also notes cough, orthopnea, nocturnal dyspnea, and generalized abdominal discomfort. He has been taking large doses of his furosemide without relief. He denies fever, chills, diaphoresis, anxiety, chest pain, pleurisy, nausea, vomiting, diarrhea, rashes, lightheadedness, and syncope. On physical examination, the patient is acutely dyspneic at rest. He is afebrile, but tachypnic and diaphoretic. There is a diminished first heart sound, S3 gallop, laterally displaced PMI; the lungs have bibasilar rales. The abdominal exam reveals distension with hepatomegaly in the right upper quadrant. There is 2+ pitting edema of the lower extremities to the level of the mid-calf. A bedside chest X-ray reveals the following image. Question What is the most likely expected diagnostic test result in this case?

Correct answer: Hyponatremia Explanation This patient is experiencing an acute exacerbation of congestive heart failure. In cases of severe heart failure, prolonged, rigid sodium restriction, coupled with intensive diuretic therapy and the inability to excrete water, may lead to dilutional hyponatremia. This occurs because of a substantial expansion of extracellular and intravascular fluid volume and a normal or increased level of total body sodium. In patients with long-standing heart failure, albumin synthesis may be impaired, leading to hypoalbuminemia and intensifying the accumulation of fluid. Patients with severe heart failure, particularly those on large doses of diuretics for long periods, may have elevated BUN and creatinine levels indicative of renal insufficiency because of chronic reductions of renal blood flow from reduced cardiac output. Hypochloremia, not hyperchloremia, is expected in edematous states such as congestive heart failure. Extended use of kaliuretic diuretics can lead to hypokalemia, and the use of potassium-sparing diuretics and ACE inhibitors may result in hyperkalemia.

A 39-year-old previously well Caucasian man presents to the emergency department with a 10-day history of fever >101°F and acute dyspnea with pleuritic chest pain. His past medical history is notable only for childhood asthma (no recurrences since age 12) and appendectomy. He has no known drug allergies. He denies taking prescribed medications on a regular basis. Social history is notable for use of IV drugs. Vital signs show: Temperature - 100.8°F, pulse - 108, respirations - 24, and blood pressure - 98/60. O2 saturation is 90% on room air. Physical examination reveals mild crackles of the mid-lung fields bilaterally and a grade II/VI soft systolic murmur, loudest at the left lower sternal border. A spiral CT reveals evidence of multiple pulmonary emboli. He is admitted to the general medical floor of an acute care hospital. Additional diagnostic tests are ordered; preliminary results of blood cultures showed 4+ growth of Gram-positive cocci. Infectious Diseases is consulted and he is started on an IV antibiotic regimen. Question What is the most likely causative risk factor for this patient's underlying diagnosis?

Correct answer: IV drug abuse Explanation Infective endocarditis affecting the right side of the heart (typically the tricuspid valve) is most commonly associated with IV drug abuse and is often associated with multiple pulmonary emboli. Only 5-10% of cases of endocarditis are right-sided. Bacterial endocarditis most frequently develops when bacteremia (which may be transient) occurs in the setting of a structurally abnormal cardiac valve or structurally abnormal heart. Thus, endocarditis risk factors include: valvular heart disease (including rheumatic heart disease and previous cardiac valve surgery) structural abnormalities of the heart conditions that raise the likelihood of bacteremias such as skin infection, dental infection (abscess or periodontitis), dental procedure, or IV drug abuse Right-sided endocarditis is also associated with the presence of foreign material in the heart such as: pacemaker implantable cardioverter defibrillator a portal of entry for bacteria such as central venous catheter or dialysis Although the pathophysiology has not been definitively demonstrated, IV drugs are hypothesized to cause damage to the cardiac valves via several different mechanisms, including "subendothelial damage, physiologic effects of injected drugs, differences in infecting organisms and bacterial load in IV drug users, and immunologic changes" (Frontera, et.al.).

A 64-year-old man with a history of a remote myocardial infarction and congestive heart failure presents for his 3-month follow-up. A recent echocardiogram reveals severe left ventricular dysfunction. Question Which of the following interventions has been shown to reduce the risk of sudden cardiac death in a patient such as this?

Correct answer: Implantation of a cardioverter-defibrillator device Explanation Sudden cardiac death is an unexpected nontraumatic death in clinically well or stable patients and is usually caused by ventricular fibrillation, either in isolation or preceded by ventricular tachycardia. Over 75% of victims have severe CAD, and many have old myocardial injury. Ventricular tachycardia/fibrillation can also occur in the initial 24 hours after an acute myocardial infarction. Additional conditions that predispose to sudden death include congestive/dilated cardiomyopathy, severe LVH, hypertrophic cardiomyopathy, hypoxia, electrolyte abnormalities, drug toxicity, and other dysrhythmias and conductive disturbances.

A 48-year-old man presents with a log book of home-recorded blood pressure readings between 125 - 185/75 - 100 mm Hg. His past medical history is significant for hypertension and obesity (BMI 31 kg/m2). He is a non-smoker and does not drink alcohol. He has been adhering to a sodium-restricted diet and tries to walk 2 miles at least twice a week. For the past 2 years, he has been taking hydrochlorothiazide 12.5 mg daily with no apparent side effects. Physical exam is unremarkable, and vital signs reveal a heart rate of 85 beats per minute and blood pressure of 150/85 mm Hg. Question Based on this information, what is the most appropriate next step in the management of the patient's hypertension?

Correct answer: Increase hydrochlorothiazide dose to 25 mg daily Explanation This patient is on a low dose of hydrochlorothiazide with no reported side effects. In addition, his blood pressure is consistently higher than the target of <140/90 mm Hg. In light of this information, it is most appropriate to increase his daily dose of hydrochlorothiazide in order to optimize blood pressure control. Another option would be to add a second agent.

Question Ico-delete Highlights A 42-year-old white female delivers her third child while you are on call at a local hospital on the Atlantic coast. She thanks you for your help and mentions that she is grateful that her baby was on time. She states that the family is planning on taking their newest with them to Colorado in 2 weeks. You advise her not to attempt the trip, as the baby will have difficulty adjusting to the high altitudes. Which of the following factors is a major component in closure of the ductus arteriosus?

Correct answer: Increase in oxygen tension within the blood Explanation To state that increase in pulmonary arterial pressure associated with increase blood flow to the lungs is the major component in closure of the ductus arteriosus is to state the opposite of actual events. Pulmonary pressure and pulmonary blood pressure decreases, resulting in a decrease flow from the pulmonary circuit to the systemic circuit. To state that reduction of aortic arterial pressure due to reduction in systemic resistance is a major component in closure of the ductus arteriosus is to state the opposite of actual events. After birth, systemic blood pressure rises as a result of increased effective blood volume and decreased pulmonary resistance. The ductus no longer serves as a shunt from pulmonary to systemic circulation. As gas exchange increases, the amount of carbon dioxide in the neonate will decrease. However, this does not initiate vasoconstriction and closure of the ductus arteriosus. Increase in oxygen tension is thought to be the primary initiator of closure of the ductus arteriosus. The pO2 increases from 15 to 20 mm Hg to around 100 mm Hg in a few hours. Prostaglandins are thought to be responsible for maintaining patency of the ductus arteriosus

A 78-year-old man with known left-sided congestive heart failure presents with a complaint of cough, worsening dyspnea with exertion, and orthopnea. What is the most direct cause of his symptoms?

Correct answer: Increased pulmonary venous pressure Explanation In left-sided congestive heart failure, the predominant feature is low cardiac output and elevated pulmonary venous pressure, resulting in dyspnea. As dyspnea worsens, the patient will also begin to experience shortness of breath at rest, which is worsened in the supine position. Tricuspid function is not related to CHF. Peripheral vascular resistance typically increases in CHF, which is designed to help maintain perfusion to vital organs. Mucus plugging is not associated with CHF.

A 70-year-old woman with a history of hypertension, hyperlipidemia, and myocardial infarction presents with a 3-day history of shortness of breath at rest. She has found it difficult to walk short distances due to shortness of breath and is experiencing orthopnea and nocturnal dyspnea. She denies cough, fever, chills, nausea, abdominal pain, vomiting, diarrhea, rashes, or edema. Upon physical examination, the patient is short of breath, requiring numerous pauses during conversation. She is tachycardic, diaphoretic, and extremities are cool. There is a diminished first heart sound, S3 gallop, laterally displaced PMI, bibasilar rales and dullness to percussion, and expiratory wheezing noted. There is no JVD noted, and 2+ pitting edema of the lower extremities to the level of the mid calf. Question What intervention will provide the greatest symptomatic relief to this patient?

Correct answer: Intravenous diuretic Explanation This patient exhibits signs and symptoms of congestive heart failure. The patient's manifestations represent left-sided failure predominance. Her current NYHA classification class is IV, which is confirmed by her symptoms at rest. Diuretics are the most effective means of providing symptomatic relief to patients with moderate to severe CHF.

A 55-year-old man presents with severe central chest pain. Pain started suddenly and it radiates to the back and neck. He is unable to lie flat. He feels sick but has not vomited. He has no major illnesses and knows of none that run in his family. He does not use alcohol, tobacco, or illicit substances. He is allergic to sulfa drugs. On physical exam he appears in extreme pain and is lying on his side. Temperature is 98.6°F, heart rate is 110, blood pressure is 180/105 mm Hg, and respiratory rate is 20. Cardiac exam reveals normal S1 and S2 without rubs or gallop. The top of his internal jugular venous column is present at 2 to 3 cm above the sternal notch. Chest auscultation shows normal vesicular breathing. He has normal active bowel sounds tympanic to percussion. Extremity exam is normal, and the lower motor and sensory function is intact. ECG shows left ventricular hypertrophy. Chest X-ray shows widened mediastinum. Question What treatment should be given immediately to this patient?

Correct answer: Intravenous labetalol Explanation Aortic dissection is more common in patients with hypertension, connective tissue disorders, congenital aortic stenosis or bicuspid aortic valve, and in those with first-degree relatives with history of thoracic dissections. Chest pain is the most common presenting symptom in patients with an aortic dissection. The pain is usually sudden and severe and described as ripping or tearing. In all individuals with aortic dissections, medication should be used to control high blood pressure, if present. Intravenous beta blockers, such as labetalol, are effective at reducing blood pressure and heart rate. The target systolic blood pressure should be 100 - 120 mm Hg. The target heart rate should be below 60 beats/min.

A 26-year-old African-American man with no significant past medical history presents with a history of dyspnea on exertion, which occurs after running. The dyspnea is associated with mild substernal chest pain. All symptoms are relieved with rest. He denies fever, chills, cough, wheezing, pleurisy, calf pain, abdominal complaints, peripheral edema, cigarette, drug, or alcohol use, sick contacts, or travel. His physical exam reveals a harsh murmur best heard at the left lower sternal border and an S4 gallop. A bedside electrocardiogram was remarkable for left ventricular hypertrophy and septal Q waves in the inferolateral leads. An echocardiogram noted asymmetric LVH, anterior motion of the mitral valve during systole, a small and hypercontractile LV, and delayed relaxation and filling of the LV during diastole. The septum was 2 times the thickness of the posterior wall. Question What is expected to be true regarding the murmur in this patient?

Correct answer: It decreases with squatting. Explanation This patient's presentation is significant for hypertrophic cardiomyopathy. The murmur is a loud, harsh systolic murmur present along the left sternal border. The gradient and the murmur may be enhanced by maneuvers that decrease ventricular volume, such as an upright posture, standing, or Valsalva maneuver. It is decreased by increasing ventricular volume or vascular resistance, which occurs with squatting, sustained handgrip, lying down, or straight leg raises. These maneuvers help differentiate the murmur of HOCM from that of aortic stenosis, since in HOCM, reducing the LV volume increases obstruction and the murmur intensity; whereas, in valvular aortic stenosis, reducing the stroke volume across the valve decreases the murmur

A 68-year-old woman presents with a several-month history of feeling dizzy and light-headed. She admits brief syncope once. These episodes often occur first thing in the morning when she gets out of bed and often after dinner. She admits some general weakness but otherwise feels well. She denies seizures, headaches, numbness, paresthesias, and gait or balance disturbances. The patient did some routine labs for a health fair last month and reports all values returned normal. The patient's past medical history is noncontributory. She is menopausal, takes no medications, has had no surgeries, and has no allergies. She is retired, lives with her husband, and does not exercise regularly. On physical exam, her blood pressure is taken both lying supine and then again 3 minutes later while standing upright. Readings are 110/78 mm Hg supine and 82/63 mm Hg standing. Mucus membranes are moist, and skin turgor is good. The patient's physical exam, including cardiovascular, pulmonary, and neurological, are normal. An electrocardiogram is performed and is normal. Question Assuming the workup does not reveal an underlying cause of this patient's condition, what non-medication intervention is the most appropriate for her?

Correct answer: Liberal salt and fluid intake Explanation This patient is presenting with orthostatic hypotension (OH), which is defined by a supine-to-upright positional drop in systolic blood pressure by >/= 20 mmHg, and/or a drop in diastolic blood pressure by >/= 10 mmHg. Common symptoms are lightheadedness, dizziness, and syncope, which may occur more frequently with position changes, such as getting up out of bed. Recommended interventions for OH include liberal salt and fluid intake, physical maneuvers such as leg crossing, and bending over to increase blood pressure and prescription fludrocortisone.

A 73-year-old Caucasian man presents due to worsening shortness of breath with activity over the last few months. He mentions being unable to complete as many physical activities during the day as he could 3 months ago. Physical examination reveals hypotension, tachycardia, and extremities that are cool to the touch; there is also expiratory wheezing, rhonchi auscultated during the pulmonary exam, as well as a diminished first heart sound with an S3 gallop heard during the cardiac exam. You order an ECG, but even before receiving the results, you are highly suspicious of the most likely diagnosis. Question What would be the most critical lifestyle modification necessary in regard to the patient described above?

Correct answer: Limiting salt Explanation The patient should limit his salt intake. The patient above is displaying multiple key signs and symptoms suggestive of heart failure. Heart failure may consist of primarily left ventricular (left side) or only right ventricular (right side) heart failure. Very often, patients will exhibit signs and symptoms of both right- and left-sided heart failure. Common signs and symptoms of heart failure include those of low cardiac output and congestion, as well as fluid overload. These could include dyspnea, exertional dyspnea, orthopnea, chronic nonproductive cough, edema, hepatic congestion, loss of appetite, and even nausea. Tachycardia, hypotension, reduced pulse pressure, cold extremities, diaphoresis, crackles in the lungs, expiratory wheezing and rhonchi may also be present. Cardinal cardiac signs include a parasternal lift, enlarged/sustained left ventricular impulse, and even an S3 gallop.

A 50-year-old man presents to your office for a follow up appointment of his hypertension. He has complaints of some non-specific chest discomfort, so you decide to perform an ECG. The ECG demonstrates peaked T waves in several leads without any other abnormality. Question Which of the following medications is most likely to cause this ECG finding?

Correct answer: Lisinopril Explanation The correct answer is lisinopril, as it is an angiotensin-converting enzyme inhibitor that blocks the production of aldosterone, leading to possible hyperkalemia. Peaked T waves on ECG are indicative of hyperkalemia. Furosemide is a loop diuretic and is associated with hypokalemia. Atenolol is a beta-blocker and is not associated with hyperkalemia. Hydrochlorothiazide is a thiazide diuretic and may cause potassium loss. Prazosin is an alpha-blocker antihypertensive and is not associated with hyperkalemia.

A 76-year-old man presents with substernal chest pain; it is associated with progressive exertional dyspnea, easy fatigability, and dizziness. These symptoms are exacerbated with walking short distances, and they are relieved with rest. He denies fever, chills, cough, wheezing, pleurisy, calf pain, abdominal complaints, peripheral edema, cigarette, drug use, alcohol use, sick contacts, or travel. His physical exam reveals a normal blood pressure and a rough, harsh, low-pitched crescendo-decrescendo systolic murmur beginning after the first heart sound; it is best heard at the second intercostal space in the right upper sternal border. Its intensity is increased toward midsystole; the murmur radiates to both carotid arteries and is accentuated upon squatting, and it is reduced during Valsalva strain. His lungs are without adventitious sounds. Question What medication is most appropriate for the medical management of the patient?

Correct answer: Lopressor (Metoprolol) Explanation This patient's presentation is significant for aortic stenosis. The only definitive treatment for aortic stenosis is aortic valve replacement. The development of symptoms due to aortic stenosis provides a clear indication for replacement. For patients who are not candidates for aortic replacement, percutaneous aortic balloon valvuloplasty may provide some symptom relief. The medical treatment options are limited in symptomatic patients with aortic stenosis who are not candidates for surgery. Beta-blockers might be used if the predominant symptom is angina. Those with congestive heart failure or pulmonary congestion may benefit from the cautious use of digitalis, diuretics, and angiotensin-converting enzyme (ACE) inhibitors. Although vasodilators may be used for heart failure and for hypertension associated with aortic stenosis, they should be used with extreme caution to prevent excessive decreases in preload or systemic arterial blood pressure. Reducing the blood pressure to normal levels in patients with hypertension and aortic stenosis is advisable, but hypotension must be avoided

A 65-year-old man presents with a 2-week history of progressive dyspnea, orthopnea, and pedal edema. His history is significant for heavy smoking (30 cigarettes/day for more than 40 years), a 20 pound weight loss over past 3 months, loss of appetite, and weakness. His physical examination reveals pulse of 130 per minute, blood pressure of 105/90 mmHg, paradoxic pulse, distant heart sounds, nonpalpable cardiac impulse, slight dullness at both lung bases, and bilateral pedal edema. His chest X-ray shows enlarged cardiac silhouette and a large pericardial effusion; they are confirmed by echocardiography. Acid-fast staining of the pericardial fluid is negative for acid-fast bacilli (AFB). CT scan reports are awaited. Question What is the most likely etiology of this pericardial effusion?

Correct answer: Malignant pericardial effusion Explanation Pericardial effusion is an accumulation of fluid in the pericardial space, with or without associated pericarditis. While an effusion may present with recognizable clinical findings, the most common method of detection is by echocardiography. (See "Diagnosis and treatment of pericardial effusion".) In different case series of patients with pericardial effusion, malignancy accounted for 13 to 23 percent of cases [1,6]. In areas where tuberculosis is not highly prevalent, malignancy may be the most common cause of a hemorrhagic effusion [1,7]. (See "Etiology of pericardial disease", section on 'Pericardial effusion'.) Malignancy may also be a more common cause of large symptomatic pericardial effusions. In some cases, the effusion may be the initial clinical manifestation of the malignancy. This was suggested in a review of 173 consecutive patients undergoing pericardiocentesis for symptomatic pericardial effusion [6]. Symptomatic effusions were defined as those with cardiorespiratory symptoms (eg, dyspnea), signs (eg, tachycardia), echocardiographic features of right heart compromise, or if pericardiocentesis was deemed therapeutic by the clinician. A cause for the effusion was found in all but 13 patients. The most common cause was malignancy, which was found in 58 patients (33 percent), 45 of whom had known malignant disease. After exclusion of pericardial effusions in which the etiology was apparent from the history, physical examination, and simple laboratory tests (eg, trauma, uremia, postpericardiotomy, rheumatic disease, known malignancy), newly found cancer was responsible for 18 percent of the remaining 74 cases. A pericardial effusion that arises in a patient with a known malignancy usually represents metastatic spread of the malignancy [1]. However, other etiologies for the pericardial effusion should be considered. Patients with a history of thoracic radiation treatment can develop a radiation-induced pericarditis with a pericardial effusion. In addition, patients who are immunocompromised due to treatment of an underlying malignancy can develop infectious or autoimmune pericardial effusions [1]. (See "Etiology of pericardial disease", section on 'Pericardial effusion' and "Diagnosis and treatment of pericardial effusion" and "Cardiotoxicity of radiation therapy for breast cancer and other malignancies", section on 'Esophageal cancer'.)

A 35-year-old Costa Rican woman emigrated to the United States 5 years ago. She presents to the office with complaints of chronic, progressive dyspnea. She reports no chest pain, fever, or cough and is a non-smoker. Her daily activities are becoming increasingly limited due to her shortness of breath. On auscultation she has an accentuated S1, a loud opening snap, and a diastolic murmur heard best at the apex in the left lateral recumbent position. She has 1+ ankle edema bilaterally. Question What is the most likely cause of this patient's symptoms?

Correct answer: Mitral stenosis Explanation Mitral stenosis causes an opening snap and a diastolic murmur. It is most often a result of rheumatic fever. Incidence of rheumatic fever is declining in the US, but remains a problem in developing countries with tropical climates. Most symptoms begin in the 4th decade and it is more common in females. Mitral valve prolapse usually causes a murmur and a mid-systolic click. Tricuspid stenosis rarely occurs without mitral stenosis. Aortic stenosis symptoms more commonly develop in the 6th to 8th decades, with sounds heard best at the right second intercostal space, radiating into the neck. Aortic regurgitation is accompanied by a high pitched blowing decrescendo murmur, found predominantly in males, and is heard best at the 3rd intercostal space, left sternal border, with the patient sitting and leaning forward.

A 22-year-old woman sees you for a physical examination. When questioning her for the medical history, you discover that she has a history of rheumatic fever. Upon listening to her heart, you detect a diastolic murmur. The murmur is low-pitched and begins with a loud snapping sound. Question What valvular abnormality did you probably hear?

Correct answer: Mitral stenosis Explanation Mitral stenosis is the valvular abnormality that is seen most frequently as a consequence of rheumatic fever, although other valvular lesions may also occur. Mitral stenosis produces a diastolic murmur that is heard loudest at the apex and with the patient in the lateral decubitus position. The murmur starts just after S2 and begins with a loud snapping sound. Mitral regurgitation is a systolic, not diastolic, murmur. Aortic regurgitation is also a diastolic murmur, but it is most commonly described as a high-pitched murmur. Aortic stenosis is a systolic murmur. Tricuspid stenosis is almost always a consequence of rheumatic heart disease. A mid-diastolic murmur is usually present and heard best over the left sternal border. Classically, the murmur increases in intensity with inspiration.

A 42-year-old man with a prior history of rheumatic fever as a child presents with the sudden onset of acute aphasia and a right hemiparesis. On auscultation of the heart, a diastolic murmur is heard over the mitral area. Question What is the most likely cause of the patient's neurological findings?

Correct answer: Mitral stenosis with an atrial thrombus Explanation Rheumatic heart disease (RHD) leads to a significant risk of systemic arterial embolization. Approximately 20% of patients with RHD experience arterial thromboembolic complication at some time in their disease. The cerebral circulation is affected about 40% of the time. The highest risk of arterial embolization occurs in patients with atrial thrombosis. The proportion of patients developing left atrial thrombosis increases when atrial fibrillation (AF) is present. Approximately 87% of patients with atrial thrombosis are in AF. Aterial embolization occurs most commonly when the valvular lesion is associated with mitral stenosis MS). Echocardiography plays a key role in the diagnosis of left atrial thrombosis. Cerebral embolism is associated with an increased morbidity and mortality along with a high probability of recurrence. Within the first 14 days of the initial presentation, the recurrence rate is approximately 13%. Numerous studies have shown that anticoagulants can reduce the rate of these thromboembolic events. Therefore, coumadin is currently recommended in patients with RHD, especially in those with a prior history of embolization or known valvular disease or AF.

Case Ico-delete Highlights An 80-year-old woman has a history of a myocardial infarction about 5 weeks ago. The patient now presents to the ED with a history of congestive heart failure with symptoms of shortness of breath and peripheral edema, which appear to be worsening since her MI. An ECG is conducted, which reveals a prior inferior wall infarct. Cardiology is consulted where a 2-dimensional echo is conducted, and displays severe leaflet tethering and an enlarged left ventricle. Question What structure is most likely to be affected?

Correct answer: Mitral valve Explanation The clinical picture is suggestive of the malfunction of the mitral valve due to mitral regurgitation. Ischemic mitral regurgitation following a myocardial infarction is connected to a high percentage of morbidity and mortality. Ischemic mitral regurgitation can result from 3 types: type I, II, and IIIb dysfunction. Type II dysfunction after myocardial infarction can result from the rupture of a papillary muscle. This usually involves the posteromedial papillary muscle. Additionally, this can also occur when a fibrotic papillary muscle elongates, which can cause leaflet prolapse. However, type IIIb, or Carpentier's, dysfunction is the most common and significant form of ischemic mitral regurgitation. Pulmonic valve is not correct, and occurs mostly as a result from a congenital heart defect, which would have been diagnosed earlier in his life. Tricuspid valve is not the correct answer because even though this structure can be affected, it is not the most affected in light of the information provided from the echo.

A 62-year-old man with a 15-year history of hypertension presents with severe, tearing chest pain radiating through to the back. Blood pressure is 180/110 mmHg, heart rate is 120 bpm, and respiratory rate is 34/min. Physical examination findings include neck negative for bruits/JVD, lungs clear to auscultation, regular heart rhythm, normal S1/S2 with an S4 present, and a grade III/IV diastolic rumbling murmur noted with the patient leaning forward. Radial pulses are 1+ on right and 3+ on left. EKG reveals a sinus tachycardia and evidence of left ventricular hypertrophy. A STAT chest X-ray shows a widening of the mediastinum. The patient is stabilized by delivering a beta blocker intravenously. Question In order to confirm the diagnosis, what test should be ordered?

Correct answer: Multiplanar chest CT scan Explanation The correct response is multiplanar chest CT scan. The confirmatory imaging techniques most useful in detecting dissection include unenhanced CT prior to contrast-enhanced CT, transesophageal echocardiography, MRI, and contrast angiography.

A 55-year-old man presents with a 30-minute history of retrosternal pain that radiates to the jaw. He describes it as constricting in nature and worsening in intensity. He denies any trauma to the chest. On examination, he is diaphoretic and dyspneic. Question What is the most likely cause of this patient's chest pain?

Correct answer: Myocardial infarction Explanation Patients with myocardial infarction usually present with a left-sided or retrosternal pain that may radiate to the jaw, neck, and shoulder. They describe it as heaviness or a squeezing sensation. It is of variable duration and often lasts for more than 30 minutes. There is a gradual intensification of the pain. The onset of the pain may be during physical exertion or at rest. It is not relieved by nitroglycerine. On examination, patients may be dyspneic and diaphoretic. Tuberculous pleuritis chest pain ispleuritic and aggravated by coughing. It is described as sharp. Other symptoms such as fever, dyspnea, and weight loss may also be present. On examination, there may be dullness on percussion as well as absent breath sounds on the affected side. In Tietze's syndrome, patients present with anterior chest pain that is aggravated by taking a deep breath, sneezing, and turning motions. On examination, the affected costochondral junctions are erythematous, warm, and tender on palpation. The most commonly affected are the 2nd or 3rd costochondral joints. Chest pain due to a panic disorder has a variable presentation. It can be retrosternal or localized; it can be brief or last for over 30 minutes. Numerous terms, such as aching and sharp, can be used to describe it. Other symptoms of a panic disorder, such as feeling lightheaded, short of breath, or nauseated, paresthesias, palpitations, derealization, and the fear of losing control, may also be present. The history helps elicit precipitating factors and prior panic attacks. In the Mallory-Weiss Syndrome, patients present with hematemesis, especially after repeated severe retching and vomiting, which results in a mucosal tear at the gastroesophageal junction. They may also have retrosternal chest pain.

A 66-year-old man with a past medical history of diabetes mellitus type II, reflux esophagitis, hypothyroidism, and hyperlipidemia presents with constant, severe "squeezing, crushing chest pressure and tightness" that is experienced in left chest, left shoulder pain and epigastrum. He states that he had just eaten breakfast when the pain began and that his pain has persisted over the last 45 minutes. He denies any additional provoking or relieving factors, including an over-the-counter antacid. He admits to associated weakness, lightheadedness, nausea, and 2 episodes of vomiting. He denies fever, chills, cough, shortness of breath, hemoptysis, or pleurisy. Upon physical exam, he is found to be anxious and restless while his skin was cool and diaphoretic. His blood pressure is 140/91, and he has an obese BMI. His chest is free of deformity or tenderness. His heart rate was normal with a regular rhythm and free of murmurs, gallops, or rubs. His cardiac, musculoskeletal, and pulmonary exams were normal. A bedside EKG revealed ST-segment elevations of 2 mm in 3 anatomically contiguous leads. Question What is the most likely diagnosis?

Correct answer: Myocardial infarction Explanation This patient's most likely diagnosis is acute myocardial infarction. The pain associated with this diagnosis is deep and visceral; it is typically described as heavy, squeezing, and crushing, and less commonly as stabbing or burning. It is similar in character to the discomfort of angina pectoris but commonly occurs at rest, is usually more severe, and lasts longer. Unlike angina, it is not relieved with rest if precipitated by exertion. Typically, the pain involves the central portion of the chest and/or the epigastrium, and, on occasion, it radiates to the arms. Less common sites of radiation include the abdomen, back, lower jaw, and neck. It is often accompanied by weakness, sweating, nausea, vomiting, anxiety, and a sense of impending doom. Additional physical exam findings include anxiousness, restlessness, pallor, diaphoresis, cool extremities, an S3 or S4 gallop, paradoxical splitting of the second heart sound, a transient midsystolic or late systolic apical systolic murmur due to dysfunction of the mitral valve, pericardial friction rub with transmural STEMI. Notable ECG findings include ST-T segment (>1-mm elevation or depression) and T-wave (inversion) changes suggest ischemia; Q-wave suggests accomplished infarction; ST-elevation is absent in unstable angina and NSTEMI; new bundle branch block or sustained ventricular tachycardia indicates a higher risk of progression to infarction. Burning epigastric pain is the most classic symptom of peptic ulcer disease. The pain also may be described as sharp, dull, an ache, or an "empty" or "hungry" feeling. Pain may be relieved by ingestion of milk, food, or antacids, presumably due to buffering and/or dilution of acid.

A 62-year-old woman presents with extreme fatigue and shortness of breath. The symptoms began about 24 hours ago and have progressively worsened within the last 4 hours. Vital signs on arrival are as follows: HR 90 beats per minute; BP 165/72 mm Hg; RR 16/min; SpO2 98% on 4L/min supplemental oxygen by nasal cannula. 12-lead ECG demonstrates ST-segment elevation of 2 mm in leads V4-V6. Question In addition to an aspirin tablet, what medication would be most appropriate in the emergency management of this patient?

Correct answer: Nitroglycerin Explanation The patient's presentation is consistent with acute myocardial infarction. Emergency department management of patients with acute coronary syndromes — which include acute myocardial infarction and unstable angina — should consist of supplemental oxygen to maintain SpO2 >90%, oral aspirin 160-325 mg, and sublingual nitroglycerin unless contraindicated (for example, in the context of hypotension).

A 21-year-old man with a history of cocaine and methamphetamine abuse is brought to the emergency department after being found unconscious. He undergoes an extensive work up in the ED and is admitted to the ICU with acute heart failure as evidenced by an enlarged left ventricle, decreased cardiac contractility, and systolic dysfunction on echo. Currently, the patient is awake and alert. His vital signs are as follows: BP 132/88 mm Hg, HR 86 and regular, RR 20, Temp 98.2 F, and O2 sat of 100% on 4L via nasal cannula. Physical exam reveals no murmur, clear lung sounds, and no peripheral edema. He has vastly improved over the last 24 hours, and his oxygen needs are decreasing. Question What is the most appropriate therapy at this time?

Correct answer: No medication therapy at this time Explanation The patient's heart failure should be classified as dilated cardiomyopathy, likely secondary to cocaine and methamphetamine abuse. In general, roughly 50% of patients with dilated cardiomyopathy will have spontaneous recovery, especially patients with a known causative factor such as substance abuse.1 Our patient has a known causative factor, is spontaneously improving, young and otherwise healthy, and stable at this time; therefore, no pharmacological therapy is needed at this time. For patients requiring pharmaceutical intervention, a combination of a β-blocker and ACE-I provide the most benefit.1 There is no indication that this patient requires anticoagulation at this time, and therefore Warfarin and Lovenox are not recommended. While Lasix, a diuretic, will provide some relief, it is likely unnecessary in an improving patient with no signs or symptoms of volume overload. Amiodarone, used to treat dysrhythmias, is not necessary in our patient with a regular heart rhythm.

A 65-year-old man with a 10-year history of hypertension controlled with lisinopril comes for preoperative evaluation before his arthroscopic knee surgery. He has never smoked and has never had either a heart attack, congestive failure, arrhythmia, valvular problems, or abnormal lipids. His activity level is limited by knee pain; he does light to moderate housework and occasional swimming without dyspnea or chest pain. He is sexually active. His medications include ibuprofen for knee pain. His BMI is 26.5, blood pressure is 120/80 mm Hg, and the remainder of his exam is normal. His last electrocardiogram (EKG), taken 5 years ago shortly after a panic attack, showed normal sinus rhythm. His most recent (fasting) serum creatinine was 1.6 mg/dl, with a blood urea nitrogen of 20 mg/dl. Past serum creatinines were in the 1.4-1.8 mg/dl range. He has no proteinuria. Question What further preoperative cardiac evaluation, if any, is indicated?

Correct answer: No perioperative cardiac testing needed, consider EKG. Explanation This patient requires no preoperative cardiac testing, based on his functional capacity, surgical risk, and clinical risk factors. Although he has some minor (age >45 years old), and intermediate clinical predictors of increased cardiac risk (some of his creatinines have been elevated to >1.5 mg/dl), under the most recent American College of cardiology guidelines, he may proceed to surgery without testing, given that his exercise tolerance is good and his anticipated type of surgery( orthopedic) represents moderate cardiac risk. Prior to surgery, an assessment of (A) the cardiac risks associated with the surgery, (B) the patient's functional capacity (METS tolerated), and (C) the patient's overall cardiac risk should be assessed.

A 34-year-old man presents with a 2-week history of severe fatigue, increased swelling in both feet, and slight pain in the right abdomen. He gives a history of shortness of breath on severe exertion such as trekking and intense exercises. Exam reveals an afebrile patient with a prominent right atrium on the right border of the sternum on palpation. There is pedal edema and tender hepatomegaly. There are prominent 'a' waves on the jugular venous pulsations (JVP). Auscultation reveals a tricuspid opening snap. A diastolic murmur is also heard in the tricuspid area, which increases on inspiration. There is also a widely split S1 heard. Diagnostic tests reveal a normal hemogram. Right atrial enlargement is seen on the chest X-ray, and the echocardiogram shows a decreased tricuspid valvular diameter. What is the cause of tender hepatomegaly in this patient?

Correct answer: Obstructed venous flow to the right ventricle leading to backflow pressure and hepatomegaly Explanation The above clinical features suggest the presence of tricuspid stenosis. Tricuspid stenosis results from alterations in the structure of the tricuspid valve that leads to incomplete closure of the valve leaflets. The most common cause is rheumatic fever, and tricuspid valve involvement occurs in conjunction with mitral and aortic valve involvement. In this case, the valve leaflets become thickened and sclerotic as the chordae tendineae become shortened. This hinders blood flow into the right ventricle and to the pulmonary vasculature. The obstructed venous flow into the right ventricle results in backflow of blood, which leads to back pressure, hepatic enlargement, decreased pulmonary blood flow, and edema of the lower extremities. This also leads to right atrial enlargement. Other causes of tricuspid stenosis are endocarditis, endomyocardial fibrosis, congenital tricuspid atresia, and carcinoid syndrome.

A 3-month-old term infant presents for a wellness exam, and his parents report worsening in his feeding habits and rapid breathing. On exam, there is a grade III/VI harsh heart murmur heard over the left subclavicular region that starts in early systole, peaks at S2, and decrescendos until the next S1. The infant is tachypneic and has bounding pulses of his upper and lower extremities. All other findings are within normal limits. A chest x-ray shows an enlarged left ventricle and atrium and increased pulmonary vasculature markings. Question Based on the above findings, what is the most likely diagnosis?

Correct answer: Patent Ductus Arteriosus Explanation The correct answer is patent ductus arteriosus (PDA), because this is a continuous murmur heard throughout S1 and S2 with a crescendo decrescendo sound. Patients often experience exercise intolerance, palpitations, tachycardia, dyspnea, and fatigue at an older age. As an infant, they can present with early congestive heart failure symptoms, tachycardia, poor feeding, slow growth, tachypnea, and recurrent lower respiratory tract infections. The murmur is best heard over the left subclavicular region. With a large sized PDA, chest x-ray findings show enlargement of the left ventricle and left atria. There is also an increase in pulmonary vasculature. This is due to the left-to-right shunting, which causes volume overload and enlargement of these areas. Atrial septal defect (ASD) is incorrect because the murmur is a widened, fixed splitting pattern during S2. The systolic murmur is heard at the upper left sternal border. On chest x-ray, there is enlargement of the right atrial and right ventricle because of the left-to-right shunting that occurs. Congenital pulmonic valve stenosis is incorrect because this tends to be asymptomatic. If symptoms do occur, they present with right-sided heart failure, abdominal fullness, and pedal edema in advanced disease. On exam, a prominent a wave is present, and a right ventricular heave is palpated. The murmur is a loud, late-peaking crescendo decrescendo systolic ejection murmur heart at the upper left sternal border. Ventricular septal defect is incorrect because this is a harsh, holosystolic murmur heard best at the left sternal border. Most people remain asymptomatic. A systolic thrill is often palpated on exam. Congenital aortic valve stenosis is incorrect because this is a harsh, crescendo decrescendo systolic murmur. It is loudest at the base of the heart and radiates to the neck. This is present from birth unlike the murmurs in ASD, VSD, and PDA.

A 48-year-old man with hypertension and coronary artery disease presents with protracted fever, fatigue, anorexia, weight loss, night sweats, and non-specific, non-radiating joint pains. These symptoms began insidiously following a routine dental cleaning, but they have progressed over the past 4 weeks. He denies any chills, myalgias, sore throat, palpitations, shortness of breath, pleurisy, cough, wheezing, abdominal pain, nausea, vomiting, diarrhea, peripheral edema, trauma, travel, insect bites, or sexual contact within the past year. His physical exam is remarkable for a fever of 101.3°F as well as petechiae of the soft palate and palpebral conjunctiva. His fundoscopic examination is notable for cytoid bodies and hemorrhages, while his oral mucosa displays conjunctival petechiae. There is a palpable purpuric skin rash of the lower extremities, reduced bilateral radial and ulnar pulsations, linear hemorrhages under the nails not reaching the nail margin, as well as tender, erythematous nodules occurring in the pulp of the fingers. His cardiac exam demonstrates a soft, medium-pitched holosystolic murmur located at the apex with radiation to the axilla, while his foot exam reveals the findings in the attached image. A comparison to the patient's last physical exam reveals no abnormal physical exam findings. Question What pharmacotherapeutic agent is most appropriate for this patient?

Correct answer: Penicillin G Explanation The correct response is penicillin G. This patient's presentation is most consistent with native valve endocarditis caused by Viridans group streptococci ( -hemolytic streptococci). These are a frequent cause of community-acquired native valve endocarditis. Viridans streptococci are normal residents of the oropharynx and easily gain access to the circulation after dental or gingival trauma. Adult native valve endocarditis caused by penicillin-susceptible S. viridans, S. bovis, and other streptococci should be treated with one of the following regimens: penicillin G at 12 - 18 million U/d IV by continuous pump or in 6 equally divided doses for 4 weeks, ceftriaxone at 2 g/d IV for 4 weeks, or penicillin G or ceftriaxone and gentamicin at 1 mg/kg (based on ideal body weight) every 8 hours for 2 weeks. The emergence of methicillin-resistant S. aureus (MRSA) and penicillin-resistant streptococci has led to a change in empiric treatment with liberal substitution of vancomycin in lieu of a penicillin antibiotic. Rifampin should be considered if a prosthetic heart valve has been placed within the past year. It is necessary in treating individuals with infection of prosthetic valves or other foreign bodies because it can penetrate the biofilm of most of the pathogens that infect these devices. However, it should be administered with vancomycin or gentamicin. Linezolid or daptomycin are options for patients with intolerance to vancomycin or resistant organisms Doxycycline combined with rifampin, trimethoprim-sulfamethoxazole, or a fluoroquinolone for a duration of 3 - 4 years is used to treat Q fever caused by C burnetii infection. 2015 American Heart Association Clinical Guidelines for Infective Endocarditis lists Ampicillin as a "reasonable alternative to penicillin if a penicillin shortage exists". Also, continuously infused ampicillin is probably the best therapy for aminoglycoside-resistant enterococci. Alternative choices are imipenem, ciprofloxacin, or ampicillin with sulbactam.

A 52-year-old patient, with a known case of renovascular hypertension, presents with poorly-controlled hypertension. He has been treated with both enalapril and nifedipine. He had been diagnosed with unilateral left renal artery stenosis, but recent tests have demonstrated mild changes in the right renal artery also. What should be the next step in management?

Correct answer: Percutaneous transluminal angioplasty Explanation The correct choice would be percutaneous transluminal angioplasty because the patient is already on 2 antihypertensives, yet his hypertension is poorly controlled. Also, he is gradually developing bilateral renal artery stenosis. This will probably worsen his hypertension. If there are no contraindications for a surgical repair of the arteriosclerotic artery, it is the preferred course of action. It cures the hypertension without the need for medication. Percutaneous transluminal angioplasty is the procedure of choice in symptomatic stenosis. Additional stenting can also be done. This procedure has shown 90% success rates.

A 45-year-old man presents with a 6-hour history of retrosternal chest pain. He describes it as sharp and episodic; it is relieved by sitting upright and worsened by lying down. He denies any trauma to the chest. ECG reveals diffuse ST elevations in inferior and lateral leads with depression of the PR segments. Question What is the most likely cause of his chest pain?

Correct answer: Pericarditis Explanation Pericarditis pain is usually described as a sharp, retrosternal pain that is often worse when the patient is supine and relieved by sitting upright. It is aggravated by deep inspiration and changes in position. It may be episodic and last for hours to days. On examination, a pericardial friction rub may be auscultated. This patient's ECG findings are characteristic of pericarditis. Angina pain is usually described as tightness or pressure on the chest; it lasts for less than 10 minutes. It is usually left-sided or retrosternal, and it may radiate to the jaw, neck, and shoulder. It is precipitated by physical exertion or emotional stress. It is relieved by rest and sublingual nitroglycerin.

A 35-year-old woman presents for follow-up. She has a 6-month past medical history of hypertension, which has responded poorly to lifestyle approaches. She denies using any medications; she does not smoke or use illicit drugs. Her review of systems is notable for muscular weakness, paresthesias, headaches, polyuria, and polydipsia. On physical exam, her blood pressure is 155/95 mm Hg. She has generalized muscular weakness that is measured in all 4 extremities. The remainder of her exam is unremarkable. Laboratory analysis reveals hypokalemia and a hemoglobin A1c level of 5.5. Question What is the most likely diagnosis?

Correct answer: Primary Aldosteronism Explanation The correct response is Primary aldosteronism. This patient is demonstrating signs and symptoms consistent with primary hyperaldosteronism, which is most commonly caused by a unilateral adrenocortical adenoma (Conn's syndrome), but in a minority of patients, it is caused by adrenal hyperplasia. Screening for primary aldosteronism is done by the measurement of serum aldosterone (ng/dL) and plasma renin activity (PRA) (ng/mL/hour) and the expression of the results as a ratio. A ratio greater than 30 is highly suspicious if the absolute level of serum aldosterone is greater than 15 ng/dL. Patients with a high aldosterone/PRA ratio, but levels of serum aldosterone less than 9 ng/dL, are almost never found to have primary aldosteronism. Hyperaldosteronism is suggested when the plasma aldosterone concentration is elevated (normal: 1 - 16 ng/dL) in association with suppression of plasma renin activity.

A 5-day-old male infant has subtle, unusual facial features (i.e., a triangular face, hypertelorism, and down-slanting eyes). He also has a webbed neck and low-set ears. Suspecting a congenital disorder, you order a complete work-up, including a CBC, coagulation profile, cardiac evaluation, karyotyping, and mutation analysis. PTPN11 (protein-tyrosine phosphatase, nonreceptor-type 11) mutations are detected. Echocardiography detects a cardiac defect. Question What is most likely to be found on echocardiography?

Correct answer: Pulmonary stenosis Explanation This neonate most likely has Noonan syndrome (NS). Pulmonary stenosis is the most common cardiac defect in this condition. Noonan syndrome is a sporadic, or autosomal dominant, congenital disorder with typical phenotypical features that may not be visible to the casual onlooker. The most common facial features include hypertelorism and low-set, backward-rotated ears with a thick helix. The philtrum is deeply grooved in more than 90% of cases. Congenital cardiac defects, bleeding disorders, intellectual disability, webbed neck, and a short stature are other features. Pulmonary valve stenosis is the most common cardiac defect and is seen in 50% of those affected. Other cardiac findings include atrial and/or ventricular septal defects and cardiomyopathies. PTPN11 mutations occur in 50% of affected persons. SOS1 and KRAS mutations are other causes, but the absence of a mutation does not rule out NS. Although all types of cardiac valvular defects have been described in NS, pulmonary stenosis is the most common.

A 72-year-old man presents for worsening shortness of breath, orthopnea, and chest pain for the last few weeks. He admits some chronic heart problems with fatigue, dyspnea, and non-productive cough, but feels like symptoms have worsened recently. He denies fever, chills, and productive cough. On physical exam, the man has mildly increased respiratory effort but does not appear in distress. He is barrel-chested. His breath sounds are diminished bilaterally, with dullness to percussion over right and left lower lungs. No pleural friction rub is noted. On cardiovascular exam, an S3 gallop and mild tachycardia (110 bpm) is noted. Clubbing of the fingers, dependent edema in the lower extremities, and jugular venous distention is also noted. His cardiac enzymes and electrocardiogram demonstrate no acute cardiac pathology. Pleural fluid and cardiomegaly are found on the chest x-ray. Question What explanation for this patient's dependent edema is most plausible, given his history and physical?

Correct answer: Renal sodium and fluid retention, leading to increased capillary pressure Explanation This patient is presenting with an acute exacerbation of heart failure (HF). HF is a common, chronic condition and can have acute exacerbations. Common HF presentations include fatigue, dyspnea with exertion, and fluid overload. Peripheral (dependent) edema takes time to develop and is renally-caused. Because of the initially low blood flow to the kidneys, their response is to activate the renin-angiotensin-aldosterone system, which also leads to increased antidiuretic hormone (ADH or vasopressin) release. The result is renal sodium and fluid retention, leading to increased capillary pressure. The serous fluid leaks out into the interstitial tissue, causing the edema.

A 39-year-old man presents with a 1-week history of severe chest pain. He states that the pain seems to worsen when he lies down. He describes the pain as radiating to the back and worsening when he takes a deep breath. His vital signs are as follows: blood pressure 124/84 mm Hg, respiratory rate 18/min, temperature 101°F, and pulse 74 beats per minute. On auscultation of the chest, you cannot distinguish an S1 or S2 but hear a scratching or grating sound. Question What is the first step in the treatment of this patient?

Correct answer: Rest and NSAIDs Explanation The clinical picture is suggestive of acute pericarditis. Most cases are due to viral infections with the treatment being rest and non-steroidal agents, e.g. aspirin or indomethacin. If this pericarditis progressed to tamponade, pericardiocentesis would be indicated. Symptoms of tamponade are not seen in this patient (dyspnea, elevated jugular venous pressure, hypotension, paradoxical pulse, and muffled heart sounds). Beta-blockers and calcium channel blockers are not indicated for treating pericarditis. Corticosteroids are usually given in cases unresponsive to rest and NSAIDs. Most cases of pericarditis are self-limiting and usually run their course from 1-3 weeks, but initial treatment consists of rest and NSAIDs.

A 35-year-old Caucasian woman presents with consistent Stage I hypertension on 3 separate visits and has already attempted lifestyle changes to decrease blood pressure with little success. You decide to start her on hydrochlorothiazide 25 mg daily and have her return in 1 week for a blood pressure check. Which of the following statements is true about thiazide diuretics?

Correct answer: They inhibit sodium reabsorption in the distal tubule Explanation The correct answer is they inhibit sodium reabsorption in the distal tubule by inhibiting the sodium-chloride transporter. Decreased sodium in the interstitium leads to less water reabsorbed. Carbonic anhydrase inhibitors act on the proximal convoluted tubule by inhibiting carbonate from being reabsorbed in the interstitium leading to increased sodium, bicarbonate, and water in the urine. Potassium-sparing diuretics act as aldosterone receptor antagonists in the distal tubule, causing more water and sodium to pass through the tubule. Loop diuretics inhibit the reabsorption of sodium in the thick ascending loop increasing sodium and water loss, and eventually potassium loss in the distal tubule.

A 72-year-old man presents with 'being short of breath'; he feels as though his belly is bloated and his legs are swollen. Past medical history includes high blood pressure (for which he is currently taking lisinopril), as well as high cholesterol being controlled with diet modifications. The patient also has a past diagnosis of mediastinal lung cancer around 5 years ago for which he received radiation treatment as part of his prescribed therapeutic regimen. Physical examination reveals an elevated jugular venous pressure and Kussmaul sign. Moderate pitting edema and ascites are also observed. Chest radiograph reveals only mild cardiomegaly. Echocardiogram reveals a normal left ventricle chamber size, normal LVEF, and a thickened atrial septa. Question Based on the history, physical exam, and diagnostic studies, what is the most likely diagnosis?

Correct answer: Restricted cardiomyopathy Explanation The most likely diagnosis for this patient is restricted cardiomyopathy (RCM). This is likely to be due (secondarily) to the radiation treatment that the patient had for his past diagnosis of mediastinal lung cancer. RCM also is referred to as radiation cardiomyopathy; RCM caused by radiation is more common in patients who have a history of breast or lung cancer that has been treated with thoracic-area radiation. RCM is categorized as a type of fibrotic restrictive cardiomyopathy. In RCM, right heart failure pathologies usually predominate over left heart failure. Significant symptoms and signs of pulmonary hypertension will be evident (e.g., peripheral edema and ascites); dyspnea and fatigue will also be present, as will slight cardiomegaly. In RCM seen via echocardiogram, this pathology may have only slight (or even normal) left ventricular function and size with a decreased LVEF.

A 73-year-old man presents with worsening shortness of breath on activity over the last few months. Another issue he mentions is not being able to complete as many physical activities during the day as he could perform 3 months ago. He states he has to use at least 3 pillows to allow him to sleep at night, otherwise he cannot breathe. Physical examination further confirms your suspicions regarding the patient's diagnosis. Question What sign would be consistent with this patient's most likely diagnosis?

Correct answer: S3 gallop Explanation The patient above is displaying multiple key signs and symptoms suggestive of heart failure. Heart failure may consist of primarily left ventricular (left side) or right ventricular (right side) heart failure. Very often patients will exhibit signs and symptoms of both right and left sided heart failure. Common signs and symptoms of heart failure include those of low cardiac output and congestion as well as fluid overload. This could include dyspnea, exertional dyspnea, orthopnea, chronic nonproductive cough, edema, hepatic congestion, loss of appetite, and even nausea. Tachycardia, hypotension, reduced pulse pressure, cold extremities, diaphoresis, crackles in the lungs, expiratory wheezing, and rhonchi may also be present. Cardinal cardiac signs include a parasternal lift, enlarged/sustained left ventricular impulse, and even an S3 gallop.

Case Ico-delete Highlights A 47-year-old man presents with dyspnea on exertion, palpitations, and a persistent cough. He had been generally healthy up until about a week ago, when he began to notice shortness of breath and weakness. Physical examination of the chest reveals crackles at the bilateral bases and tachycardia. An echocardiogram shows a small mass within the right atrium. Vital signs are as follows: blood pressure 166/110 mm Hg, pulse 91 BPM, respirations 22/min, temperature 98.7°F. Biopsy reveals a malignant tumor. Question What is the most likely diagnosis?

Correct answer: Sarcoma Explanation The majority of malignant heart tumors are sarcomas. A large portion of sarcomas are angiosarcomas and have a predilection for the right atrium. They occur mostly in the age group 20 - 40 years; however, no age is spared. These types of tumors may include angiosarcomas, fibrosarcomas, rhabdomyosarcomas, and liposarcomas. Rhabdomyosarcomas and angiosarcomas are more common in men than in women. Diagnosis of cardiac tumors is usually by echocardiography and biopsy. Left heart disease may present as obstruction of the mitral valve or often subacute bacterial endocarditis. Right heart disease may present as tricuspid valve disease, hypertension, or pulmonary embolism. Malignant cardiac tumors are very rare, and the majority of them are due to metastatic involvement of the heart. Primary cardiac tumors are even rarer and may arise from mesothelial or epithelial tissue. They may be benign or malignant. Apart from sarcomas, malignant tumors that metastatize to the heart include carcinomas, leukemia, and reticuloendothelial tumors. Lung and breast cancers often invade the heart. Lymphomas are extremely rare. They are mainly seen in patients with underlying immunodeficiencies. They are known to cause arrhythmias, cardiac tamponade, superior vena cava syndrome, and heart failure. They also grow very rapidly, and prognosis is generally poor. Because tumors in the heart can mimic other heart diseases, they are difficult to diagnose accurately. Tumors of the heart may involve the endocardium, myocardium, or epicardium. Symptoms may include palpitations, syncopy, and heart failure. Symptoms may appear similar to other diseases, such as endocarditis. Blood flow blockage may occur. This is seen commonly around the mitral valve, but it can occur around any valve. Other symptoms may include those of lung congestion.

A 76-year-old man with a past medical history of hyperlipidemia and diabetes mellitus presents to the emergency room with a 2-hour history of acute, severe, "crushing" left precordial chest pain; it is associated with nausea, vomiting, diaphoresis, and altered mental status. His physical exam is notable for an ashen and cyanotic appearance, hypotension, rapid and weak peripheral pulsations, distant heart sounds, elevated jugular venous distension and pulmonary crackles. A stat bedside chest X-ray reveals the following image. Question What is correct regarding the diagnostic work-up of this patient?

Correct answer: Serum lactate levels are expected to be elevated Explanation The patient is presenting with signs and symptoms consistent with cardiogenic shock due to myocardial infarction. On chest X-ray, findings of left ventricular failure are visualized. These radiologic features include pulmonary vascular redistribution, interstitial pulmonary edema, enlarged hilar shadows, the presence of Kerley B lines, cardiomegaly, and bilateral pleural effusions. Alveolar edema manifests as bilateral perihilar opacities in a so-called butterfly distribution. An elevated serum lactate level is an indicator of shock. Serial lactate measurements are useful markers of hypoperfusion and are also used as indicators of prognosis. Elevated lactate values in a patient with signs of hypoperfusion indicate a poor prognosis; rising lactate values during resuscitation portend a very high mortality rate. A low BNP level may effectively rule out cardiogenic shock in the setting of hypotension. The hemodynamic measurements of cardiogenic shock are a pulmonary capillary wedge pressure (PCWP) of greater than 15 mm Hg and a cardiac index of less than 2.2 L/min/m2.

A 56-year-old man is diagnosed with primary hypertension. His additional medical history includes only nephrolithiasis. Past analysis of his kidney stones has revealed a calcium oxalate composition. Question In addition to controlling his blood pressure, what class of antihypertensive agents might be helpful in managing his kidney stones?

Correct answer: Thiazide diuretics Explanation All of the agents listed might be used as antihypertensive therapy. However, only the thiazide diuretics may also reduce urinary calcium excretion and therefore limit the occurrence of calcium oxalate nephrolithiasis. The thiazide diuretics act in the kidney at the luminal side of distal convoluted tubule to inhibit the Na+/Cl- cotransporter, thereby increasing urinary sodium, chloride, and water excretion. This leads to decreases in extracellular fluid and plasma volumes. Thiazide diuretics also directly reduce peripheral arteriolar resistance, which in turn lowers both systolic and diastolic blood pressures. In addition to their antihypertensive effect, thiazide diuretics reduce urinary calcium excretion by indirectly increasing the activity of the 2Na+/Ca2+cotransporter, also in the distal tubule. This occurs because relatively more sodium is delivered to this transporter due to the blockade of the Na+/Cl- cotransporter. Therefore, thiazide diuretics can be helpful in controlling kidney stones that occur as a result of hypercalciuria. Like the thiazides, loop diuretics reduce blood pressure by decreasing fluid retention and normalizing plasma volume. Loop diuretics act on the ascending limp of the loop of Henle and block the Na+/K+/2Cl- cotransporter. They are more potent than thiazide diuretics because the transporter on which they act is responsible for a greater fraction of salt and water absorption. Unlike the thiazide diuretics, loop diuretics increase urinary calcium excretion, which may enhance the formation of calcium oxalate kidney stones. Angiotensin-converting enzyme (ACE) inhibitors block the conversion of angiotensin I to angiotensin II. Angiotensin II promotes the production of aldosterone, which causes sodium and water retention. Angiotensin II also stimulates the sympathetic nervous system. ACE is also responsible for the breakdown of the vasodilator bradykinin. Blockade of this enzyme therefore leads to decreased peripheral arterial resistance by a combination of these mechanisms. Because they reduce the secretion of aldosterone, ACE inhibitors can produce hyperkalemia. However, they have no impact on urinary calcium excretion. Beta-blockers are one of the most commonly prescribed medications to treat hypertension. In the heart, beta-1 receptor blockade has negative inotropic and chronotropic effects, leading to decreased cardiac output. Nonselective beta-blockers such as propranolol also block beta-2-mediated vasodilation. Combined with reduced cardiac output, this promotes peripheral vasoconstriction. While nonselective beta-blockers still usually lead to net reductions in systolic and diastolic blood pressures, they must be used with caution in patients with peripheral vascular disease because they reduce peripheral circulation. Similarly, since beta-2 activation produces bronchodilation, nonselective beta-blockers may exacerbate respiratory symptoms in patients with reactive airway disease. Selective beta-blockers act only upon beta-1 receptors, thereby improving the side effect profile of this class of antihypertensives. Reductions in cardiac output caused by beta-blockers may lead to decreased renal perfusion, promoting aldosterone production and sodium retention. Beta-blockers do not impact calcium or other electrolyte transport in the kidneys. Calcium channel blockers reduce intracellular transit of calcium in both cardiac and vascular smooth muscle cells. This causes a reduction in peripheral arterial resistance and coronary artery dilation. Some calcium channel blocks are also used as antiarrhythmics because they block atrial-ventricular conduction. Calcium channel blockers have no impact on renal calcium handling.

A 79-year-old man with a past medical history of coronary artery disease, diabetes mellitus, hypertension, smoking, alcohol use, and hyperlipidemia presents with severe chest pain and dyspnea. He appears pale, apprehensive, and diaphoretic. He is in a confused state and agitated. His pulse is weak and tachycardic, with a systolic blood pressure of 60 mmHg. He has a narrow pulse pressure, tachypnea, a weak apical impulse, and significant jugular venous distention. His lungs are free of crackles. Bedside electrocardiogram revealed the following imaging. Question Following stabilization, what health maintenance statement should be provided to this patient?

Correct answer: Smoking and alcohol use should be eliminated. Explanation This patient's exhibits signs and symptoms of cardiogenic shock due to myocardial infarction. The leading risk factor for cardiogenic shock is due to coronary ischemia. The best way to prevent cardiogenic shock is to prevent a heart attack by using the same lifestyle changes used to treat heart disease. These include controlling hypertension by exercising, managing stress, maintaining a healthy weight, and limiting salt and alcohol. Compliance to prescribed antihypertensives is essential. Another important intervention includes maintaining a healthy body weight, which lessens high blood pressure, cardiovascular disease, and diabetes. Regular exercise lower lowers blood pressure, increases high-density lipoprotein (HDL) cholesterol, and improves coronary circulation. Additionally, it controls weight and diabetes and reduces stress. A recommended program involves the gradual work up to 30 minutes of aerobic activity most to all days of the week. Finally, lowering dietary cholesterol and saturated fat in the diet is recommended, as this reduces the risk of heart disease.

Case Ico-delete Highlights A 32-year-old man with no significant past medical history presents to his primary care provider with a 2-month history of increased dyspnea upon exertion, which becomes apparent following walking 10 city blocks. He denies any other associated symptoms such as fever, chills, changes in weight, chest pain, abdominal pain, nausea, or vomiting. He further denies any history of cigarette smoking, occupational risk factors, sick contacts, or recent travel. His physical exam revealed normal vital signs and no distension of his jugular vein. However, there was a prominent right ventricular impulse along the lower-left sternal border associated with a palpable pulmonary artery and a midsystolic ejection murmur at the upper left sternal border that does not vary in intensity with respiration. There is a fixed split second heart sound. The remainder of his examination is normal. Following diagnostic testing, this patient was referred for surgical repair. Question What is the major long-term complication that requires monitoring following surgical repair?

Correct answer: Supraventricular arrhythmia Explanation This patient's presentation represents an atrial septal defect. The major long-term complication following surgical transcatheter device closure of ASD is the development of supraventricular arrhythmias, although the risk is lowered when the ASD is closed in childhood. The persistence of this risk despite relief of right-sided volume overload is thought to be related to incomplete atrial remodeling or due to the presence of the atriotomy scar. Longer follow-up is required to determine whether device closure alters the risk of atrial dysrhythmias. Generally, short-term complications have included device embolization, aortic root or atrial wall perforation, and cardiac tamponade.Mid- and long-term complications include thrombus formation, device erosion into the aortic root, atrial dysrhythmias, and infective endocarditis. Transient ischemic attacks are associated with untreated atrial septal defects. Patients with ASD who have symptoms related to stroke or transient ischemic attack (especially if the age is under 55) or who have hypoxemia should have the ASD closed if no other cause for symptoms is evident.

A 76-year-old man presents with progressive exertional dyspnea associated with substernal chest pain, easy fatigability, and dizziness. Symptoms are exacerbated with walking short distances, and they are relieved with rest. He denies fever, chills, cough, wheezing, pleurisy, calf pain, abdominal complaints, peripheral edema, cigarette, drug, or alcohol use, sick contacts, and travel. His physical exam reveals hypertension and a rough, harsh, low-pitched crescendo-decrescendo systolic murmur beginning after the first heart sound; it is best heard at the second intercostal space in the right upper sternal border. Its intensity is increased toward midsystole; the murmur radiates to both carotid arteries. It is reduced increased upon squatting and decreased during Valsalva strain. Question What regarding the management of this patient is correct?

Correct answer: Surgical intervention provides the only definitive treatment Explanation This patient's presentation is significant for aortic stenosis. The only definitive treatment for aortic stenosis is aortic valve replacement. The development of symptoms due to aortic stenosis provides a clear indication for replacement. For patients who are not candidates for aortic replacement, percutaneous aortic balloon valvuloplasty may provide some symptom relief. In cases of moderate aortic stenosis, moderate-to-severe physical exertion and competitive sports should be avoided. Beta-blockers might be used if the predominant symptom is angina.

A 55-year-old man presents for a routine physical. His vital signs reveal a blood pressure of 160/100 mm Hg. He states that he has never had high blood pressure. After further inquiry, he states that it has been over 10 years since his last checkup. You discuss lifestyle modifications to reduce his blood pressure, and you tell him that you want to see him in 2 weeks. 2 weeks pass, and his blood pressure is still high, so you start him on captopril. 3 weeks later, the patient presents with tachycardia, weakness, and dizziness with postural changes. The history reveals several days of vomiting and diarrhea. Question An EKG would most likely demonstrate what characteristic secondary to the combination of the newly prescribed antihypertensive and the concurrent gastroenteritis?

Correct answer: Tall, peaked T waves Explanation The clinical picture is suggestive of hyperkalemia due to the loss of HCl from vomiting and being placed on an ACE inhibitor. The loss of HCl causes the reabsorption of potassium to counteract the resulting alkalosis. Since ACE inhibitors indirectly reduce the serum aldosterone concentration, the serum potassium concentration may rise. The resulting hyperkalemia results in tall, peaked T waves on EKG. The QRS complex would widen. The flat T wave and U wave would appear with hypokalemia. The P waves in hyperkalemia nearly disappear, or based on potassium levels, they may become wide and flat.

Case Ico-delete Highlights A 4-year-old boy presents with clubbing of his fingers, cyanosis, and a heart murmur; he is easily fatigued. A chest X-ray reveals right ventricular hypertrophy. Cytogenetic analysis of a skin biopsy reveals a 46,XY karyotype. Question What is the most likely diagnosis?

Correct answer: Tetralogy of Fallot Explanation This child is most likely suffering from tetralogy of Fallot. This is the most common cyanotic congenital heart defect; it occurs in approximately 1/1000 live births. There are 4 major components to this condition: Ventricular septal defect Stenosis of the pulmonary artery Overriding aorta (right displacement) Hypertrophy of the right ventricle Most cases of male patients with Down syndrome will have a 47,XY,+21 karyotype (trisomy 21). Most cases of male patients with Edwards syndrome will have a 47,XY,+18 karyotype (trisomy 18). Isolated ventricular septal defects and atrial septal defects also occur, but they would not usually be associated with right ventricular hypertrophy.

You are called to the emergency department at 2 P.M. to see a 44-year-old male patient. He is a 3-pack-a-day, unfiltered cigarette smoker with crushing chest pains. He has a wide-complex, rapid, regular tachyarrhythmia at 160 beats per minute. When you reach his examination room, you note his monitor also reveals evidence of "P" waves at 75 beats per minute. What type of rhythm do his symptoms show?

Correct answer: Ventricular tachycardia Explanation The answer is Ventricular tachycardia (VT). The patient has a wide-complex tachyarrhythmia at a ventricular rate of 160 beats per minute. The most likely diagnosis of a wide QRS tachycardia is VT. This patient's ECG reading is furthermore associated with P waves at a rate of only 75 beats per minute. P waves represent atrial depolarization while the QRS complex represents ventricular depolarization. Such variation between the two rates represents an absence of electrical coordination between the atria and the ventricles (AV dissociation). If AV dissociation is present on an ECG the diagnosis is VT. Having diagnosed ventricular tachycardia, you will endeavor to obtain a more complete history and reasonably complete medical physical examination. It is essential that you provide timely placement of this patient in a medical intensive care unit or a coronary care unit with placement on the critically ill list and advise out-of-town family to travel to visit him. Should the patient survive, then his healthcare providers will need to document his verbalized understanding of their medical professional advice in his chronological record of medical care, which includes the recommendation that he stop smoking cigarettes immediately and abstains from smoking cigars and pipes as well as from chewing tobacco and dipping snuff. Paroxysmal supraventricular tachycardia (PSVT) is a narrow-complex tachyarrhythmia. Sinus tachycardia is also a narrow-complex tachyarrhythmia. Ventricular fibrillation (VF) is a very chaotic tachyarrhythmia with no readily discernible P waves and no readily discernible QRS complexes. Asystole represents a total absence of cardiac contractions and is seen as a flat line on the monitor. Atrial fibrillation is both irregular and narrow-complex. These answers, therefore, are incorrect.

A 74-year-old man presents with a 1-hour history of constant moderate-to-severe "squeezing, pressure, and tight" left-sided chest pain, accompanied by nausea. He has a past medical history of diabetes mellitus type II, hypothyroidism, and hyperlipidemia. He also has a 1-week history of similar, recurrent chest pain of about 10 minutes duration; the pain occurs following exposure to cold weather and consumption of a meal. He denies fever, chills, abdominal pain, diarrhea, cough, pleurisy, and shortness of breath. Question What additional findings would be most consistent with his most likely diagnosis?

Correct answer: Vomiting, diaphoresis, and weakness are associated findings Explanation This patient's most likely diagnosis is acute myocardial infarction. His symptoms prior to this episode represent angina pectoris. Common symptoms associated with myocardial infarction include dyspnea, nausea, vomiting, diaphoresis, and weakness.

A 40-year-old man presents with irregular heartbeats lasting several days. His past medical history is significant for the presence of mitral valve stenosis and atrial fibrillation (AF). He takes beta blockers regularly. His ECG shows atrial fibrillation with an irregular heart rate around 80. Question To prevent further complications, you decide to restore his sinus rhythm by cardioversion and prescribe what treatment?

Correct answer: Warfarin Explanation Beta blockers decreased ventricular response in your patient, but the presence of atrial fibrillation puts him at risk of thromboembolic complications. Thrombosis might already be present in patients with AF lasting more than 2 days. Warfarin is given to prevent the tendency for thrombosis or as secondary prophylaxis (prevention of further episodes) in those individuals that have already formed thrombus. Warfarin should be given for at least 3 weeks prior to cardioversion, and it should be continued for at least 4 weeks after effective cardioversion and return of normal sinus rhythm. Digoxin will be indicated in patients experiencing symptoms of heart failure as first-line therapy for the ventricular rate control. Your patient has no symptoms or signs of heart failure.

A 78-year-old Caucasian man has a past medical history of HIV, hepatic cirrhosis secondary to chronic alcohol abuse and hemochromatosis, obesity, and thiamine deficiency; he presents with a 7-month history of progressive exertional shortness of breath, lower extremity edema, and lightheadedness. His symptoms are improved with rest. The physical examination reveals rales, an elevated JVP, cardiomegaly, S3 gallop rhythm, a high-pitched, blowing holosystolic murmur at the apex, peripheral edema, and abdominal distension that is suggestive of ascites. A bedside EKG notes sinus tachycardia, with nonspecific ST-T wave changes and Q waves. An echocardiogram demonstrates the following imaging. Question What is correct regarding this patient's health maintenance?

Correct answer: Water consumption should be limited as disease progresses Explanation This patient's presentation and the attached echocardiogram findings of dilated chambers and thin walls suggest an underlying diagnosis of dilated cardiomyopathy. Dietary recommendations include sodium and water restrictions. Patients with CHF should restrict their salt intake to less than 2 - 4 g/day. Fluid restriction is only necessary in very late stages of the disease. An internist, intensivist, or a cardiologist referral should be made when a patient is diagnosed with dilated cardiomyopathy for the first time or for continued inpatient treatment or monitoring. Cardiology consultation may also be indicated for echocardiography in the emergency department. A cardiothoracic surgeon should be consulted for all patients with NYHA functional class III-IV dilated cardiomyopathy for possible surgical intervention (e.g., partial left ventriculectomy, heart transplant). Patients should be encouraged to exercise moderately; deconditioning is a very common cause of dyspnea. Cardiac rehabilitation has been shown to improve patient outcomes.

A 60-year-old man with a history of recurrent sinus infections presents with hemoptysis and hematuria. Physical examination shows a temperature of 101 degrees Fahrenheit, a blood pressure of 145/85 mmHg, decreased breath sounds on his right lower lobe, and palpable purpura on his bilateral lower legs. No warm or swollen joints were noted. Anti-neutrophil cytoplasmic antibody Positive Question What is the most likely diagnosis?

Correct answer: Wegener's Granulomatosis Explanation This patient most likely has Wegener's Granulomatosis. Wegener's Granulomatosis is a small vessel vasculitis of unknown pathogenesis that causes inflammation and necrotizing granulomas in the upper and lower respiratory tract and kidneys, as well as rash; 15 - 50% of patients have cutaneous involvement (Brenner, chapter 31). Palpable reddish-purple lesions on the lower extremities should raise suspicion of vasculitis; macules and nodules, may also erupt. Some 50 - 95% of patients with Wegener's will eventually have renal involvement (Brenner), which may range from mild findings to fulminant dialysis-requiring nephritis. On renal biopsy, focal segmental necrotizing glomerulonephritis is noted; crescents may be seen. Immune staining is generally negative. Treatment involves immunosuppressive therapy, as is the case with SLE, and dialysis as needed. In some resistant cases, plasmapheresis may be helpful. Patients with Wegener's are often Anti-Neutrophil Cytoplasmic Antibody (ANCA) positive. Patients with Goodpasture's syndrome typically present with a triad of glomerulonephritis, pulmonary hemorrhage, and anti-glomerular basement membrane (GBM), type IV collagen antibodies. Many patients with Goodpasture's syndrome are ANCA positive. SLEis a systemic inflammatory disease that causes a variety of signs and symptoms affecting many organs. Renal involvement is frequent in SLE and may include a variety of glomerular diseases, causing hematuria, proteinuria, and urinary casts

PAC-PRA (plasma aldosterone concentration: plasma renin activity) ratio

In 90% of cases, primary hyperaldosteronism is usually due to an aldosterone-producing adenoma or bilateral idiopathic adrenal hyperplasia, and it typically occurs in adults ages 20 to 60. Secondary hyperaldosteronism is typically due to overproduction of renin with activation of the renin-angiotensin-aldosterone system. Renin overproduction may be triggered by renal hypoperfusion or a renin-secreting tumor. Hyperaldosteronism should be suspected in patients with hypertension and hypokalemia, severe hypertension resistant to treatment, hypertension with an adrenal mass, or hypertension with onset prior to age 30. The aldosterone-renin ratio, also known as the PAC-PRA (plasma aldosterone concentration: plasma renin activity) ratio, is the best screening test for suspected primary hyperaldosteronism. It is performed by obtaining paired measurements of PAC and PRA in a random morning blood sample, preferably after correction of potassium and prior to starting therapy with aldosterone or eplerenone. An elevated aldosterone-renin ratio in conjunction with an elevated absolute aldosterone level strongly suggests, but is not diagnostic of, primary hyperaldosteronism. A definitive diagnosis of primary hyperaldosteronism can subsequently be established with an aldosterone suppression test. If hyperaldosteronism is confirmed, then adrenal CT should be performed to identify possible adrenal adenoma or hypertrophy.

Orthostatic blood pressure recordings

Orthostatic blood pressure recordings (blood pressure when lying, sitting, and standing) can quickly identify the presence of orthostatic hypotension (low blood pressure upon standing). Orthostatic hypotension may be due to several causes: dehydration, blood loss, hypoglycemia, autonomic neuropathy, adrenal insufficiency, anemia, medications affecting blood pressure, and some cardiac conditions such as severe aortic stenosis or severe bradycardia.


Set pelajaran terkait

Demographic and Social Structure

View Set

Chapter 9: New Product Development

View Set

Chapter 11 wound healing, sutures, needles and stapling devices

View Set